Download as pdf or txt
Download as pdf or txt
You are on page 1of 62

Solutions for SM1001920

Exercise – 1 10. When divided by two, the second equation is 2x + 3y = 121/2.


(Simple Equations) Since both are inconsistent the number of solutions will be
zero. Ans: (0)
Solutions for questions 1 to 50:
1. Since x term has to be eliminated in order to find the value 11. Let the worker be absent for x days. He works for (30 – x) days
of y, multiply the first equation by 3 and second equation by 2. (3 – x)120 – 30x = 2700 ⇒ x = 6 Choice (D)
We get,
12. Let the costs of an apple, a banana and a pineapple be `a,
6x + 9y = 57 ------- (1)
`b, `p respectively.
6x + 4y = 32 ------- (2)
3a + 4b + 5p = 50 ------ (1)
By subtracting the second equation from the first.
6a + 2b + 10p = 82 ------ (2)
5y = 25 y = 5 Ans: (5)
2x(1) − (2) ⇒ 6b = 18 ⇒ b = 3
2. –2(x – y) = 3x + 3y + 6 – 2x – 4y ∴2b = 6 Ans: (6)
–2(x – y) = (x – y) + 6
13. Let the fraction be x/y. Given that
–3(x – y) = 6
x – y = 6/–3 = –2 Ans: (–2) x −5
= 1/3 ⇒ 3x – 15 = y ------ (1) and
y
3. Let the three consecutive integers be x – 1, x and x + 1
x – 1 + x + x + 1 = 72 ⇒ x = 24 x
= 1/2 ⇒ 2x – y = –1 ----- (2)
The three numbers are 23, 24, and 25 y −1
(23)2 + (25)2 = 1154 Choice (D)
Solving (1) and (2), 3x – 15 = 2x + 1
4. Let the numbers be x and y x = 16
x + y = 9(x – y) ⇒ 8x = 10y ∴The fraction is 16/33 Choice (D)
4x = 5y
But x = y + 33 14. Let the ten’s digit be x, the units digit = 2x + 1. Given that
4(y + 33) = 5y ⇒ y = 132 x + 2x + 1 = 10 ⇒x=3
x = 165 ∴ The two digit number is 37. Choice (C)
Sum = x + y = 297 Ans: (297)
15. Let the income and the expenditure of B be `x and `y
5. Let the fraction be x/y respectively.
x +1 ∴Income of A = `2x and expenditure of A = `3y
= 1/4 4x – y = –5 ----- (1) ⇒ Given 2x − 3y = 10,000 - (1) and
y −1
x − y = 10,000 - (2)
x+2
= 1/4 4x – y = –5 ------ (2) (1) − (2) ⇒ y = 10,000
y +3 ∴ x = 2000
Since two identical equations are obtained, we don’t have a ∴Income of A = `40,000 Choice (A)
unique solution. Choice (D)
16. Let the number of apples in the two baskets be x and y.
6. Let x and y be the unit's digit and the ten’s digit of a number Given that y – 4 = x + 4
respectively.
y+4
x + y = 9 ------ (1) ⇒ y – x = 8 and x – 4 = +4
The number is in the form of 10x + y 2
10y + x = 10x + y + 9 ⇒ 2x – y = 20 ----- (2)
9y – 9x = 9 Solving the equations, we get
y – x = 1 ------ (2) x = 28, y = 36 Choice (C)
Solving equations (1) and (2), we get
17. A’s salary = B’s salary + 3000
x = 4, y = 5
Let x = y + 3000
The number is 45 Ans: (45)
But, x + y = 27000
7. Let the number of 25np and one rupee coins be 'a' and ‘b’. y + 3000 + y = 27000
Given a + b = 56 ------- (1) y = 12,000
and 25a + 100b = 3500-------(2) x = 15,000
By 100 × (1) − 25 × (2), we get 75a = 2100 Let the expenditure be `p for each of them
∴a = 28 Choice (B) A’s savings = 15,000 – p
B’s savings = 12,000 – p
8. Let the man’s present age and the daughter’s present age 15,000 – p = 2(12000 – p) + 1000 ⇒ p = `10,000
be x years and y years respectively. 2p = `20,000 Ans: (20,000)
x – 3 = 7(y – 3)
x – 7y = –18 ------ (1) 18. Let the number of marbles with P, Q and R be p, q and
x + 2 = 4(y + 2) r respectively.
⇒ x – 4y = 6 ------ (2) p + q + r = 210
Solving both the equations, y = 8 years Ans: (8) 1 1
p= (q + r) = (210 – p)
9. Let the number of horses and peacocks be x and y 4 4
respectively. 4p = 210 – p
x + y = 35 ------- (1) 5p = 210; p = 42
Each horse has four legs and each peacock has two legs. 3 3
4x + 2y = 88 q= (p + r) = (210 – q)
4 4
⇒ 2x + y = 44 ------ (2) 4q = 3(210 – q) = 630q
Solving the equations (1) and (2), we get x = 9, y = 26. 7q = 630; q = 90
There are 26 peacocks Choice (A) ∴ r = 210 – (p + q) = 78. Choice (D)
 Triumphant Institute of Management Education Pvt. Ltd. (T.I.M.E.), 95B, Siddamsetty Complex, Park Lane, Secunderabad – 500 003.
All rights reserved. No part of this material may be reproduced, in any form or by any means, without permission in writing.
This course material is only for the use of bonafide students of Triumphant Institute of Management Education Pvt. Ltd. and its
licensees/franchisees and is not for sale. (62 pages) (alcq/alcr) SM1001920.Sol/1
n ∴ a is divisible be 8
19. Let the fraction be 9 ≥ a ≥ 1.
d
d = h + 17 ------ (1) ∴ a must be 8 and b = 1
n −1 2 ∴ ab = 81
= ∴ ab has only 1 possible value. Choice (B)
d 5
n −1 2 28. Let the amounts with Ganesh and Harish be `g had `h
=
n + 17 5 respectively.
5(n – 1) = 2(n + 17) g + h = 180 ----- (1)
5n – 5 = 2n + 34 if Ganesh gave `20 to Harish, they would have `(g – 20)
3n = 39 and `(h + 20) respectively.
n = 13 g – 20 = h + 20 + 20
(1) ⇒ d = 30, (1) ⇒ g = 180 – h ⇒180 – h – 20 = h + 20 + 20
n 13 ⇒ 120 = 2h ⇒ h = 60. Ans: (60)
= . Choice (C)
d 30 29. Let the number of cows and hens on the farm be c and h
respectively c + h = 30
20. Number of subsequent copies made = 10000 – 3000 = 7000 Each cow has 4 legs while each hen has 2 legs
Total cost of making them = `7000z. = 4c + 2h
Total cost of making the first 10000 copies = 2(c + h) + 2c = 60 + 2c
= `(9000 + 7000z) As c > 0, 2c > 0
15510 = 9000 + 7000z ∴ 60 + 2c > 60
6510 = 7000z Only Choice (D) violates this condition. Choice (D)
z = 0.93. Ans: (0.93)
30. x + y – z = 4 ------ (1)
21. Original number = TU
x – y + z = 2 ------ (2)
Number formed = TU4
= 100T + 10U + 4. Choice (C) (1) + (2) ⇒ 2x = 6 ⇒ x = 3. Choice (C)
2 2
31. (z + 3) = z + 6z + 9
22. 9x + 11y = 40 and 36x + 44y = 120 (z – 3)2 = z2 – 6z + 9
Simplifying the second equation we get 9x + 11y = 30
1
∴ 9x + 11y = 40 and 9x + 11y = 30 ∴ z2 + 6z + 9 = 6 + z2 – 6z + 9 ⇒ 12z = 6 ⇒ z = .
i.e 40 = 30 which is not possible. 2
∴ No solution exists. Choice (B) Choice (A)

23. Let the costs of each chair and each table be `C and `T 32. Let the costs of each vada and each dosa be `v and `d
respectively. respectively.
3C + 2T = 1300 ------- (1) 6v + 5d = 80 --------- (1)
2C + 3T = 1200 -------- (2) 6(v – 2) + 5(d + 1) = 73
∴ 6v – 12 + 5d + 5 = 73
Normal method: ∴ 6v + 5d = 80 ---------- (2)
2(1) ⇒ 6C + 4T = 2600 ------ (3) (1) and (2) are the same equation. We have only one
3(2) ⇒ 6C + 9T = 3600 ------ (4) equation with two unknowns v and d. ∴ d cannot be found.
(4) – (3) ⇒ 5T = 1000 Choice (D)
T = 200
Alternate method: 33. Let the shares of A, B and C (in lakhs of rupees) be a, b,
(1) + (2) ⇒ 5(C + T) = 2500 ⇒ C + T = 500 ------ (3) and c respectively.
(2) – (1) ⇒ T – C = – 100 ------- (4) a+c
a + b + c = 24 and b = or 2b = a + c
(3) + (4) ⇒ 2T = 400 2
T = 200. Ans: (200) Adding b on both sides
24. Let the number be ab. 3b = a + b + c, 3b = 24 or b = 8. Choice (B)
b = a + 8 and a ≥ 1
34. Let the present ages of the father and his son be F years
If a = 1, b = 9
and S years respectively.
If a > 1, b > 9. As b is a digit, a cannot be > 1
F + 10 = 2(S + 10) + 10
∴ a = 1 and b = 9 i.e. ab = 9
⇒ F + 10 = 2S + 20 + 10
∴ ab has only 1 possibility. Choice (A)
⇒ F = 2S + 20
Let us say x years hence the father will be twice his son’s
25. Bala would have the maximum possible amount if he has
age
the maximum number of 10-rupee coins. He must have at
least one 5-rupee coin. F + x = 2(S + x) ⇒ 2S + 20 + x = 2S + 2x ⇒ x = 20.
Choice (A)
∴ He must have at most four 10-rupee coins.
∴ He would have the maximum possible amount when he
35. Let the present ages of X and Y be x years and y years
has four 10-rupee coins. This value (in `)
respectively. Suppose z years ago X was y years old. Y
= (4) (10) + (1) (5) = 45. Choice (D)
x
must have been years old at that time. Also ages of X
26. Let the number of pens with x and y be x and y 5
respectively. and Y must have been (x – z) years and (y – z) years
x y 2x + y respectively.
+ = 18 ⇒ = 18
2 4 4 x
x – z = y and y – z =
⇒ y = 72 – 2x 5
x > 0, ∴2x > 0, ∴y < 72 x
Only Choice (D) violates this condition. Choice (D) z=x–y=y–
5
27. Let T be ab. ab = 9(a + b) 3x x 5
⇒ =y⇒ = . Choice (A)
10a + b = 9a + 9b ⇒ a = 8b 5 y 3
Triumphant Institute of Management Education Pvt. Ltd. (T.I.M.E.) HO: 95B, 2nd Floor, Siddamsetty Complex, Secunderabad – 500 003.
Tel : 040–27898195 Fax : 040–27847334 email : info@time4education.com website : www.time4education.com SM1001920.Sol/2
36. Let the share of the eldest son be `e 44. Let the present ages of the man and his son be F years
Combined share of the other sons = `(1200 – e) and S years respectively.
5 F – 5 = 3 (S – 5) and F + 15 = 2 (S + 15)
Given e = (1200 – e) ⇒ 7e = 6000 – 5e ⇒ F – 5 = 3S – 15 and F + 15 = 2S + 30
7
⇒ 12e = 6000 ⇒ e = 500 Ans: (500) ⇒ F = 3S – 10 and F = 2S + 15
⇒ F = 3S – 10 = 2S + 15 ⇒ S = 25 Ans: (25)
n 45. Let the number of marbles with Mohan and Sohan be M
37. Let the fraction be and S respectively.
d
n = d + 13 ------------ (1) M + S = 200 ------------------ (1)
If Mohan gave 20 marbles to Sohan, then he and Sohan
n 5
= ------------- (2) would have M – 20 marbles and S + 20 marbles respectively.
d+8 4 M – 20 = S + 20
d + 13 5 ∴ M – S = 40 --------------- (2)
∴ = ⇒ 4 (d + 13) = 5 (d + 8) (1) + (2) ⇒ 2M = 240 i.e., M = 120 Choice (D)
d+8 4
⇒ 4d + 52 = 5d + 40 ⇒ d = 12 46. Let the number of `1 coins and ` 2 coins with
(1) ⇒ n = 25 Ravi be x and y respectively.
n 25 x + y = 12 -------------- (1)
∴ = Total worth of the coins = `(x + 2y)
d 12
x + 2y = 20 ------------ (2)
Alternate method: (2) – (1) ⇒ y = 8 Ans: (8)
Among the choices, only Choice (D) satisfies the given
47. Let the costs of each pen and each eraser be `p and
conditions. Choice (D)
`e respectively.
38. Number of subsequent copies made = 7500 – 1500 = 6000 2p + 3e = 17 – (1)
Total cost of making them = `6000y. 2(p + 4) + 3 (e – 2) = 17 + 2
Total cost of making the first 7500 copies = `(3000 + 6000 y) ∴ 2p + 8 + 3e – 6 = 19
7980 = 3000 + 6000 y ⇒ 6000y = 4980 ⇒ y = 0.83 ∴ 2p + 3e = 17 – (2)
Choice (A) (1) and (2) represent the same equation. We have only one
equation with two unknowns p and e.
39. 5x + 7y = 24 and 15x + 21y = 72. ∴ p cannot be found. Choice (D)
Simplifying the second equation we get 48. (y + 2)2 = y2 + 4y + 4 ; (y – 2)2 = y2 – 4y + 4
5x + 7y = 24 which is the first equation.
∴ (y + 2)2– (y – 2)2 = y2 + 4y + 4 – (y2 – 4y + 4)
∴ We have only one equation with two unknowns x and y. = 4y – (–4y) = 4y + 4y = 8y = 2
∴ The number of solutions of x and y is infinite. y = 1/4 Choice (C)
Choice (A)
49. Let the number of one-rupee coins and two-rupee coins
40. Let the costs of each chair and each table be `C and `T with Ravi be x and y respectively.
respectively. x + y = 12 -------------------- (1)
4C + 5T = 4100 ------------ (1) 2y – x = 12 ------------------- (2)
5C + 4T = 4000 ------------ (2) (1) + (2) ⇒ 3y = 24 i.e., y = 8 Choice (B)
Normal method
4(1) ⇒ 16C + 20T = 16400 – (3) 50. Let the number of marbles with P and Q be p and q
respectively.
5(2) ⇒ 25C + 20T = 20000 – (4)
(4) – (3) ⇒ 9C = 3600 q
3p + q = 24 ⇒ p = 8 –
C = 400 3
Alternate method: q>0
(1) + (2) ⇒ 9C + 9T = 8100 ∴p<8
⇒ C + T = 900 ------------------- (3) Only Choice (D) satisfies this condition. Choice (D)
(2) – (1) ⇒ C – T = – 100 -------- (4)
Exercise – 2
(3) + (4) ⇒ 2C = 800 or C = 400 Ans: (400)
(Ratio, Proportion and Variation)
41. Let the two-digit number be ab.
Solutions for questions 1 to 50:
ab = 10 (a + b)
10a + b = 10a + 10b ⇒ b = 0
2x + 3 y 1
∴ ab = a0 1. = ⇒ 4x + 6y = 3x + 7y ⇒ x = y
a has 9 possible values. 3x + 7y 2
∴ ab has 9 possible values. Choice (A) ∴ x/y = 1/1 Choice (B)
2. Let 2x = 4 k where k is non-zero x = 2 k
42. Let the number of cars and motorbikes be C and M respectively.
3y = 9 k, y = 3 k
C + M = 40 ------------- (1)
5 x + 7 y 5 x 2k + 7 × 3k 31
Each car has 4 tyres and each motorbike has 2 tyres. = = Choice (C)
∴ Total number of tyres = 4C + 2M 7 x + 5 y 7 × 2k + 5 × 3k 29
4C + 2M = 140
2C + M = 70 ------------ (2) 3. Let p = 3x
l=y
(2) – (1) ⇒ C = 30 Ans: (30)
q = 2x m = 3y
3pl − 2mq 3 × 3 x × y − 2 × 3 y × 2x −3 xy
43. Let the number be abc = = = −1
a + b + c = 12 mq − pl 3 y × 2x − 3 x × y 3 xy
a c Choice (D)
b= =
2 3 4. a:b=3:2
∴ a = 2b and c = 3b b:c=5:4
a + b + c = 6b = 12 ---------------
b=2 Choice (B) a : b : c = 3 x 5 : 5 x 2 : 2 x 4 = 15 : 10 : 8 Choice (B)

Triumphant Institute of Management Education Pvt. Ltd. (T.I.M.E.) HO: 95B, 2 Floor, Siddamsetty Complex, Secunderabad – 500 003.
nd

Tel : 040–27898195 Fax : 040–27847334 email : info@time4education.com website : www.time4education.com SM1001920.Sol/3


5 4 14 1 f E f
5. a/d = a/b x b/c x c/d = × × = Choice (D) 18. E α ⇒ 1 = 1
7 15 8 3 w E 2 f2
3 30
= ⇒ E2 = 2 cm Ans: (2)
6. Mean Proportional of a and b = ab i.e. 4 × 25 = 10 E2 20
Ans: (10)
a 5 5
7. Let the numbers be x, 3x, 5x. 19. a : b = 5 : 6 ⇒
= ⇒a= b
b 6 6
x2 + 9x2 + 25x2 = 315
b 4 4
x2 = 9 ⇒ x = 3 b:c=4:3⇒ = ⇒b= c
∴ The largest number is 5x = 15 Ans: (15) c 3 3
5 4  10
8. Let the amounts received by A, B and C be 3x, 4x and x a=  c  = c.
6 3  9
respectively. Given x = 3000
Total amount = 3x + 4x + x = 8x = 8 x 3000 = `24,000 10 4
∴a:b:c= c: c:c
Choice (C) 9 3
= 10 : 12 : 9 Choice (D)
9. Ratio for values of one rupee, two rupee and five rupee
notes will be 1 x 2 : 2 x 3 : 5 x 5 = 2 : 6 : 25 20. When two or more ratios are equal each of them is equal to
25 the sum of all the numerators divided by the sum of all the
Amount in `5 notes = × `66 = `50
2 + 6 + 25 denominators provided the sum of the denominators is not zero.
∴Number of 5 rupee notes is 10 Ans: (10) b a c
∴ If = = ,
a+b−c a+c −b b+c −a
10. Let the present ages be 5x, 3x b+a+c
5 x + 12 3 each of them equals =1
= = 3/2 a+b−c +a+c −b+b+c −a
3 x + 12 2 When a + b + c ≠ 0 each ratio can be 1. Choice (A)
⇒ x = 12
⇒ Age of man = 5x = 60 years Choice (B) 21. Let the present ages of Anil and his wife be a years and w
years respectively.
11. Let the number of boys and girls be 3x and 2x respectively a + w = 88 ------- (1)
3x − 5 1 a+8 7
= ⇒ x = 10 =
2x + 5 1 w+8 6
Number of boys = 30 Ans: (30) (1) ⇒ w = 88 – a
a+8 7
12. Let the numbers of `20, `50 and `5 be 4x, 3x, 2x ∴ =
88 − a + 8 6
respectively.
4x(20) + 3x(50) + 5(2x) = 240x = 480 6a + 48 = 7(6 – a) ⇒ 6a + 48 = 672 – 7a
x=2 ⇒ 13a = 624 ⇒ a = 48. Ans: (48)
Number of `50 notes = 3x = 6. Choice (D)
22. 2.7p = 0.09q ⇒ 30p = q
13. Let the income ratio be 7x : 5x q + 6p 36p 3
∴ = = . Choice (B)
Let the expenditure ratio be 5y : 4y q − 6p 24p 2
---------------------
Savings ratio 7x – 5y : 5x – 4y 23. Let the first and second numbers be 9x and 11x respectively.
7x – 5y = 2000 - - I
9x + 6 11
5x – 4y = 1000 - - II =
Solving I and II x = 1000 y = 1000 11x − 6 9
Income of Vijay = `5000 Ans: (5000) 81x + 54 = 121x – 66 ⇒ 120 = 40x
x = 3 ⇒ 9x = 27. Choice (B)
14. 2x + 3y = 3 (5x –3y)
⇒13x = 12y ----> x/y = 12/13 Choice (A) 24. Let the monthly incomes of A and B be `5x and `4x
15. Given W = 2S and S = 2D respectively. Let the monthly expenditure of A and B be `4y
W:S:D=4:2:1 and `3y respectively.
Let shares be of wife, son and daughter be 4x, 2x and x Monthly savings of A = `(5x – 4y)
respectively. Given that Monthly savings of B = `(4x – 3y)
4x – x = 9,00,000 ⇒ x = 3,00,000 4x – 3y = 2(5x – 4y)
Total worth of property = 7x = `21,00,000 Choice (D) 4x – 3y = 10x – 8y
5y = 6x
1 X1 Y2 20 6x
16. Given X α ⇒ = = Y=
Y X2 Y1 10 5
18 x
Y2 Required ratio = 4x : 3y = 4x : = 10 : 9. Choice (C)
= 2 ⇒ Y2=100 Ans: (100) 5
25
25. Let the first and second numbers be 2x and 7x respectively.
1 7x + 12 = 3(2x + 12) ⇒ 7x + 12 = 6x + 36
17. P α Q and P α ⇒ x = 24 ⇒ 7x = 168. Ans: (168)
3 R
P1 3 R1 Q1 26. Let x = 4a and y = 9a
⇒ =
P2 3 R 2 Q2 x = 2 a and y =3 a

10 × 3 8 16 4 x +5 y 4(2 a) + 5(3 a) 23 a 23
= ⇒ P2 = 10 Choice (A) = = = .
P2 × 3 27 36 6 x +7 y 6(2 a ) + 7(3 a) 33 a 33
Choice (D)
Triumphant Institute of Management Education Pvt. Ltd. (T.I.M.E.) HO: 95B, 2nd Floor, Siddamsetty Complex, Secunderabad – 500 003.
Tel : 040–27898195 Fax : 040–27847334 email : info@time4education.com website : www.time4education.com SM1001920.Sol/4
27. Let the income of P and Q be `2a and `3a respectively. A
Let the expenditure of P and Q be `5b and `6b 33. For A to vary directly with B, must be a constant.
B
respectively. Choice (A)
Method 1
A
Savings of P = `(2a – 5b) saving of Q = `(3a – 6b) 3 = a constant
2a − 5b 2.5b B
Part of his income that P saves = =1– A
2a a ∴ = (a constant)3 = a constant
3a − 6b 2b B
Part of his income that Q saves = =1– ∴ A varies directly with B.
3a a
Choice (B)
b b 3
2.5 >2 A
a a   = a constant.
b b B
∴ – 2.5 <–2 A
a a = 3 a constant = a constant.
B
b b
∴ 1 – 2.5 <1–2 ∴ A varies directly with B
a a Choice (C) follows. Choice (C)
∴ Q saves a greater part of his income.
Method 2 P2 R
The person who saves a greater part of his income will 34. P2 : Q = R : P or =
Q P
spend a lesser part of his income.
∴ P3 = RQ and R = 27Q2
Parts of their incomes spent by P and Q are
∴ P3 = RQ = 27Q3
5b 6b b b
and respectively i.e. 2.5 >2
2a 3a a a P= 3
(27Q)3 = 3Q
∴ Q spends a lower part of his income and hence saves a P : Q = 3 : 1. Choice (C)
greater part of his income. Choice (B)
p 3
e 35. p : q = 3 : 4 ⇒ =
28. Let =k q 4
f
3
a c e ⇒p= q
= = =k 4
b d f
q 6
a = bk, c = dk and e = fk. q:r=6:7⇒ =
r 7
a+c bk + dk K(b + d)
= = =K 6
b+d b+d b+d ⇒ q= r.
7
a+e c+e
Similarly and equal K 3 6  9
b+f d+f p=  r = r
4  7  14
∴ Choice (C) follows. Choice (C)
9 6
∴p:q:r= r: r:r
29. Let the number of married employees and unmarried 14 7
employees be 5x and 7x respectively. = 9 : 12 : 14 Choice (C)
Number of employees in the office = 5x + 7x = 12x
This must be divisible by 12. Only Choice (D) violates this 36. Let the ages of Raj and his wife 15 years ago be 3x years
condition. Choice (D) and 2x years respectively.
Their present ages are (3x + 15) years and (2x + 15) years
30. If A received `10 less, total amount with A, B and C would respectively.
have been `1210 – 10 = `1200 (3x + 15) + (2x + 15) = 130 ⇒ 5x = 100 ⇒ x = 20
A, B and C would have divided this among themselves in ∴3x + 15 = 75 Ans: (75)
the ratio 3 : 4 : 5. A would have then received
3 37. When two or more ratios are equal, each of them must be
(1200) = `300
3+4+5 equal to the sum of their numerators divided by the sum of
their denominators, provided the sum of their denominators
∴ A’s share (in `) = 300 + 10 = 310. Ans: (310)
is not zero.
p + q−r q+r −p p+r −q
31. Let the total number of stamps be x. Ratio of the number of ∴ If = = , each of them equals
p q r
stamps with P, Q and R
1 1 1 3 2 1 (p + q − r ) + (q + r − p) + (p + r − q)
= : : = : : = 3 : 2 : 1. The total is 6 parts. p+q+r
2 3 6 6 6 6
If this ratio had been 2 : 3 : 6, the total is 11 parts. p+q+r
i.e., = 1 when p + q + r ≠ 0
3 : 2 : 1 = 33 : 22 : 11 p+q+r
2 : 3 : 6 = 12 : 18 : 36
∴ each ratio can be 1. Choice (A)
∴P and Q would have less and R would have more.
Choice (C)
38. 1.6 a = 0.08b ⇒ 20a = b.
b + 2a 22a 11
32. Let p + q – r, p + r – q and r + q – p be 2k, 3k and 4k ∴ = = Choice (C)
respectively. Their sum = p + q + r = 2k + 3k + 4k b − 2a 18a 9
∴ p + q + r = 9k
(p + q + r) – (r + q – p) = 5k ∴ 2p = 5k 39. Let the numbers be 10x and 13x.
p + q + r – (p + r – q) = 6k ∴ 2q = 6k 10 x − 90 7
= ⇒ 100x – 900 = 91x – 630 ⇒ 9x = 270
p + q + r – (p + q – r) = 7k ∴ 2r = 7k 13 x − 90 10
p : q : r = 2p : 2q : 2r = 5 : 6 : 7. Choice (A) ⇒ x = 30
The smaller number = 10x = 300 Ans: (300)
Triumphant Institute of Management Education Pvt. Ltd. (T.I.M.E.) HO: 95B, 2nd Floor, Siddamsetty Complex, Secunderabad – 500 003.
Tel : 040–27898195 Fax : 040–27847334 email : info@time4education.com website : www.time4education.com SM1001920.Sol/5
40. Let the monthly incomes of X and Y be `8a and `5a 45. The data is tabulated below
respectively. Let their monthly expenses be `5b and `3b A B C Total
respectively. Intended 2 3 4 9
Monthly savings of X = `(8a – 5b) 1 1 1
Monthly savings of Y = `(5a – 3b) Actual
2 3 4
2
8a – 5b = 5a – 3b = 200 ⇒ 3a = 2b ⇒ a = b Actual 6 4 3 13
3 Multiplying the intended row by 13 and the actual by 9 we get
2  A B C Total
∴ 8  b  – 5b = 200 Intended 26 39 52 117
3 
Actual 54 36 27 117
b
= 200 i.e., b = 600 and a = 400 ∴A gained, while B, C lost Choice (D)
3
∴8a = 3200 Ans: (3200) 46. P, Q, R, S are in proportion ∴ PS = QR.
Choice (A) ⇒ PS = QR ; Choice (B) ⇒ QR = PS
41. Let the present ages of P and Q be 5x years and 4x years ∴both (A) & (B) are true Choice (C)
respectively.
5 x + 10 5(x + 2) 47. Let the shares of P, Q and R be `p, `q and `r respectively.
Ratio of their ages 10 years hence = =
4 x + 10 4 (x + 2) + 2 p + q + r = 30 lakhs
p – q = q – r i.e., p + r = 2q – (1)
which is always less than 5/4. Only Choice (D) violates this
Adding q on both sides, p + q + r = 3q
condition. Choice (D)
∴ 3q = 30 lakhs ; q = 10 lakhs. Ans: (10)
42. Choice (A)
48. From the previous solution,
x
If x varies directly with y , = constant. (1) ⇒ p + r = 20 lakhs.
y Also p = 4r ∴ p + r = 5r = 20 lakhs.
Let this constant be k. r = 4 lakhs. Ans: (4)
x
=k 49. x varies directly with y2
y x
∴ 2 = a constant.
x2 y
Squaring both sides, = k2
y Let this constant be K.
k2 is a constant. 12 x
When y = 2, x = 12. ∴ K = =3;∴ =3
x 2 22 y2
= a constant.
y x
When y = 6, =3
∴ x2 varies directly with y. y2
Choice (A) must be true. ∴ x = 108 Ans: (108)
Choice (B):
50. Let the present ages of P and Q be 4x years and 3x years
1 1 respectively.
If x varies inversely with , x = a constant.
y y 4x + 7 9
=
Let this constant be k. 3x + 7 7
x y 1 28x + 49 = 27x + 63
=k; = x = 14 ; 4x = 56 Choice (B)
y x k

 1  1 Exercise – 3
y. = (Percentages, Profit & Loss, Partnerships)
 x k
 
1 Solutions for questions 1 to 75:
is a constant.
k 1. 23.33% = 33.33% – 10%
 1  = 1/3 – 1/10 = 7/30 Choice (A)
∴ y.  = a constant.
 x
  2. 162/3% of 600 = 12½% of x
1 ⇒ 1/6 × 600 = 1/8 × x
∴ y varies inversely with . ⇒ x = 800 Ans: (800)
x
Choice (B) must be true. Choice (C) 3. Let the number be 100.
∴ New number is 140 (as it is increased by 40%)
43. Let a + b = 3K --------- (1), b + c = 4K ------- (2) and 140 → 175
c + a = 5K--------------- (3) 40 → ?
a + b + b + c + c + a = 3K + 4K + 5K 40 x 175
2 (a + b + c) = 12K ?= = 50 Choice (C)
140
a + b + c = 6K
(1) ⇒ c = 3K
4. 27% of 600 – 15% of 840
(2) ⇒ a = 2K
= 27 × 6 – 15 × 84/10
(3) ⇒ b = K
= 162 – 126 = 36 Choice (B)
∴a:b:c=2:1:3 Choice (B)
5. The ratio of male to female employees is 2 : 3
44. Let the number of students in P, Q and R be 3x, 7x and 8x
∴ The percentage of male employees
respectively.
8x – 3x = 20 ⇒ 5x = 20 ⇒ x = 4 3
= × 100 = 60% Choice (B)
7x = 28 Ans: (28) ( 2 + 3)

Triumphant Institute of Management Education Pvt. Ltd. (T.I.M.E.) HO: 95B, 2nd Floor, Siddamsetty Complex, Secunderabad – 500 003.
Tel : 040–27898195 Fax : 040–27847334 email : info@time4education.com website : www.time4education.com SM1001920.Sol/6
6. Let the third number be 100. 18. Expenditure is given by price x consumption
∴ First and second numbers are 150 and 180 respectively. Let the initial price be `100 and consumption be 100 units.
30 is equivalent to 120 ∴ Initial expenditure = 100 × 100 = `10,000
100 × 120 New price = `50 (50% decrease)
∴ 100 is equivalent to × 100 = 400
30 10000
∴ New consumption = = 200
Ans: (400) 50
7. Let the fourth number be 100. ∴ Percentage increase in consumption
∴ First, second and third numbers will be 80, 75 and 50 200 − 100
× 100 = 100% Choice (D)
respectively. 100
∴The first number is more than the third number by
30 19. As 60% are boys, 40% will be girls.
× 100 = 60% Choice (A) Given 40 is equivalent to 300
50
60 × 300
8. Let the price of the cell phone in 2007 be `100 ⇒ 60 is equal to = = 450 are boys
40
∴ In 2010 its value will be `80 Given half of the boys are in primary classes i.e., 450/2 = 225
80 → 3240 ∴Required difference = 300 – 225 = 75 Ans: (75)
100 → ?
100 × 3240 20. First number should be increased by
?= × 100 = `4050 Ans: (4050)
80 9−4
× 100 = 125%
4
9. Let the total number of students in 2009 be 100. So that both the numbers will be equal. Choice (A)
∴ In 2010, number of students = 1331/3
Given, 1331/3 – 6000 21. Let the maximum marks be x.
100 × 6000 Given, 20% of x = 30 ⇒ x = 150 Ans: (150)
100 – = 4500 Choice (B)
400/3
22. The percentage change in the area of the rectangle
10. 60% of 120 + x = 120% of x  30 × 10 
=  30 − 10 −  % = 17% Choice (D)
⇒ 72 = 20% of x ⇒ x = 360
 100 
∴ Value of x/10 = 360/10 = 36 Ans: (36)
11. Let the income of the man be `100 23. Let the number be x. Given, 80% of x is 192
∴ Expenditure of the man will be `60 80
x = 192 ⇒ x = 240 Ans: (240)
∴ His savings is `40 100
When saving is `40, income is `100
When saving is `3,000, income is 24. Let the population be 100.
3,000 × 100 Given, that 20% of the population is unemployed graduates.
= `7,500 Choice (C)
40 Also number of employed graduates is 30% of 40% of 100
= 12%
12. Given 5% of total population = 6000 20 + 12
∴ Required percentage = × 100 = 32%
∴ 45% of total population 100
(number of persons below 50 years of age) is equal to Choice (A)
45 × 6000
= 54,000 Ans: (54,000)
5 25. Let the total number of votes polled be x.
As 49% of the votes were polled to P, remaining 51% of the
100 × 33 1
13. Percent = 3 = 25% Choice (C) votes were polled to Q.
133 1 Given, 2% of x = 40,000 ⇒ x = 20,00,000.
3 Choice (A)

14. Revenue is given by Price × Sales 26. Let, David’s income be `100
Let the initial price be `100 and sales units be 100 60
respectively. Initial expenditure = × 100 = `60
100
∴Initially revenue = 100 × 100 = `10,000
∴ New revenue = 125 × 76 = `9,500 Initial savings = 100 – 60 = `40
Income after increase of 30%
500
∴ Percentage decrease in revenue = × 100 = 5% 130
10000 = × 100 = `130
100
Choice (C)
120
New savings = × 40 = 48
15. Percentage decrease in the breadth of the rectangle 100
 100 × 60  New expenditure = 130 – 48 = 82
=   % = 37.5% Choice (D) Percentage increase in expenditure
 160 
82 − 60 2
= × 100 = 36 % Choice (B)
16. The man spends (20% + 23% + 16% + 11%) of his income. 60 3
∴ He spends 70% of his income.
∴ The man saves 30% of his income. 27. 37
1
%=
3
; 12
1
%=
1
;
Given 30% of income = 2100 2 8 2 8
⇒ Income = `7000 Ans: (7000) 2 1 1 1
16 % = ; 33 %=
17. Let the income of A be `800 and that of B be `500 3 6 3 3
A’s expenditure = 50% of 800 = `400 3x y
+ = 15
To have same expenditure, B should save `100 8 8
∴ B’s savings percentage = 100/500 × 100 = 20% ⇒ 3x + y = 120 → (1)
Choice (A)

Triumphant Institute of Management Education Pvt. Ltd. (T.I.M.E.) HO: 95B, 2nd Floor, Siddamsetty Complex, Secunderabad – 500 003.
Tel : 040–27898195 Fax : 040–27847334 email : info@time4education.com website : www.time4education.com SM1001920.Sol/7
y x 37. Let the total marks = x
− = 12 Therefore 0.4x – 10 = 0.2x + 20
3 6
30
2y − x ⇒ 0.2x = 30 ⇒ x = = 150 Ans: (150)
⇒ =12 0.2
6
⇒ 2y – x = 72 → (2) 38. Difference in the sales tax
Multiply equation (1) by 2 and subtract it from equation (2) = 12.5% of 40,000 − 8% of 40,000 = 4.5% of 40000
we get x = 24 = 1800. Choice (D)
3x + y = 120
⇒ 3(24) + y = 120
55000 − 44000
⇒ y = 48 39. Loss percentage = × 100 = 20%
x + y = 24 + 48 = 72. Ans: (72) 55000
Ans: (20)
28. Let the number of students be x.
Number of students passed = 38% of x 40. The man makes a profit of 3 articles on every 9 articles that he
Number of students failed sells
= (100 – 38)% of x = 62% of x Therefore profit percent = 3/9 × 100 = 331/3%
So (62% of x) – (38% of x) = 60 Choice (D)
24% of x = 60 ⇒ x = 250 Choice (D)
CP − 132 12
41. =
29. Let the population of the village two years ago be 100. CP 100
Then the population last year was 120 and present 88CP = 132 x 100
population will be 144. Given, 24 is 60000 CP = `150 Choice (A)
100 × 6000
100 is = 25,000. Ans: (25,000)
24
42. Loss percentage =
(20)2 = 4% Choice (D)
83.33 100
30. 83.33% = = 0.833
100
34000
= 0.8 + 0.033 43. Cost price of Arun's vehicle = = 4,000 `
0.85
8 1 24 + 1 25 5
= + = = In order to earn a profit of 10% he should sell it for
10 30 30 30 6 1.1 × 40000 = `44000 Choice (B)
Choice (D)
44. Let cost price for A = `100. Then cost price for B = 1.2 x 100
31. Let the cost price of each article be C and the selling price = `120 and cost price for C = 1.15 x 120 = `138. When C
be S. pays `138, cost price for B is `120
S 3 2C S When C pays `2760, the cost price for B is
C– = C =
3 5 5 3 120
x 2760 = `2400 Choice (B)
6 1 138
S= C = C + C i.e., 20% Choice (D)
5 5
45. 45S – 45C = 15S
32. Given, 20% of n − 12.5% of 400 = 30 30S = 45C
⇒ 20% of n = 30 + 1/8 × 400 ⇒ 20% of n = 80 45 C
S= C=C+ i.e., 50% gain Ans: (50)
⇒ n = 400 30 2
∴ 36% of n = 36 × 4 = 144 Ans: (144)
33. Let Robin's score = x 46. SP = `15.30
Ajay's score = 1.3x 100
Robin's score : Ajay's score = x : 1.3x = 10 : 13 CP = 15.30 × = `15
102
Choice (B)
SP @ profit of 4% = `15.60
34. Let us say C scored 100 marks Discount = 20%
Then B scored 200 marks and A scored 150 marks 15.60 × 100
Marked Price = = `19.50
Percentage of score that C got less than A 80
150 − 100 4.5
= × 100 = 33.33% Percent markup = × 100 = 30% Choice (A)
150 15
Choice (A)
47. Let the cost prices of articles A and B be a and b
35. Percentage change (decrease) in respectively
(15)2 Given, 1.3a + 0.8b = 1160 --------- (1)
area = = 2.25% Choice (A) 0.8a + 1.05b = 1160 ----------------- (2)
100
1.3 × (2) − 0.8 × (1) gives 0.725b = 580
580
36. Let there be 100 students in the class b= = 800 Ans: (800)
0.725
Number of boys = 70; Number of girls = 30
Number of boys who passed the examination = 0.4 x 70 = 28 48. Let the cost price of the product be `x.
Number of boys who failed in the examination = 70 – 28 = 42 Therefore 1.27x – 1.5x = 24
Number of girls who passed the examination = 0.6 x 30 = 18 24
Number of girls who failed in the examination = 30 – 18 = 12 ⇒ 0.12x = 24 ⇒ x = = 200 Choice (B)
When the number of boys failed is 42, the number of girls failed
0.12
is 12
49. Let the cost price of the product be `100
When the number of boys failed is 84, the number of girls Therefore the marked price = 1.6 x 100 = `160
12 Selling price = 0.7 x 160 = `112
failed will be x 84 = 24 Ans: (24) Therefore the trader earns `12 on every `100
42
Therefore the profit percent = 12% Choice (D)
Triumphant Institute of Management Education Pvt. Ltd. (T.I.M.E.) HO: 95B, 2nd Floor, Siddamsetty Complex, Secunderabad – 500 003.
Tel : 040–27898195 Fax : 040–27847334 email : info@time4education.com website : www.time4education.com SM1001920.Sol/8
50. Let cost price of the product `x 100 − 72
Therefore marked price = `1.4x Therefore total discount = x 100 = 28%
100
Selling price = `0.8 x 1.4 x =`1.12x
"OR"
Therefore profit = 1.12x – x = 0.12x
20 × 10
24 Total discount = 20 + 10 – = 28% Choice (A)
Now, 0.12x = 24 ⇒ x = = 200 Ans: (200) 100
0.12

51. If the successive discounts are x% and y%, then net 62. Let M.P = `100
S.P for trader A= 0.7 x 100 = `70
 xy  S.P for trader B = 0.8 x0.9 x 100 = `72
discount =  x + y − %
 100  S.P for trader C = 0.85 x 0.9 x 0.95 x 100 = `72.675
15 × 10 Therefore a customer should buy from trader A
15 + 10 – = 25 – 1.5 = 23.5% Choice (C) Choice (A)
100
63. Let the cost price of the article be `x. Then the marked
52. Total discount given by trader B
price of the article will be ` (x + 200)
25 × 5 1.25x = 5/6 (x + 200) ⇒ 2.5x = 1000 ⇒ x= 400
= 25 + 5 – = 30 – 1.25 = 28.75%, which is less
100 ∴ It’s marked price is `600.
than that given by trader A. Choice (A) If there is no discount, S.P. = `600.
200
53. Let C.P. = x ∴ Profit percentage = × 100= 50%
0.8x = 12,000 400
12,000 Alternate method:
x= = 15,000
0.8 Let the CP be `100.
Percentage gain by selling it for `20,000 SP = `125
20,000 − 15,000 100
=
15,000
x 100 = 33.33% Choice (B) MP =
(
100 − 16 2 / 3)× 125 = `150.

If there is no discount, SP = MP = `150


54. As the cost price of the two articles are equal and their
Profit = 50%. Choice (A)
profit and loss percentages are equal, on the whole it is
neither profit nor loss. Choice (B)
64. Given, profit = C.P. of 5 mangoes.
i.e. S.P. of 25 mangoes − C.P. of 25 mangoes
C−S 16 − 12
55. Profit % = x 100 = x 100 = C.P. of 5 mangoes.
S 12 ⇒ S.P. of 25 mangoes = C.P. of 30 mangoes.
1 C−S
= x 100 = 33.33%. Choice (D) Using profit% = × 100, we have profit %
3 S
56. Let C.P for Suraj = `100 30 − 25
= × 100 i.e. 20%. Choice (B)
then C.P for Chand = 1.2 x 100 = `120 25
and C.P for Sitare = 1.1 x 120 = `132
When Sitare pays `132, C.P for Suraj = `100 65. C.P. of score of articles = `4x
100 4x x
When Sitare pays `2640, C.P for Suraj = x 2640 C.P. per article = =`
132 20 5
= `2000 Choice (B) S.P. of gross of articles = `36x
36 x x
57. For every 900gms that the trader sells, he earns a profit of S.P. per article = = Rs
144 4
100 gms
x x
100 1 −
Therefore percentage of profit = x 100 = 11 %
900 9 Profit% = 4 5 × 100 = 25% Choice (C)
x
Choice (D) 5
58. C.P of 24 items = `30 (L.C.M of 8 and 6 = 24) 66. Let the CP of Meetal = `100
S.P of 24 items = `36 SP of Meetal = CP of Sheetal = `128
36 − 30 SP of Sheetal = CP of Neenta = `108.80
Therefore percentage of profit = x 100 = 20% Difference of CPs of Meetal and Neental = `8.80
30
Choice (B) Difference Selling price
8.80 ----------- 108.80
59. Let C.P of the purse = x 1320 ----------- ?
0.92x + 75 = 1.07x 1320 × 108.8
?= = `16,320 Ans: (16,320)
75 8.8
⇒ 0.15x = 75 ⇒ x = = 500 Ans: (500)
0.15
67. Let CP = `100
60. Let C.P = `100 MP = 20% over CP = `120
M.P = 1.5 x 100 = `150 SP = 8% discount on MP = `110.40
S.P. = 0.8 x 150 = `120 Profit percent = 10.4% Choice (A)
Profit = 120 – 100 = `20 68. Let the CP of the article = `100
When profit = `20, C.P = `100 Its SP = `80
100 Loss = `20
When profit = `12, C.P = x 12 = `60 Ans: (60)
20 Increased CP = `112
Increased SP = `96.80
61. Let, marked price = `100 Changed loss = 15.20
After a discount of 20% S.P = 0.8 x 100 = `80 4.8
After another discount of 10% S.P =0.9 x 80 = `72 % decrease in loss = × 100 = 24% Choice (B)
20

Triumphant Institute of Management Education Pvt. Ltd. (T.I.M.E.) HO: 95B, 2nd Floor, Siddamsetty Complex, Secunderabad – 500 003.
Tel : 040–27898195 Fax : 040–27847334 email : info@time4education.com website : www.time4education.com SM1001920.Sol/9
69. Ratio of the investments of A and B is 5. Let the sum be `x then,
(1,20,000 x 6 + 30,000 x 6) : (1,80,000 x 6 + 1,50,000 x×6×5
x 6) = 1,62,000 : 1,98,000 = 9 : 11 = 306 ⇒ x = `1,020 Ans: (1020)
100
Therefore the profit sharing ratio of A and B = 9:11
Choice (C)
6. Let the sum be `100.
70. Let Q's investment = `2x Amount after three years at S.I. = `300
Therefore P's investment = `x Interest for three years = `200
and Q's investment = `x. Amount which is nine times of the sum = `900
P's share in the profit =
x
=
1 Interest = `800
2x + x + x 4
Therefore total profit = 4 x 500 = `2,000 Ans: (2000) Interest (`) Number of Years
200 --------------- 3
71. Let Z invest his money for x months 800 ----------------- ?
Therefore the ratio of the investments of X, Y and Z is 800 × 3
(50,000) (12) : (60,000) (12) : (50,000) (x) = 60 : 72 : 5x ?= = 12 years
200
5x 15
Now = ∴ For the sum to become nine times of itself, the time
60 + 72 + 5 x 20 + 24 + 15 period is twelve years Choice (C)
⇒ 59 x = 3(132 + 5x) ⇒ 59 x = 396 + 15 x
⇒ 44 x = 396 ⇒ x = 396/44 = 9 7. Let the sum be `x and the rate of interest be r%
Therefore Z joined after (12 – 9) i.e., 3 months x ×5×r 5×r
x+ = 12480 i.e = 12480 − x ------------ (1)
Choice (C) 100 100
72. Ratio of investments of Sun, Moon and Star = 5 : 6 : 7 x ×8×r 8×r
x+ = 13728 i.e = 13728 − x ------------ (2)
6 100 100
Moon's share in profit = x 5,40,000
5+6+7 Dividing (1) and (2),
= 1,80,000 Ans: (1,80,000) 12480 − x 5
=
73. Ratio of Shiva's investment to Pradeep's investment 13728 − x 8
= (12 x 1,00,000) : (9 x 1,50,000) ⇒ x = `10,400 Ans: (10,400)
Therefore Shiva's share in profit = 8/(8 + 9) x 51,000
= `24,000 Choice (D) 8. Let the sum be `100
Amount after five years = `1,900
74. Let Kumar leave after x months Interest for five years = `1,800
Ratio of Ravi's investment to that of Kumar Amount of four times of the sum = `400
= (12 x 40,000) : (50,000 x)= 48 : 5x Interest = `300
48 8 Interest (`) Number of years
But, = ⇒x=6 Ans: (6)
5x 5 1800 --------------- 5
300 ----------------- ?
75. Let the periods of investment of A, B and C be x, y and z
300 × 5 5
months respectively ?= = years
5x : 3y : 2z = 5 : 6 : 5 1800 6
5 6 5 i.e. 10 months. Choice (A)
x:y:z= : :
5 3 2
88000 × 100 × 45
= 1 : 2 : 5/2 = 2 : 4 : 5 9. Interest = = `30,000 Ans: (30,000)
x:y:z=2:4:5 Choice (A) 11× 100 × 12

Exercise – 4
10. Let the sum be `x
(Simple Interest and Compound Interest)
x× 4× 4 x×8×3 x×5×2
+ + = 8500
Solutions for questions 1 to 25: 100 100 100
1. Let the principal be x and interest per annum be y. 50
⇒x× = 8500 ⇒ x = `17,000 Choice (D)
x + 2y = 10328 → (1) 100
x + 3y = 11248 → (2)
Solving (1) and (2) we get x = 7600 11. Let the sum be `100
⇒ P = `7600 Ans: (7600) Amount after eight years = `400
Interest = `300
PTR 17.5 times of sum = `1,750
2. S.I =
100 Interest = `1,650
P × 3 × 12 Interest (`)
⇒ 1152 = ⇒ P = `3200 Choice (C) Number of years
100
300 ----------- 8
6500 × 4 × 13 1650 --------- ?
3. S.I = = `3380 Choice (D) 1650 × 8
100 ?= = 44 Ans: (44)
300
4. Let the sum be `P
P × 2 × 10 P × 3 × 12 P × 2 × 15
+ + = 5332 12. S.I =
PTR
100 100 100 100
20P 36P 30P 3860 × 4 × 10.25
+ + = 5332 = =`1582.60 Choice (C)
100 100 100
100
86P
= 5332
100 13. Let the sum be `x
100 x (24) (2) 48x
P = 5332 × =` 6200 Choice (A) S.I. for two years = =
86 100 100
Triumphant Institute of Management Education Pvt. Ltd. (T.I.M.E.) HO: 95B, 2nd Floor, Siddamsetty Complex, Secunderabad – 500 003.
Tel : 040–27898195 Fax : 040–27847334 email : info@time4education.com website : www.time4education.com SM1001920.Sol/10
2 r 1560
 120  44 x = × = 1653.60 − 1560
C.I. for two years = x   − x = 100 2
 100  100
78r
4x ⇒ = 93.6
Difference = = 180 10
100
⇒ r = 12
⇒ x = `4,500 Choice (B)
Also,
r% of sum = S.I. for one year
14. Let the sum be `x.
Then, the amount after three years = `2.197 × x 12
⇒ × x = 780
3 100
 100 + r 
⇒ 2.197 × x = x   ⇒ x = `6,500 Choice (D)

 100 
3 21. Given, P = `8000, R = 5% and n = 3
 100 + r 
⇒ (1.3)3 =  
  R 
n

 100  A = P1 + 
 100 
100 + r
⇒ 1.3 = 3
100  5 
So, A = 80001 + 
⇒ 130 = 100 + r ⇒ r = 30% Choice (A)  100 
3
15. Let the value of the machine two years ago be `x  21 
A = 8000 
2  20 
 90 
x   = 24300 8000 × 21 × 21 × 21
 100  = = Rs.9261
⇒ x = `30,000 Choice (D) 20 × 20 × 20
∴Compound interest = A – P = 9261 – 8000 = `1261
16. Let the sum be `100 Choice (B)
Amount after three years at C.I. = `300
Amount after six years at C.I. = `900 22. Let the principal be `P
Amount after nine years at C.I. = `2,700  18 
2
 18 
Amount after twelve years at C.I. = `8,100 P1 +  − P1 +  = 2655
 100   100 
2
 118   118 
 100  P −  100  P = 2655
100 300 900 2700 8100    
118P  118 
 − 1 = 2655
100  100 
3 years 3 years 3 years 3 years 118P 18
× = 2655
∴ time period is twelve years Ans: (12) 100 100
2665 × 100 × 100
17. Let the sum be x. P=
Amount after three years at C.I. 118 × 18
108 110 120 P = `12500. Choice (C)
=x× × × = 14,256
100 100 100
23. `(3398.4 – 2880)
⇒ x = `10,000 Choice (A) = `518.4 is the interest on `2880 for 1 year.
100 × 518.4
18. Let the sum be `100 ∴r = = 18% p.a. Choice (D)
Amount after eight years = `400 2880 × 1
Amount after sixteen years = `1,600
i.e., it takes 8 + 8 = 16 years for the sum to become sixteen 24. Let the principal be `P
times of itself. 4 4
 R   R 
2P = P1 +  ⇒  1 +  =2
100 400 1600  100   100 
we require P to become 8P
n
 R 
8 8 so 8P = P1 + 
 100 
8 + 8 = 16 years Choice (D) n
 R 
⇒ 8 = 1 + 
19. Let the sum be `100  100 
As compounding is done every four months, rate of interest n
 R 
for four months is 30/3 = 10% ⇒ 23 = 1 + 
3  100 
 110 
Amount after one year = 100   = `133.1 
3
R  
4 n
 100   R 
⇒ 1 +   = 1 + 
Interest = `33.1  100    100 
 
33.1
∴ rate of simple interest = × 100 = 33.1% ⇒ n = 12. Ans: (12)
100
Choice (D) 25. Let each be `x.
20. Let the sum be x and the rate of interest be r% 1 year interest on `x at 14% p.a. under CI or SI =
(x )(14)
Difference of S.I. and C.I 100
= r% of S.I. for one year = 0.14x

Triumphant Institute of Management Education Pvt. Ltd. (T.I.M.E.) HO: 95B, 2nd Floor, Siddamsetty Complex, Secunderabad – 500 003.
Tel : 040–27898195 Fax : 040–27847334 email : info@time4education.com website : www.time4education.com SM1001920.Sol/11
1 year interest on `x at 16% p.a. under CI or SI =
(x )(16) If B covers 900 m in the same time C can cover 800 m.
100 If B covers 2700 m in the same time C can cover
2700
= 0.16x × 800 = 2400
0.14x + 0.16x = 2250 900
x = 7500. Ans: (7500) So in 2700 m race B beats C by 2700 − 2400 = 300 m.
Choice (B)
Exercise – 5
(Time and Distance) 10. Let the distance AB be x km and BC be 2x km.
x 2x
Solutions for questions 1 to 50: Total time taken to cover 3 x km = +
30 40
5 4x + 6x x
1. 45 kmph = 45 × m/s = 12⋅5 m/s. Choice (B) = =
18 120 12
Total dis tan ce
Average speed =
15 15 18 Total time taken
2. m/s = × kmph = 27 kmph. Choice (D)
2 2 5 3x 3x
= = × 12 = 36 kmph. Ans: (36)
x x
3. Speed = Distance / Time
12
260 260
Speed = = × 2 = 40 kmph . Ans: (40)
6 .5 13 11. Let the time taken be 4x hours
Total distance = 4x × 30 = 120x km
4. Let the distance be 2x km
3
x x 60x km is covered in × 4 x = 3 x hours.
+ = 4.5 4
40 50
The remaining (120x − 60x) km he has to cover in 4x − 3x
5x + 4x 9
= ⇒ x = 100 = x hours.
200 2 dis tan ce 60 x
The distance is 2x = 200 km. Choice (A) Required speed = = = 60 kmph .
time x
5. Murali leaves Chennai 2 hours after Kalyan, so in two hours Choice (C)
Kalyan covers 2 × 40 = 80 km.
12. If a person covers two equal distances at speeds of x kmph
Kalyan is 80 km away from Murali when Murali starts.
2xy
Dis tan ce and y kmph respectively then average speed = kmph
Time = x+y
Ralative speed
2 × 20 × 60
80 ∴ Average speed = = 30 kmph. Choice (D)
Time = = 4 hours 80
60 − 40
Murali travels for 4 hours and meets Kalyan 4 hours after
10:00 a.m. that is they will meet at 2:00 p.m. 13. Distance = time × relative speed
Choice (C) = 4(35 + 25) = 240 km. Ans: (240)

Dis tan ce 8 14. Let the distance be x km


6. Speed = = m/s x x
Time 3 − =3
8 8 18 20 30
m/s = × kmph 3 x − 2x
3 3 5 = 3 ⇒ x = 180 km. Choice (A)
Distance covered in 3 hours 45 min. 60
8 18 15
= × × = 36 km . Ans: (36) 15. Let the distance between the school and the house be
3 5 4
x km.
7. Let the original speed be x kmph x x 30
− =
800 800 4 6 60
− =2
x − 20 x 3 x − 2x 1
= ⇒ x = 6 km. Ans: (6)
800 x − 800 x + 16000 12 2
=2
x ( x − 20 )
16000 = 2x (x − 20) 16. Speeds are in the ratio of 6 : 5. Hence the time taken to
16000 = 2x2 − 40x cover a certain distance will be in the ratio of 5 : 6.
Time taken by Kumar to cover the distance
⇒ x2 − 20x − 8000 = 0
⇒ x(x − 100) + 80 (x + 80) = 0 ⇒ x = 100 12
= × 6 = 72 minutes. Choice (A)
So reduced speed = 100 − 20 = 80 kmph. Choice (D) 6−5

8. Let the distance be d km 17. Let the speed in still water be x kmph and speed of stream
Let the usual speed be 4x kmph be y kmph
3 3
of the usual speed = × 4x = 3x Speed down stream =
42
= 7 kmph
4 4 6
d d 20 d 1
− = ⇒ = ⇒ d = 4x Speed upstream =
24
= 4 kmph
3 x 4 x 60 12 x 3 6
d 4x So x + y = 7 → (1)
Time taken to cover d km at 4x kmph = = = 1 hour.
4x 4x x−y=4 → (2)
Ans: (60) Subtracting the equation (2) from equation (1), we get y = 1.5
So speed of the current = 1.5 kmph
9. A B C Choice (D)
1000 900 800
Triumphant Institute of Management Education Pvt. Ltd. (T.I.M.E.) HO: 95B, 2nd Floor, Siddamsetty Complex, Secunderabad – 500 003.
Tel : 040–27898195 Fax : 040–27847334 email : info@time4education.com website : www.time4education.com SM1001920.Sol/12
18. Distance covered = 500 m 26. Speed upstream = 8 − 2 = 6 kmph
5 60
Speed = 36 kmph = 36 × = 10 m/s Time taken to row 60 km, upstream = = 10 hours.
18 6
Time taken to cross an electric pole Ans: (10)
Length of the train 500
= = = 50 sec . Choice (B) 27. A takes 8 seconds less than B to complete the race.
Speed 10
B takes 8 seconds to cover 120 m.
120
19. A B B C Speed of B = = 15 m/s
1000 900 1000 850 8
900
Time taken by B to complete the race = = 60 sec
A B 1000 1000 1000 15
× = × =
B C 900 850 765 So A takes 60 − 8 = 52 seconds to complete the race.
A 1000 Choice (B)
=
C 765 28. Time taken to meet for first time any where on the track
If A covers 1000 m, in the same time C can cover 765 m. Length of track
So in a 1000 m race, A beats C by 1000 − 765 = 235 m. =
Re lative speed
Ans: (235)
12 12 2
20. Tiru = = = hours = 40 min. Choice (A)
Hyd 10 + 18 18 3
600 km
29. Time taken to meet for the first time at the starting point
60 × 2 = 120 km 600 – 120 = 480 km  length of track length of track 
= LCM of  , 

 speed of A speed of B 
8:00 a.m. 10:00 a.m.
10:00 a.m.  600 600 
= LCM  , 
60 kmph 60 kmph 20 kmph  20 30 
= LCM of 30, 20 = 60 sec. Ans: (60)
X X Y
30. Let the length of train be x m and speed of the train be
X travels 2 hours before Y. Distance covered in 2 hours s m/s
= 2 × 60 = 120. At 10:00 a.m. The distance between X and Y x
is 480 km. = 12 ⇒ x = 12s
s
Time taken for X and Y to meet
x + 200
Dis tan ce 480 = 20
= = 6 hours s
Re lative Speed 60 + 20
⇒ x = 20s − 200
So X and Y meet 6 hours after 10:00 am.
So 12s = 20s − 200
i.e. at 4:00 p.m. Choice (B)
⇒ 8s = 200
21. Let the distance between A and B be x km ⇒ s = 25 m/sec
x x Length of train = 12s
+ =6 = 12 × 25 = 300 m. Choice (C)
3 +1 3 −1
x x x + 2x 31. Let the length and the speed of the train be L and S
⇒ + =6 ⇒ =6
4 2 4 respectively.
⇒ 3x = 24 ⇒ x = 8 km. Ans: (8) L = 40 S → (1)
Time taken by the train to cross a platform of length ‘L’
22. Distance covered in crossing the platform = (800 + 300) m L + L 2L
= = = 80 (∵ L = 40 S) Choice (B)
= 1100 m S S
Time = 110 sec
1100 32. Let the speeds of the trains P and Q be S and 3S
Speed = = 10 m/s
110 respectively. Let the length of the train Q be L.
18 L
10 m / s = 10 × = 36 kmph . Choice (C) = 20 ⇒ L = 40S.
5 3S − S
Time taken by the train Q to cross the person if the trains
23. Let the speed in still water be x kmph, L
are moving in opposite direction = = 10 seconds.
x − 2.5 = 15 3S + S
⇒ x = 17⋅5 Choice (C)
Speed down stream = 17⋅5 + 2⋅5 = 20 kmph.
Choice (B) 33. Let the time taken by Ajay to reach his office travelling at
his usual speed be t. If Ajay doubles his speed, he takes
320 + 355 half the time he usually takes to travel to his office.
24. Time taken =
5 1 1
(30 + 24 ) t– t = ⇒ t = 1 hour.
18 2 2
675 If Ajay travels to office at thrice his usual speed, he would
= sec = 45 sec . Ans: (45)
15 1
take one-third of the usual time i.e., hours.
3
180 180 Ans: (20)
25. Time taken = sec = = 18 sec .
5 10
( 40 − 4) 34. Let the total distance travelled by Ajay be 16 km. Sum of
18
time durations in which the distances travelled at 2 km/hr,
Choice (D)
4 km/hr and 1 km/hr were covered

Triumphant Institute of Management Education Pvt. Ltd. (T.I.M.E.) HO: 95B, 2nd Floor, Siddamsetty Complex, Secunderabad – 500 003.
Tel : 040–27898195 Fax : 040–27847334 email : info@time4education.com website : www.time4education.com SM1001920.Sol/13
16 / 2 16 / 4 16 / 4 1
= + + 43. Speed of the boat in still water = (Speed of the boat
2 4 1 2
= 4 + 1 + 4 = 9 hours. downstream + Speed of the boat upstream)
Total distance 16 Hence speed of the boat upstream = 2 (Speed of the boat
Average speed = = kmph. in still water) – Speed of the boat upstream
Total time 9
= 2 (10) – 12 = 8 kmph. Choice (A)
Choice (A)
35. If A takes t seconds to run the race, B would take t + 30 44. Let the speed of the boy and the length of the train be S
seconds to run the race. C would take t + 60 seconds to run and L respectively.
the race. L = 30 (8 – S) = 25(9 – S)
Ratio of speeds of A and C is the reciprocal of the ratio of 240 – 30S = 225 – 25S
t + 60 S = 3 m/sec Ans: (3)
the times taken by them to run the race i.e., . When
t
A finishes the race, C would have run 1000 – 400 = 600 m. 45. Let the length of the train be L m. Let the speed of the train
t + 60 1000 be S m.
= L + 200 = 60 S
t 600 L + 200 = 50 (S – 5)
t = 90 seconds. Choice (B) Hence 40 S = 50 (S – 5)
S = 25 m/sec. Choice (C)
36. Let the speeds of the boat in still water and speed of the
stream be x and y respectively. 46. When A finishes the race, B would have run the length of
Let the distance AB be d km. the race – (the start distance + the distance by which A
d 2d x beats him) = 600 – (200 + 120) = 280 m.
= ⇒ = 3. Choice (D)
x−y x+y y Ratio of speeds of A and B = 600 : 280 = 15 : 7.
Choice (B)
37. Let the lengths of the faster and slower trains be L1 and L2 47. Let the time taken by B to complete the race be t. Time
respectively. Let the speeds of the faster and slower trains taken by A to complete the race is (t – 15) seconds. As A is
be S1 and S2 respectively. twice as fast as B, he takes half the time taken by B to
L1 + L 2 cover the length of the race.
= 40 → (1)
S1 − S 2 t − 15 1
= ⇒ t = 30 seconds. Choice (C)
L1 + L 2 t 2
= 80 → (2)
2S 2 − S1
48. Let the length of the train be L m.
2S 2 − S1 1
Dividing (1) by (2), = Let the speed of the train be S m/sec
S1 − S 2 2 L = 30S
4S2 – 2S1 = S1 – S2 L + 200 = 40S
S1 5 ⇒ 30S + 200 = 40S
= . Choice (A) S = 20 m/sec
S2 3
Alternate method:
The train takes 10 seconds to cover 200 m of its own
38. Let the speed of the boat upstream be x kmph. length. Hence its speed is 20 m/sec. Choice (A)
20 20
+ =6
2x x 49. Let the length of the train be L
x = 5 kmph. Ans: (5) L + 300 = 40 × 11 = 440 → (1)
L = 140 m. Ans: (140)
39. Let the distance covered by the three runners be d. Let the
speeds of the runners A, B and C be x, 2x and 3x 50. If the train is moving at its usual speed, it would take (40)
respectively. (2) = 80 sec to cross the bridge.
d d 800 + 200 100
− =2 Speed = = = 12.5 m/sec. Choice (B)
x 3x 80 8
d
= 3 hours. Exercise – 6
x (Time and Work)
Time taken by B to cover the same distance
d 3 Solutions for questions 1 to 50:
= = hours. Choice (B)
2x 2
1. One day’s work of Eswar and Ramesh
40. Time taken by the 220 m long train to overtake the boy 1 1 3+2 1
220 = + = = th of total work.
= = 8 seconds. Choice (B) 20 30 60 12
5 They together can complete the work in 12 days.
(108 − 9) ×
18 Ans: (12)
41. Let the speed of the stream be S kmph. 2. One day’s work of A, B and C
Speed of the boat in still water = 3S. 1 1 1 3 + 2 +1 1
40 40 = + + = = th of total work.
= +4 10 15 30 30 5
3S − S 3S + S They together can do the work together in 5 days.
S = 2.5 kmph. Ans: (25) Choice (D)
42. Time (in sec) taken by the three runners to meet for the first
3. We have M1 D1 H1 = M2 D2 H2
time at the starting point
⇒ 21× 56 × 8 = M2 × 32 × 6
 120 120 120 
= LCM  , ,  21× 56 × 8
 5 6 4  M2 = = 49
32 × 6
= LCM (24, 20, 30) = 120 i.e. 2 minutes. Choice (C) ∴ 49 men are required. Choice (A)

Triumphant Institute of Management Education Pvt. Ltd. (T.I.M.E.) HO: 95B, 2 Floor, Siddamsetty Complex, Secunderabad – 500 003.
nd

Tel : 040–27898195 Fax : 040–27847334 email : info@time4education.com website : www.time4education.com SM1001920.Sol/14


4. Let us say the total work was completed in x days. 14. Let us say the second pipe was closed after x minutes
x x −8 18 x
= + =1 = + =1
35 28 33 22
4 x + 5 x − 40 x 18
⇒ =1 ⇒ = 1−
140 22 33
⇒ 9x = 180 ⇒ x = 20 x 15
The total work is completed in 20 days. Ans: (20) ⇒ = ⇒ x = 10 Choice (B)
22 33
5. The part of the tank filled by both the pipes in one hour
15. The part of the tank filled in 1 minute when both the pipes
1 1 3 −1 1 were simultaneously opened in.
= – = = th. Time taken to fill the tank by
6 18 18 9 1 1 3−2 1
both the pipes is 9 hours. Choice (A) = − = = .
44 66 132 132
The tank will become full in 132 minutes. Ans: (132)
6. We have M1 D1 = M2 D2
(k – 1) (k – 8) = (k + 4) (k – 11)
16. Let us say A can do the work in x days. B can do the work
k2 – 8k – k + 8 = k2 – 11k + 4k – 44
⇒ k2 – 9k + 8 = k2 – 7k – 44 1 1
in 3x days. One day’s work of A and B = +
⇒ 2k = 52 x 3x
⇒ k = 26 Ans: (26) 1 1 1
⇒ + =
x 3x 6
1 1 5−4 1 3 +1
7. Vikram’s one day’s work = − = = th of ⇒ =
1
8 10 40 40 3x 6
the total work.
⇒x=8
Vikram can do the work in 40 days. Choice (A)
A can do the work in 8 days. Choice (C)
M1 D1 H1 M2 D 2 H2
8. We have = 17. Let the work be completed in x days
W1 W2 x x−3 x−5
= + + =1
26 × 21× 8 M × 20 ×14 21 42 14
= = 2
1 2 .5 2x + x − 3 + 3 x − 15
⇒ = 1
26 × 21× 8 × 2.5 42
M2 = = 39 Ans: (39)
20 ×14 ⇒ 6x = 60 ⇒ x =10
∴ A worked for 10 days. Ans: (10)
9. The part of the tank filled by the three pipes in one minute
1 1 1 13 + 10 − 20 3 18. We have M1 D1 H1 = M2 D2 H2
= + − = = th ⇒ 22 × 18 × 6 = 33 × 8 × H2
20 26 13 260 260
22 × 18 × 6
The time taken to fill the tank by the three pipes is H2 = =9 Choice (B)
260 2 33 × 8
= 86 minutes Choice (D)
3 3
19. 9 men’s work = 12 women’s work
⇒ 3 men’s work = 4 women’s work.
10. Let the work be completed in x days.
∴ 9 men can do the work in 24 days.
x−2 x
= + =1 ⇒ 6 men + 8 women or 12 men can do the work in
40 30 9 × 24
3x − 6 + 4x = = 18 days Choice (B)
⇒ =1 12
120
⇒ 7x = 126 4
⇒ x = 18 20. A can do the work in 12 x = 16 days. B can do the work
3
The total work will be completed in 18 days.
Choice (C) 8
in 20 × = 32 days . One day’s work of A and B
5
11. One day’s work of A, B and C
1 3+2 1 1 2 +1 3
=
1
+ = =
5
th of total work. = + = = . Time taken by A and B to
18 27 54 54 16 32 32 32
32 2
54 4 complete the work is = 10 days Choice (D)
The time taken to do the work is =10 days 3 3
5 5
Choice (B) 21. Let the number of men be x. We have M1 D1 H1 = M2 D2 H2
3x
1 X = x × 17 × 19 = × 34 × H2
12. Kavita’s one day’s work = Kavita’s x day’s work = 5
K K
x × 17 × 9 × 5 1
Choice (D) H2 = =7 Choice (B)
34 × 3 x 2
13. Let the total work be completed in x days
x−6 x−3 x 22. The time taken by the outlet pipe to empty the tank is less
= + + =1
15 45 30 than the time taken by the inlet pipe to fill the tank. ∴if the
6 x − 36 + 2x − 6 + 3 x inlet pipe and the outlet pipe are opened when the tank is
⇒ =1 partially or totally full, the tank will get emptied. As both
90 pipes are opened when the tank is half full the tank will
⇒ 11x = 132 ⇒ x = 12 become empty.
The total work will be completed in 12 days. So the part of tank emptied by both the pipes working
Ans: (12) together in one minute

Triumphant Institute of Management Education Pvt. Ltd. (T.I.M.E.) HO: 95B, 2nd Floor, Siddamsetty Complex, Secunderabad – 500 003.
Tel : 040–27898195 Fax : 040–27847334 email : info@time4education.com website : www.time4education.com SM1001920.Sol/15
1 1 1 44
= − = In 16 days, th of the total work is completed.
8 12 24 45
The time taken to empty half the tank 44 1
The remaining work after 16 days = 1 – = th of the
1 24 45 45
= × = 12 min . Ans: (12)
2 1 1
total work. On 17th day, A works and he can do th of
23. Whenever two or more people work together to complete a 45
piece of work and all work for the same time, the ratio of 1 1
their wages is given by the ratio of the daily work they do. If the work in 15 × = rd of that day.
45 3
all do not work for the same amount of time, the ratio of
their wages is given by the ratio of their total work they do. 1
The total time taken to complete the work is 16 days.
As A and B work together for the same time, the ratio of 3
their wages is the ratio of their daily work, they do. Choice (C)
The ratio of one day’s work of A and B
1 1 29. As 9 men and 21 boys have thrice the efficiency of 3 men
= : =3:2 and 7 boys, the time taken by 9 men and 21 boys to
12 18
complete the work will be one third of the time taken by
3
A’s share = × 300 = `180 Choice (B) 3 men and 7 boys i.e.
45
= 15 days Choice (A)
5 3
24. 10(6 men’s work + 9 women’s work)
= 15(5 men’s work + 4 women’s work) 3
30. The number of doors, Ramu can paint on each day =
60 men’s work + 90 women’s work 4
= 75 men’s work + 60 women’s work
⇒ 1 man’s work = 2 women’s work
2
The number of doors Samu can paint on each day =
⇒ 6 men’s work + 9 women’s work = 21 women’s work 3
∴ 4 men’s work + 7 women’s work = 15 women’s work In one day, the number of doors painted by Ramu and
6 men’s work + 9 women’s work = 21 women’s work which 3 2 9 + 8 17
can be done in 10 days.
Somu together = + = = .
4 3 12 12
21 × 10 The time taken by them to paint 34 balls
So 15 women can do the work in = 14 days
15 34
= × 12 = 24 days Ans: (24)
Ans: (14) 17
M1 D1 H1 M2 D2 H2 31. P worked for 7 days to complete the work
25. We have =
W1 W2 7
⇒ P’s 7 day’s work = th of the total work.
12 × 28 × 5 35 × D 2 × 6 12
= =
300 240 7 5
So, Q completed 1– = th of the work.
12 × 28 × 5 × 240 32 2 12 12
D2 = = = 6 days
300 × 35 × 6 5 5 5
∴ Q’s share = × 168 = ` 70 Choice (A)
Choice (D) 12
5 1 32. We have M1 D1 = M2 D2
26. A’s 5 day’s work = = of the total work.
15 3 = 21 × 25 = 15 × D2
4 1 2 21 × 25
The remaining work = = 1− = th of the total work. D2 = = 35 days Choice (D)
5 3 3 15
2
B completed of the work in 20 days, so he can do the 33. Let the required number of days be x.
3 Provisions available = (88 × 21× 3) kg
3
total work in × 20 = 30 days. . 12 × 2 × x = 88 × 21 × 3
2 ⇒ x = 231 Choice (D)
1 1 2 +1 1
One day’s work of A and B = + = = th of the 34. Let us say B worked for x days.
15 30 30 10 15 x 3 x
total work. + =1 ⇒ + =1
40 32 8 32
They can together do the work in 10 days.
Choice (C) x 5
⇒ =
32 8
1
27. B’s one day’s work = th of total work. ⇒ x = 20 Ans: (20)
26
As A is 30% more efficient than B, he can do 30% more 35. The number of mangoes they together eat in one day
work than B in the same time. 8 6
= + =6
1 130 1 2 3
A’s one day’s work = × = th of total work.
26 100 20 The time taken by them to eat 66 mangoes
A can do the work in 20 days. Ans: (20) 66
= = 11 days Choice (C)
6
1 1 6 + 5 11
28. 2 day’s work = + = = th of the total work 1 1 3 +1 2
15 18 90 90 36. One day’s work of A and B = + = = th of
11 44 10 30 30 15
(2 X 8) day’s work = ×8 = th of the total work the total work.
90 45
So, working together, A and B can do the work in
44 15 1
16 day’s work = th of the total work. = 7 days. Choice (C)
45 2 2

Triumphant Institute of Management Education Pvt. Ltd. (T.I.M.E.) HO: 95B, 2nd Floor, Siddamsetty Complex, Secunderabad – 500 003.
Tel : 040–27898195 Fax : 040–27847334 email : info@time4education.com website : www.time4education.com SM1001920.Sol/16
37. Let the total work be completed in x days. 45. (x + 1) (x − 2) : (x + 2) (x − 4) = 9 : 8
x 4 ⇒ 9 (x + 2) (x − 4) = 8 (x + 1) (x − 2)
+ =1
28 16 ⇒ 9x2 − 18x − 72 = 8x2 − 8x − 16
⇒ x2 − 10x − 56 = 0 ⇒ x2 − 14x + 4x − 56 = 0
x 3
= ⇒ x = 21. Ans: (21) = x (x − 14) + 4 (x − 14) = 0
28 4 = (x − 14) (x + 4) = 0
38. Let us say, A can do the work in x days. Then, B can do the x – 14 = 0 or x + 4 = 0
work in 2x days. So x = 14 or −4
1 1 3 As the number of men cannot be negative, x = 14.
So one day work of A and B = + = th of total Choice (B)
x 2x 2x
work. 46. Let the number of men, who leave be x. If x men did not
3 1 leave at the end of 6 days, the remaining provisions would
So = ⇒ 2x = 63. Choice (D)
2x 21 be sufficient for 64 men for 18 − 6 = 12 days.
So, 64 × 12 = (64 − x) 16
39. Let the work be completed in x days
64 × 12
x x−7 ⇒ 64 − x = = 48 ⇒ x = 16
+ =1 16
25 20
So, 16 men leave. Ans: (16)
4 x + 5( x − 7)
⇒ =1 1 1 1 6−2−3 1
100 47. C’s one day’s work = − − = = th of
6 18 12 36 36
⇒ 4x + 5x − 35 = 100
the total work.
⇒ x = 15
C can do the work in 36 days. Choice (C)
The total work is completed in 15 days. Choice (A)
1 1 3−2 1
40. We have M1D1H1 = M2D2H2 48. Kalyan’s one day’s work = − = = th of the
18 × 22 × 7 = M2 x 14 × 6 4 6 12 12
M2 = 33. total work.
The required number of men is 33. Ans: (33) Kalyan can do the work in 12 days, while Pavan can do it in
6 days. So Pavan is twice as efficient as Kalyan.
41. One day work of A, B and C ∴x = 2 Ans: (2)
1 1 1 4+3+2 1
= + + = = th of total work. 49. Assume that the leak can empty the tank in x hours.
18 24 36 72 8
1 1 1
8 − =
A, B and C together can do the work in = 8 days. 6 x 11
1
1 1 1 11 − 6 5
Choice (B) = − = =
x 6 11 66 66
1 66 1
42. Given, (A + B)’s one day’s work = ----- (1) x= = 13 hours. Choice (B)
36 5 5
1
(B + C)’s one day’s work = ----- (2) 50. Let the number of men be x
45 We have M1D1H1 = M2D2H2
1 4x
and (C + A)’s one day’s work = ----- (3) x × 18 × 8 = × D2 × 4
60 3
Adding (1), (2) and (3), we get, D2 = 27. Choice (B)
one day’s work of 2 A’s, 2 B’s and 2C’s
1 1 1 Exercise – 7
= + + (Averages, Mixtures and Alligations)
36 45 60
5+4+3 12 1 Solutions for questions 1 to 40:
= = = th of the total work.
180 180 15
312 + 162 + 132 + 142 + 122 870
1 1. Average = = = 174
One day’s work of A, B and C = th of the total work. 5 5
30 Choice (B)
1 1 3−2 1
A’s one day’s work = − = = th of the total work. 2. Let the five numbers be P, Q, R, S and T.
30 45 90 90 ⇒ P + Q + R + S + T = 655;
B’s one day’s work =
1

1
=
1
th of the total work. P+Q
30 60 60 = 85 and R = 125
2
1 1 1 ∴ P + Q = 170 and R = 125
C’s one day’s work = − = th of the total work.
30 36 180 ∴ P + Q + R = 295
A, B and C can do the work in 90 days, 60 days and S + T = 655 – (P + Q + R) = 360
180 days respectively. Choice (B) S+T
Average of the last two numbers = = 180
43. Let the total work be completed in x days. 2
x x−2 2x + 3 x − 6 Ans: (180)
+ =1⇒ =1
27 18 54 3. Calculated average height of 35 boys = 180 cm
⇒ 5x = 60 Wrong total height of 35 boys = 180 × 35 cm. This was as a
x = 12 result of an actual height of 106 cm being wrongly written
The total work was completed in 12 days Choice (C) as 166 cm. ∴ Correct total height of 35 boys
44. The part of the tank filled by both the pipes in one minute = 180 × 35 cm – 166 cm + 106 cm
1 1 5+3 8 2 Actual average height of 35 boys.
= + = = = 180 × 35cm − 166cm + 106cm 60
12 20 60 60 15 = = 180 cm – cm
15 1
35 35
The time taken to fill the tank = = 7 min . Ans: (7.5) = 180 cm – 1.71 cm ≈ 178.29 cm Choice (B)
2 2
Triumphant Institute of Management Education Pvt. Ltd. (T.I.M.E.) HO: 95B, 2 Floor, Siddamsetty Complex, Secunderabad – 500 003.
nd

Tel : 040–27898195 Fax : 040–27847334 email : info@time4education.com website : www.time4education.com SM1001920.Sol/17


4. Total age of seven persons = (28 × 7) years ∴ 5k – 148 = 3k + 148 ⇒ 2k = 2(148) ⇒ k = 148
Total ages of the first three persons and the last three ∴ The initial quantity of water in A = 8k = 8(148)
persons are (21 × 3) years and (34 × 3) years respectively. = 1184 liters. Choice (D)
Age of the person sitting in the middle of the row
= 28 × 7 – 21 × 3 – 34 × 3 = 196 – 63 – 102 = 31 years 14. Let the rainfall on the first day of the period be x cm.
Ans: (31) For the remaining period, it is (x + 5), (x + 10) …… and so
15 [2x + 14 × 5]
5. Let the marks obtained by the student in Physics, on. Average rainfall for the period = = 125
Chemistry, and Mathematics be P, C and M respectively. 2 15
P + C + M = 150 + P ⇒ 2x + 70 = 250 ⇒ x = 90 Ans: (90)
C + M = 150
Average mark obtained by the student in Chemistry and 15. Total marks of 20 students = 20 × 30 = 600
C+M Total marks of top nine students = 9 × 40 = 360
Mathematics = = 75 Choice (A) Total marks of top ten students = 10 × 21 = 210
2 Marks of student getting the 10th rank = 600 – 360 – 210
6. Present total age of the members= 26(5) = 130 years. = 30 Choice (B)
Present age of the youngest member = 10 years
Present total age of the remaining four members 16. Let the strengths of the students in the three colleges be x,
= 130 – 10 = 120 years 2x, 4x respectively, Weighted average age
Their average age at the time of the birth of the youngest x × 30 + 2x × 25 + 4 x × 5
= = 20 years. Choice (C)
120 − (4 × 10 ) 7x
member = = 30 – 10 = 20 years
4 20× 4080− 6280− 4730+ 2480+ 1370
Ans: (20) 17. New average = = 3722
20
7. Let the four integers be A, B, C, D where A > B > C > D. Choice (C)
A +B+C+D
= 69. ∴ A + B + C + D = 276 → (1) 18. Let the average sales per week of the eleven be x
4
Then the sales of eleventh salesman = x + 10
A = 93, D = 39 and B – C = 28
10 × 7 + x + 10
(1) ⇒ B + C = 276 – (A + D) = 276 – (132) = 144 Average of all eleven = =x⇒x=8
B + B – 28 =144 11
144 + 28 ∴ Average of eleventh salesman is 18 articles.
B= = 72 + 14 = 86 Choice (B) Ans: (18)
2
8. Average mark scored by all the three boys 19. Concentration of alcohol in the mixture = 30%
Concentration of acid = 70%
64
(800) + 36 (800) + 44 (800) Quantity of acid in the mixture = 70% of 80 = 56 litres
= 100 100 100 = 384 Let x litres of conc. acid be added to the mixture
3 85% (80 + x) = 56 + x ⇒ x = 80 Choice (B)
Ans: (384)
20. Total of day temperatures of the week = 45 × 7 = 315
1150 + 920 2070
9. Average price per book = = = `18 Total of day temperatures for the first six days
65 + 50 115 = 45 + 40 + 52 + 48 + 41 + 42 = 268 day temperature on
Ans: (18) Saturday = 315 – 268 = 47°C Choice (B)
10. Let the present ages of A, B and C be a, b and c 21. Total sales for the month = 48 × 30 = 1440
respectively. Total sales for the first 14 days = 14 × 48 = 1120
(a − 5) + (b − 5) Total sales for the remaining 16 days = 1440 – 1120 = 320
Given, = 40 ⇒ a + b = 90 → (1)
2 Average sales per day for the remaining 16 days
= 320/16 = 20 Choice (D)
b+c
= 48 ⇒ b + c = 96 → (2) 22. Total salary of the group of sixteen people
2
From (1) & (2), we cannot find b. Choice (D) = 16 × 500 = `8000
Total salary of new group = 8000 + 1000 + 1200 + 1400 − 800
11. Let the marks obtained by the student in Mathematics, = `10800
Physics and Chemistry be M, P and C respectively. Average salary of the group = 10800/18 = `600
Given, M + P = 60 and C − P = 20 Ans: (600)
M + C (M + P) + (C − P) 60 + 20
∴ = = = 40. 23. Let the number of test matches he played till now be x
2 2 2 84x – 8 × 18 = (x – 8)108 ⇒ x = 30 Choice (B)
Choice (A)
24. Total sales for the year = 200 × 12 = 2400 units
12. Let the initial number of members in the group be n.
Sales form January to June = 150 × 6 = 900
Initial total weight of all the members in the group = n(48)
Sales form July to October = 240 × 4 = 960
From the data,
Sales of November and December together
48n + 78 + 93 = 51(n + 2) ⇒ 51n – 48n = 69
= 2400 – 900 – 960 = 540 units
⇒ n = 23 Average sales of November and December
Therefore there were 23 members in the group initially. = 540/2 = 270 units Choice (D)
Ans: (23)
25. Let the fifteenth number be x,
13. B has 62.5% or (5/8) of the water in A. Therefore, let the
quantity of water in container A (initially) be 8k. 30 × 7 = 14 × 9 + x + 7 + 14 × 5 ⇒ x = 7 Ans: (7)
Quantity of water in B = 8k – 5k = 3k
26. Quantity of resultant mixture = 6 lts
∴ Quantity of water in container C = 8k – 3k = 5k Quantity of wine in the resultant solution
Container : A B C
= 0.6 × 2 + 0.45 × 4 = 3.0 litres
Quantity of water : 8k 3k 5k
Quantity of water in the resultant solution = 6 – 3 = 30 lts
It is given that if 148 liters was transferred from container C
Ratio of wine and water in the resultant solution
to container B, then both the containers would have equal
=3:3=1:1 Choice (B)
quantities of water.
Triumphant Institute of Management Education Pvt. Ltd. (T.I.M.E.) HO: 95B, 2nd Floor, Siddamsetty Complex, Secunderabad – 500 003.
Tel : 040–27898195 Fax : 040–27847334 email : info@time4education.com website : www.time4education.com SM1001920.Sol/18
27. Sale price of the mixture = `60 35. Selling price of the mixture = `16.50
100 Cost price of the mixture = `15
Cost price of the mixture = 60 × = `48 Let there be x kgs of sand in the mixture.
125
30 × 20 = (30 + x) 15 ⇒ x = 10 kgs Choice (B)
Let the cost of the sugar whose quantity is less be `x
40 x 36. By alligation method, ratio = 2 : 5 48
55
5 parts = 80 litres
2 parts = ?
48% 53
2 × 80
?= = 32 litres
8 5 2 5
x – 48
Choice (A)
x − 48 3
⇒ = ⇒ x = `60 Ans: (60) 37. Ratio = 6 : 8
8 2 73 87
=3:4
4 parts = 52 litres
28. Concentration of iron ore = 95% 81
3 parts = ?
Concentration of iron in another ore = 70%
3 × 52 87 – 81 81 – 73
Concentration of iron in resultant ore = 80% ?=
4 6 : 8
95 70 = 39 litres
Choice (C)
80
38. Total manufacturing cost = 40000 + 5 × 5000 = `65,000
10 15 Average manufacturing cost = 65000/5000 = `13
Choice (D)
=2:3 39. Let there be x kgs of sand in 100 kgs of jeera.
2 ×120
∴ Quantity of 95% iron ore = = 80 kg. Profit percentage =
x
x × 100 = 25
3 (100 − x )
Choice (C)
⇒ x = 20kgs
Percentage of sand in the mixture = 20/100 × 100 = 20%
29. Concentration of acid in the resultant solution is given
n
Ans: (20)
P−Q 
by   where P = Initial quantity, Q = Quantity replaced. 40. Percentage of milk in the mixture formed
 P 
2
 80 − 8 
80
(180 ) + 36 144 + 36
=  100 ×100% =
 = 0.81 Choice (B) =
180 + 36
.100%
 80  216
5
3 = .100% = 83.3 %. Choice (D)
 150 − 30  6
30. Quantity of wine in the resultant solution =   × 150
 150 
Exercise – 8
= 76.8 litres Ans: (76.8) (Numbers)
31. Part of milk in the first vessel = 2/10 Solutions for questions 1 to 100:
Part of milk in the second vessel = 4/10
Part of milk in the third vessel = 5/10 1. The least five digit number is 10000. When it is divided by
Part of milk in the resultant solution 372, the remainder is 328.
= 5 x 2/10 + 3 × 4/10 + 2 × 5/10 = 32/10 ∴ 10000 − 328 is the greatest four digit number divisible
Part of water in the resultant solution = 10 – 32/10 = 68/10 by 372.
Ratio of milk and water = 32 : 68 = 8 : 17 Choice (D) ∴ 10000 − 328 + 372 = 10044 is the least five digit number
divisible by 372. Ans: (10044)
32. Quantity of acid in resultant solution = 4/7 × 70 = 40 ltrs
Quantity of water = 30 litres 2. Given, A = 10/13; B = 8/11; C = 11/14 and D = 13/16. As in
Let x litres of water was added each fraction starting from 8/11 to 13/16 as numerator as
well as denominator are increasing by same quantity,
40 8
= ⇒ x = 5ltrs Choice (C) 8/11 < 10/13 < 11/14 < 13/16.
30 − x 5 ∴ Ascending order is BACD. Choice (B)

33. Part of milk in the mixture = 1/6 × 1/3 + 2/6 × 5/6 + 3/6 × 7/9 3. Sum of the digits of the number = 36 + p + q. This must be
divisible by 9.
1+ 5 + 7 13
= = ∴ p + q must be divisible by 9 …. (1)
18 18 The number has 8 digits excluding p and q.
13 5 ∴ p and q are single digit numbers.
Part of water in the mixture = 1 – =
18 18 ∴ 0 ≤ p, q ≤ 9
Ratio of milk and water = 13 : 5 Choice (D) p−q=7
∴ (p, q) = (7, 0) or (8, 1) or (9, 2)
34. Percentage of wheat in the initial Only (p, q) = (8, 1) satisfies (1)
70 0 ∴ (p, q) = (8, 1). Choice (B)
mixture = 70
Percentage of wheat in the
56 4. Quotients
resultant mixture
3 90 = 30
14 14
= × 100 = 56 56 3 30 = 10
15 3 10 = 3
Ratio = 56 : 14 = 4 : 1 3 3 = 1
The required percentage = 1/5 × 100 = 20% Sum 44
Choice (C)
∴The largest power of 3 in 90! = 44. Ans: (44)

Triumphant Institute of Management Education Pvt. Ltd. (T.I.M.E.) HO: 95B, 2nd Floor, Siddamsetty Complex, Secunderabad – 500 003.
Tel : 040–27898195 Fax : 040–27847334 email : info@time4education.com website : www.time4education.com SM1001920.Sol/19
5. X = 56y + 1 = 56y + 1y which is divisible by 57 only when 15. Let the smallest such number be x.
y is odd (∵aN + bN is divisible by a + b only when N is odd). x must have the forms 17q1 + 12 and 24q2 + 19 where q1
and q2 are whole numbers.
∴ It is divisible by any factor of 57 only when y is odd.
∴ x = 17 (q1 + 1) − 5 = 24 (q2 + 1) − 5
∴ It divisible by 19 when y is odd. Choice (A) x + 5 = 17 (q1 + 1) = 24 (q2 + 1)
∴ x + 5 must be divisible by both 17 and 24 and hence by
6. Let x = 0.6324 their L.C.M. As x is the smallest, x + 5 is also the smallest
10000x = 6324.6324 ∴ x + 5 = L.C.M. (17, 24)
∴ x = L.C.M. (17, 24) − 5 = (17) (24) − 5 = 408 − 5 = 403.
10000x − x = 6324
Ans: (403)
6324 2108
x= = . Choice (D)
9999 3333 16. N = L.C.M (22 × 33 × 5, 32 × 53 × 7, 52 × 73 × 11)
= 22 × 33 × 53 × 73 × 11
7. Let the greatest number be N. Number of distinct prime factors it has = 5. Choice (B)
8505 − 45 and 9925 − 55 must be divisible by N.
∴ 8460 and 9870 must be divisible by N 17. M = (72)N − 1 = 49N − 1N which is always divisible by 48
N is the greatest possible number satisfying this condition (∵ aN − bN is always divisible by a − b)
∴ N = g. c. d. (8460, 9870)
∴ It is always divisible by any factor of 48 and hence by 6.
8460 = 6.1410
Choice (C)
Dividing 9870 by 1410, we get a quotient of 7 and a
remainder of 0.
18. N = 31.32.33.34.35…….39
∴ 9870 = 7.1410
34 × 35 ends with units digit of 4 × 5 i.e. 0.
∴ N = g. c. d. (6.1410, 7.1410) = 1410 g.c.d (6, 7) = 1410
∴ N ends with 0. Ans: (0)
(1) = 1410. Ans: (1410)
19. Next time = 11a.m + LCM (Time intervals) = 11a.m + LCM
8. Let the greatest number be N.
(45 sec, 60 sec, 75 sec, 90 sec) = 11a.m + 900 sec
Let the remainder be r. 93 − r, 131 − r and 188 − r must be
divisible by N = 11a.m + 15min = 11 : 15a.m (∵LCM (45, 90) = 90)
∴ 131 − r − (93 − r) and 188 − r − (131 − r) must be divisible ∴ LCM (45, 60, 90) = L.C.M (60, 90) = 180
by N. ∴ LCM (45, 60, 75, 90) = L.C.M (75, 180)
∴ 38 and 57 must be divisible by N = LCM (15.5, 15.12) = 15 L.C.M (5, 12) = 15 (60) = 900.
N is the greatest possible number satisfying this condition Choice (B)
∴ N = g.c.d (38, 57) = 19. Choice (B)
20. All powers of 2 starting with 2 will have only 1 odd factor.
9. (HCF) (LCM) = Product All their remaining factors will be even.
∴ 18 (LCM) = 3240 ∴ All powers of 2 will be relatively prime to 234893.
∴ LCM = 180. Choice (A) There are infinitely many powers of 2 starting with 21
∴ There are infinitely many positive integers which are
relatively prime to 234893. Choice (D)
10. 3276 = 4.819 = 4.9.91 = 4.9.7.13 = 22 . 32 . 71 . 131
Number of factors of 3276 = (2 + 1) (2 + 1) (1 + 1) (1 + 1) = 36 21. Least number divisible by 55 and 12 = LCM (55, 12)
∴ Number of ways of writing it as a product of two factors = (55)(12) = 660
= 36/2 = 18. Ans: (18) Leas t number leaving a remainder of 2 which is greater
than 10 = 660 + 2 = 662. Choice (B)

11.
(0.55)2 + (0.04)2 + (0.028 )2 22. Let the g. c. d be x. Then Q and Q + P would be divisible by x.
(0.055 )2 + (0.004 )2 + (0.0028 )2 Then Q + P − Q must be divisible by x i.e. P must be
divisible by x. But P is prime
=
((0.055 )(10 ))2 + ((10.004 )(10 ))2 + ((10.0028 )(10 ))2 ∴ x = P or 1
(0.055 )2 + (0.004 )2 + (0.0028 )2 Q<P
∴ g. c. d cannot be P

=
(
10 2 (0.055 ) + (0.004 ) + (0.0028 )
2 2 2
) = 10 2
= 100
∴ g. c. d = 1. Choice (D)
(0.055 ) + (0.004 ) + (0.0028 )
2 2 2
23. x = a4 + 6a3 + 14 = a (a3 +6a2 + 14/a)
Ans: (100) For x to be divisible by a, a3 + 6a2 + 14/a must be an integer.
a3 + 6a2 is always an integer.
12. 234845 = 23 (11615) ∴ 14/a must be an integer. 14 must be divisible by a
 23 (11615 ) 
∴Required remainder = Re m  3
 3 11615
 = Re m  2 ( ) 

∴ a = 1 or 2 or 7 or 14
∴ a has four possible values. Choice (B)
 2 −1   23 − 1 
   
Let x = 23 and f (x) = x11615 24. 342 = 34 (10) + 2 Its units digit = That of 32 = 9 (∵ cycle of 4)
By remainder theorem, required remainder = f (1) = 111615 = 1
856 = 84 (14). Its units digit = That of 84 = 6 (∵ cycle of 4)
Choice (A)
640 ends with 6 (∵ Any positive power of 6 ends with 6)
13. Among the choices, 3
64 = 4 is the only rational number. ∴ Units digit = That of 9.6.6 = 4. Ans: (4)
Choice (D)
LCM (Numerators )
25. When a perfect cube is expressed as the product of its prime
14. LCM (Fractions) = . factors, the power of each prime factor must be divisible by 3.
HCF (Deno min ators) 19200 = (16) (12) (100) = (24) (22 . 3) (22 . 52) = 28. 31. 52
24 None of the powers are divisible by 3. To make the powers
Any fraction must be in the simplest form and so will be of 2, 3 and 5 divisible by 3, the number must be divided by
32 22, 3 and 52 respectively.
taken as 3/4. ∴ Least number =(22) (3) (52) = 300.
L.CM (3, 3, 9 ) 9 Choice (B)
∴ LCM = = . Choice (B)
HCF(4, 52, 50 ) 2
Triumphant Institute of Management Education Pvt. Ltd. (T.I.M.E.) HO: 95B, 2nd Floor, Siddamsetty Complex, Secunderabad – 500 003.
Tel : 040–27898195 Fax : 040–27847334 email : info@time4education.com website : www.time4education.com SM1001920.Sol/20
26. Least perfect square divisible by 18, 20, 27 and 30 = Least
37. 15129 = 123 No other number is a perfect square.
perfect square divisible by their LCM.
LCM (18, 20, 27, 30) = L.C.M (2 × 32, 22 × 5, 33, 2 × 3 × 5) Choice (A)
= 22 × 33 × 5. The least perfect square must have even 38. The given expression is in the form of
power for each prime factor in its prime factorised form. In (200 + 6)² + (200 – 6)²
the L.C.M, 2 is the only prime factor having an even power. = 2[(200)² + 6²] {(a + b)² + (a – b)² = 2 (a² + b²)}
∴ Least perfect square = (LCM) (3) (5) = (LCM)(15) = 80072 Ans: (80072)
∴15 is the least number Choice (C)
39. If the sum of the digits of a number is divisible by 9, the
number will be divisible by 9.
27. Let x = 0.222 …….. = 0.2 2 + 6 + 3 + 2 + 5 + 1 = 19
10x = 2.2 The previous multiple of 9 is 18. ∴ The remainder is 1.
10x − x = 2 Choice (D)
2 4 4
x= x2 = ≈ = 0.05 40. If the difference between the sums of alternate digits is ‘0’
9 81 80 or a multiple of 11, the number is divisible by 11.
None of the first three choices is ≈ 0.05. Choice (D) follows. (7 + 5 + 1) – (6 + 2) = 5
Choice (D) The remainder will be 5. Ans: (5)
41. The number is (891 x K + 57) = 27 x 33 x K + 27 x 2 + 3
28. y! = y (y − 1) (y − 2)!
If this number is divided by 27 the remainder will be 3.
Given
Choice (B)
y! − 20 (y − 2)! = 0
(y − 2)! (y. (y − 1) − 20) = 0 42. Let x = 0.6666
(y − 2)! (y2 − y − 20) = 0 100x = 66.66
(y − 2)! ≥1 i.e., it is ≠ 0 x = 0.66
∴ y2 − y − 20 = 0 By subtraction, 99x = 66
(y − 5) (y + 4) = 0 66 2
x= = Choice (A)
∵y>0 99 3
y = 5. Ans: (5)
43. 61 = 6, 6² = 36, 63 = 216
Q 1 2
29. P = 169 = 169 = 13 The units digit in any positive power of 6 is 6.
P, Q > 1 71 = 7, 7² = 49, 73 = 343, 74 = 2401
∴ Only possibility is P = 13 and Q = 2 The cycle of units digit in powers of 7 is 4. If 135 is divided
∴P − 3 = 10 and Q + 1 = 3 by 4, remainder is 3.
∴ (P − 3)Q + 1 = 103 = 1000. Choice (D) The units digit in 73 is 3.
The units digit of 61256 x 7135 –256 is that of 6 × 3 – 6 i.e that
30. x + y + z − 3xyz = 0
3 3 3
of 18 – 6. The units digit is 2. Ans: (2)
(x + y + z) (x2 + y2 + z2 − (xy + yz + zx)) = 0
x + y + z = 0 or x2 + y2 + z2 = xy + yz + zx 44. 162 = 2 × 9²
Choice (D) follows. Choice (D) If 162 should be made a perfect square, it should be
multiplied by 2.

31. Re m
 3 48 
 = Re m 3 ( )
4 12 16² × 2 = 2² × 9² = 324 which is a perfect square.
Choice (B)
 82 
  3 4 − (− 1)
Let x = 3 and f (x) = x12
4 4 65 2130 ( 23 )43 × 2
45. = 3 =
By remainder theorem, required remainder 7 2 −1 23 − 1
= f (−1) = (−1)12 = 1 Using remainder theorem, by substituting 23 = 1, (1)43 × 2 = 2
(∵ (−1) raised to any even number = 1). Choice (B) will be the remainder. Choice (B)
46. The L.C.M. of the given expression is
36 × 4 (72 ×18 − 1) 23 × 33 × 52 Choice (B)
32. x2 =
70 × 74
47. When a two-digit number X3 is multiplied by three digit
36 × 4 ×1295
= = 36 number ab7, the units digit should be ‘1’ and the number of
70 × 74 digits in their product should be four or five.
x=6 Ans: (6) 1521 is the only choice satisfying these conditions.
Choice (A)
33. The units digits of the given numbers in the expression are
(7 × 9 + 9 × 2) = 63 + 18 or 3 + 8 = 11 48. The smallest four digit number is 1000. If 1000 is divided by
Hence the units digits in the given expression is 1. 112, the remainder is 104.
Choice (D) 112 – 104 = 8, if 8 is added to 1000, it will become the
smallest four digit number and a multiple of 112.
34. All the fractions in the choices are of same combination. Ans: (1008)
1 2 4
= 0.5, = 0.6, = 0.8 49. The L.C.M of 72 and 108 = 216
2 3 5
216 + 33 = 249
The ascending order is 1/2, 2/3 and 4/5 Choice (A)
If 249 is divided by 72 or 108, 33 will be the remainder.
35. If a number is divisible by both 3 and 4, the number will be Choice (B)
divisible by 12.
If the last two digits are divisible by 4, the number is 50. The L.C.M. of 24, 36, 48 = 144
divisible by 4. If the sum of the digits is divisible by 3, the The largest five digit number = 99999
number will be divisible by 3. When 99,999 is divided by 144, the remainder is 63.
By applying the divisibility tests, we find 7908, 14772 and 99,999 – 63 = 99936, which is exactly divisible by 24, 36
15942 are divisible by 12. Choice (B) and 48.
99936 + 12 = 99, 948.
36. The required prime numbers are 71, 73, 79, 83, 89 and 97. The required number which leaves a remainder of 12 in
There are six numbers. Choice (D) each case is 99, 948. Choice (D)

Triumphant Institute of Management Education Pvt. Ltd. (T.I.M.E.) HO: 95B, 2nd Floor, Siddamsetty Complex, Secunderabad – 500 003.
Tel : 040–27898195 Fax : 040–27847334 email : info@time4education.com website : www.time4education.com SM1001920.Sol/21
(0.55)2 + (0.04)2 + (0.028 )2 59. (HCF)(LCM) = Product
51. ∴ (HCF) (110) = 2420
(0.055 )2 + (0.004 )2 + (0.0028 )2 ∴ HCF = 22 Ans: (22)

=
((0.055 )(10 )) + ((10.004 )(10 )) + ((10.0028 )(10 ))
2 2 2
60. 4884 = 4(1221) = 4(3)(407) = 4(3)(11)(37)
(0.055 )2 + (0.004 )2 + (0.0028 )2 = 22(31)(111)(3071)
The number of factors of 4884 = (2 + 1) (1 + 1) (1 + 1)

=
(
10 2 (0.055 ) + (0.004 ) + (0.0028 )
2 2 2
) = 10 2
= 100
(1 + 1) = 24
∴ The number of ways of writing 4884 as the product of
(0.055 ) + (0.004 ) + (0.0028 )
2 2 2
two factors = 24/2 = 12. Choice (B)
Ans: (100)
61. The numerator is of the form a3 + 3ab2 + 3a2b + b3, while
52. Any prime number more than 3 must be of the form 6k + 1 the denominator is of the form a2 + 2ab + b2 where a = 2.35
or 6k − 1 where k is a natural number. and b = 1.45
(6k + 1)2 = 36k2 + 12k + 1 = 6k (6k + 2) + 1 a3 + 3ab 2 + 3a2b + b3
=
(a + b ) = a + b = 3.8
3

 (6k + 1)2  a 2 + 2ab + b2 (a + b )2


Re m   =1
 6  Ans: (3.8)
 
(6k − 1) = 36k − 12k + 1 = 6k (6k − 2) + 1
2 2

 260   4(15 )   4
 (6k − 1)2  62. Re m   = Re m  2  and Re m  2  = 1
Re m   =1  5   5   5 
 6       
 
The powers of 2 leave a cyclic pattern of remainder 24
∴ The required remainder must be 1. Ans: (1) when divided by 5 (or 3). The cycle length is 4, i.e., 2n and
2n+4 leave the same 5’s remainder for all values of n.
53. The sum of the alternate digits starting from the units digit 30
= 8 + 3 + 2 + 5 + 6 + 9 = 33.  2 60   2( 30 )   2 
Also, Re m   = Re m  2  = Re m  2  =1
The sum of the alternate digits starting from the tens digit is  3   3   3 
7 + 1+ 7 + y + 8 + 1 = 24 + y. The 11’s remainder of the      
number is equal to the 11’s remainder of 33 − (24 + y) ∴ 2 − 1 is divisible by both 5 and 3 and
60
hence by their
= 9 − y. This is 0 as the number is divisible by 11. LCM which is 15.
∴ y = 9. Choice (D)  260 
∴ Re m   =1
 15 
54. We divide 80 successively by 2.  
2 80 Alternate method:
2 40
Re m 
 260  ( )
 4 15 
 = Re m  2 
2 20  15   24 − 1 
   
2 10 Let x = 24 and f (x) = x15
2 5 By the remainder theorem, the required remainder = f (1) = 115
=1 Choice (C)
2 2
63. Let the number be N. Let the quotient when it is divided by
The index of the highest power of 2 that divides 80! is 129 be Q.
40 + 20 + 10 + 5 + 2 + 1 = 78 Choice (A) N = 129Q + 91
55. D is odd. (∵ Dividend = Divisor × Quotient + Remainder)
∴ 41D + 7D must be divisible by 41 + 7 = 48 = 43 (3Q + 2) + 5
∴ It is divisible by any factor of 48. Only Choice (D) is not a ∴ When N is divided by 43, the quotient is 3Q + 2 and the
factor of 48. Choice (D) remainder is 5. Ans: (5)

56. Let x = 8.931 ⇒ 1000x = 8931.931 HCF (Numerators )


64. HCF (fractions) =
8923 LCM (Deno min ators )
⇒ 1000x − x = 8931 − 8 ⇒ x = .
999 HCF (7,11,3) 1
∴ HCF = =
Choice (D) LCM (18,48,5) LCM (18, 48, 5)
57. Let the greatest number be N LCM (18, 48) = 144
6875 − 25 and 8255 − 35 must be divisible by N ∴ LCM (18, 48, 5) = LCM (144, 5) = (144) (5) = 720
∴6850 and 8220 must be divisible by N ∴ HCF =
1
. Choice (C)
N is the greatest possible number satisfying this condition 720
∴ N = g.c.d (6850, 8220)
6850 = 5(1370) 65. Let the least number be x
Dividing 8220 by 1370, we get a quotient of 6 and a x must have the form 23q1 + 13 and 27q2 + 17 where q1
remainder of 0. and q2 are whole numbers.
∴ 8220 = 6(1370) x = 23 (q1 + 1) − 10 = 27 (q2 + 1) − 10
∴N = GCD of 5(1370) and 6(1370) = 1370. x + 10 = 23 (q1 + 1) = 27 (q2 + 1)
Ans: (1370) ∴ x + 10 must be divisible by both 23 and 27 and hence by
their LCM
58. Let the greatest number be N. Let the remainder be r. As x is the smallest, x + 10 is also the smallest.
103 − r, 149 − r and 241 − r must be divisible by N. ∴ x + 10 = LCM (23, 27)
∴ 149 − r − (103 − r) and 241 − r − (149 − r) must be ∴ x = LCM (23, 27) − 10 = (23) (27) − 10 = 621 − 10 = 611
divisible by N. Choice (B)
∴ 46 and 92 must be divisible by N
N is the greatest possible number satisfying this condition 66. 2, 3, 5 and 7 are the distinct prime factors of
∴ N = (46, 92) = 46. Choice (D) (222)(333)(555)(777). Ans: (4)

Triumphant Institute of Management Education Pvt. Ltd. (T.I.M.E.) HO: 95B, 2nd Floor, Siddamsetty Complex, Secunderabad – 500 003.
Tel : 040–27898195 Fax : 040–27847334 email : info@time4education.com website : www.time4education.com SM1001920.Sol/22
67. 8y3 − 6y + 3 = (2y)3 − 3 (2y) + 3 LCM(8x, 8y) = 8 LCM(x, y) = 8xy
Let x = 2y and f (x) = x3 − 3x + 3 (∵As x and y have no common factor [x, y] = xy.
By the remainder theorem, the required remainder 8xy = 504
= f (5) = 53 − 3 (5) + 3 = 113. Ans: (113) xy = 63
(1) ⇒ the only possibilities are (x, y) = (1, 63) and (7, 9)
68. B = 132A − (22)A = 132A − 22A ∴ (8x, 8y) has two possible values. Choice (A)
ap − bp must be divisible by a + b and a − b when P is an
even natural number. 2A is even 78. The least positive integer divisible by 6, 9, 12, 18 and
∴ B must be divisible by 13 + 2 = 15 and 13 − 2 = 11 24 greater than each divisor = LCM(6, 9, 12, 18, 24).
Choice (D) follows. Choice (D) 24 is divisible by 6 and 12 while 18 is divisible by 9.
∴ LCM = LCM(24, 18) = 72
69. If N is any odd natural number, 9N ends with 9 while 4N The least positive integer greater than 24, leaving a
ends with 4 remainder of 3 = LCM + 3 = 75 Choice (A)
∴ 981 ends with 9 while 479 ends with 4
∴ Units digit = that of 9 + 4 = 3. Ans: (3) 79. 4.92 = (3.1 + 1.8)2 = 3.12 + 1.82 + 2 (3.1) (1.8)
1.32 = (3.1 − 1.8)2 = 3.12 + 1.82 − 2 (3.1) (1.8)
70. 1m 50cm = 150 cm 4.92 + 1.32 = 2 (3.12 + 1.8)2
8m 10cm = 810 cm
4 .9 2 + 1 .3 2
1m 35cm = 135 cm ∴ = 2. Ans: (2)
1m 95cm = 105 cm 3.12 + 1.8 2
The length of the longest scale required (in cm)
= GCD of (150, 810, 135, 195 80. (p + q)3 − p3 − q3 = 3(p + q)(−q)(−q) = 3pq(p + q)
= GCD (15, 10, 15, 54, 15, 9, 15, 13) as (p + q) + (−p) + (−q) = 0
= 15GCD (10, 54, 9, 13) = 15 (1) = 15. Choice (B) [∵ If a + b + c = 0, a3 + b3 + c3 = 3abc]
71. Choice (A) ∴ All the three i.e., p, q and p + q are the factors of the
It can be verified that the sum of any rational number and given expression. Choice (D)
an irrational number is irrational
81. Sum of the reciprocals of 9/11 and 11/13
∴ Choice (A) is always true
Choice (B) 11 13 112 + (13)(9) 121 + 117 238
= + = = =
If x = 0, xy = 0 i.e., rational. Otherwise xy is irrational 9 11 99 99 99
∴ Choice (B) is not always true. 99
Only Choice (A) can be concluded. Choice (A) Reciprocal of this sum = . Choice (C)
238
72. The least number greater that 45 and divisible by 45 and 13
= LCM(45, 13) = (45) (13) = 585 82. Let the numbers be 3x, 4x and 5x. Least = 3x
The least number (greater than 45) leaving a remainder of LCM = x (LCM(3, 4, 5) = x (60) = 60x
7 = 585 + 7 = 592. Ans: (592) 60x = 480
x = 8, 3x = 24. Ans: (24)
73. Divisors : 9 14
83. p + q − 24p − 36q + 468 = 0 ⇒ p − 24p + q − 36q + 468 = 0
2 2 2 2

+ × ⇒ (p − 12)2 − 122 + (q − 18)2 − 182 + 468 = 0


⇒ (p − 12)2 + (q − 18)2 − 144 − 324 + 468 = 0
Remainders : 8 5 ⇒ (p − 12)2 + (q − 18)2 = 0
The least possible value of the number = 5(9) + 8 = 53 ∴ (p − 12)2 = (q − 18)2 = 0
general form of the number = [(product of the divisors) ∴ p − 12 = q − 18 = 0
k + Its least possible value] where k is any whole number ∴ p = 12 and q = 18
= 126k + 53 This is the only possible solution
126k + 53 = 18 (7k + 2) + 17 ∴ p1 = 12 and q1 = 18
 126k + 53  ∴ GCD (p1 q1) = 6. Choice (B)
∴ Re m   = 17 . Ans: (17)
 18 
13 26
84. =
74. An integer divisible by 3, 4 and 7 must be divisible by their 14 28
LCM which is 84. N
The least integer between 150 and 550 divisible by 84 is All the numbers can be expressed in the form , when
N+2
168 = 84 (2). The greatest integer between 150 and 550
N > 0. As N increases, N/N+2 increases.
divisible by 84 = 504 = 84 (6)
11 15 26
∴ The numbers 84(2), 84(3), 84(4), 84(5) and 84(6) lie between ∴ < <
150 and 550 and are divisible by 84. Choice (C) 13 17 28
11 15 13
∴ < < Choice (D)
75. For any perfect square the index of the power for each of 13 17 14
its prime factors is even. In the given number the power of
5 and 17 have odd indices, while 13 has an even index. 85. 2400 = 24(100) = (23)(3)(22)( 52) = 25(31)(52)
∴ The least number = (5)(17) = 85 Choice (C) The number of divisors it has = (5 + 1) (1 + 1) (2 + 1) = 36
∴ It has 35 divisors other than itself.
76. The number of zeros that N! ends with = The index of the Choice (C)
greatest power of 5 in it.
86. If ap x bq x cr is the number, the number of ways it can be
5 100 resolved into two factors.
5 20 (p + 1) (q + 1) (r + 1)
4 = -----------------------------------
∴ 100! ends with 20 + 4 or 24 zeros. Ans: (24) 2
(3 + 1) (3 + 1) (2 + 1)
77. GCD = 8 = --------------------------------- = 24
∴ The numbers must be of the form 8x and 8y where x and 2 Ans: (24)
y have no common factors. Let x ≤ y ….. (1)
Triumphant Institute of Management Education Pvt. Ltd. (T.I.M.E.) HO: 95B, 2nd Floor, Siddamsetty Complex, Secunderabad – 500 003.
Tel : 040–27898195 Fax : 040–27847334 email : info@time4education.com website : www.time4education.com SM1001920.Sol/23
87. The smallest even number are 2, 4, 6. Which one divisible 98. 1036 – 12 = 1024
by 48. any three successive even numbers are divisible by 1545 – 9 = 1536
48. Choice (A) The H.C.F of 1024 and 1536 = 512 Choice (B)
88. The number of factors are (2 + 1) (5 + 1) x (1 + 1) 99. Divisors = 6, 8, 9.
= 3 x 6 x 2 = 36 Choice (D) Remainders = 3, 5, 6.
The difference between the divisors and respective
89. Divisor = 7 4 2 remainders is constant.
The required number = (LCM of divisors) k – Common
difference = (LCM of 6, 8, 9) k – 3 = 72k – 3.
Remainders 3 2 1 The least four digit number = 1000.
By arrow method, the required number When 1000 is divided by 72, the remainder is 64.
= (1 x 4 + 2) x 7 + 3 = 45 Ans: (45) 1008 will be exactly divisible by 72.
90. Divisors = 10, 9 and 5 1008 – 3 = 1005 is the required number.
Remainders = 3, 4 and 2 Choice (C)
The required number = (2 x 9 + 4) x 10 + 3 = 223 100. Divisors = 5, 6, 7, 8, 9
If 223 is divided by 22 the remainder is 3. Remainders = 4, 5, 6, 7, 8.
Choice (C) The difference between the respective divisors and
91. Since (x – a) divides (x² – 125) remainders is constant.
x–a=0⇒x=a The required number = (LCM of divisors – Common
f(a) = a² – 125 = 0 difference) = 2520 – 1 = 2519 Choice (A)
a² = 125 Exercise – 9
a=5 5 Choice (C) (Number Systems)

92. 100 should be divided by 3 and the quotients should be Solutions for questions 1 to 15:
further successively be divided by 3.
100 1.
------- = 33 (quotient) 2 2 3
3 2 1 1–1
33 2 5–1
------- = 11 (quotient) 2 2–1
3 1–0
11 ∴ (23)10 = (10111)2 Choice (C)
------- = 3 (quotient)
3 2.
3
8 2 4 1
------- = 1 (quotient), the highest power of 3 in 100!
8 3 0–1
3
3–6
= 33 + 11 + 3 + 1= 48 Choice (A)
∴ (241)10 = (361)8 Choice (B)
93. The highest power of 5 in both 80! And 120! Has to be
found out and their product gives the number of zeros in 3.
80! X 120!
80 5 2 1 5
------- = 16 5 4 3–0
5 5 8–3
16 1–3
------- = 3 ∴ (215)10 = (1330)5 Choice (A)
5
16 + 3 = 19 4. Given (212)3
120 = 2 × (32) + 1 (3) + 2 (30) = 18 + 3 + 2 = 23
------ = 24 ∴ (212)3 = (23)10 Choice (D)
5
24 5. (427)8 = 4 × 82 + 2 × 8 + 7 × 80 = 256 + 16 + 7 = (279)10
------- = 4
5 4 2 7 9
24 + 4 = 28 4 6 9–3
∴ Number of zeros at the end of 80! x 120! will be 19 + 28 4 1 7–1
= 47 Choice (B) 4 4–1
1–0
94. 15n – 1n is always divisible by 15 – 1 = 14 Choice (D)
∴ (427)8 = (10113)4 Ans: (10113)
95. Since the difference between the divisors and the
respective remainders is not constant, back substitution is 6. (1101001)2
the convenient method. None of the given numbers is = 1(26) + 1(25) + 0(24) + 1(23) + 0(22) + 0(2) + 1(20)
satisfying the condition. Choice (D) = 64 + 32 + 8 + 1 = (105)10
(105)10 = (n)10 ⇒ n = 105 Ans: (105)
96. Divisors = 12, 15
Remainders = 2, 5
The difference between the divisors and respective 7. (1234)6
remainders is constant. = 1(63) + 2(62) + 3(6) + 4(60)
The required number = (LCM of 12 and 15) – (common = 216 + 72 + 18 + 4 = 310
difference) = 60 – 10 = 50. Choice (C) ∴ (1234)6 = (310)10 Choice (A)
97. 105 – 57 = 48 8. 110011
57 – 25 = 32 + 11101
105 – 25 = 80
The H.C.F of 32, 48 and 80 is 16; Choice (D) 1010000 Ans: (1010000)

Triumphant Institute of Management Education Pvt. Ltd. (T.I.M.E.) HO: 95B, 2 Floor, Siddamsetty Complex, Secunderabad – 500 003.
nd

Tel : 040–27898195 Fax : 040–27847334 email : info@time4education.com website : www.time4education.com SM1001920.Sol/24


9. (111101)2 = 1(25) + 1(24) + 1(23) + 1(22) + 0(2) + 1(20) 14. (123)4 = 42(1) + 4(2) + 40(3)
= 32 + 16 + 8 + 4 +1 = 61 = 16 + 8 + 3 = (27)10
(123)10 + (111101)2 = (123)10 + (61)10 = (184)10 (11011)2 = 1(24) + 1(23) + 0(22) + 1(2) + 1(20)
3 1 8 4 = 16 + 8 + 0 + 2 + 1 = (27)10
3 6 1–1 (123)4 × (11011)2 = 27(27) = (729)10
3 2 0–1
3 6–2 2 7 2 9
2–0 2 3 6 4–1
2 1 8 2–0
∴ (n)3 = (20211)3 2 9 1–0
n = 20211 Ans: (20211) 2 4 5–1
2 2 2–1
10. 110010 2 1 1–0
– 11011 2 5–1
2 2–1
10111 1–0
(10111)2 = 1(24) + 0(23) + 1(22) + 1(2) + 1(20) ∴ x = 1011011001 Choice (A)
= 16 + 4 +2 + 1 = (23)10 Choice (C)
11. 3 2 0
2 7 5 15. (2112)3 = 2(3 ) + 1(3 ) + 1(3) + 2(3 )
2 3 7–1 = 54 + 9 + 3 + 2 = (68)10
2 1 8–1 6 6 8
2 9–1 6 1 1–2
2 4–1 1–5
2 2–0
1–0 ∴ (68)10 = (2112)3 = (152)6 Choice (B)
(75)2 = (1001011)2
∴ It requires 7 bits Exercise – 10
(Geometry)
Alternate method:
Solutions for questions 1 to 50:
For any number N, where 2n–1 ≤ N < 2n, the number of bits
required is n. 1.
l
As 64 ≤ 75 < 128, the number of bits required is 7. 60°
Ans: (7) 60°
m p
12. (11011)2 = 1(24) + 1(23) + 0(22) + 1(2) + 1(20) x° 60°
= 16 + 8 + 2 + 1 = (27)10 ---------- (1)
(1111)2 = 1(23) + 1(22) + 1(2) + 1(20)
= 8 + 4 + 2 + 1 = (15)10 ---------- (2)
q
(1000011)2
∴ x = 180 − 60 = 120° Choice (C)
= 1(26) + 0(25) + 0(24) + 0(23) + 0(22) + 1(2) + 1(20)
= 64 + 2 + 1 = (67)10 ---------- (3)
(1) + (2) + (3) = 27 + 15 + 67 = 109 2. ∠CDX = ∠CBX = 60°
∠DEG = ∠CDX = 60° Ans: (60)

2 1 0 9 3. ∠ACD = 120° = ∠A + ∠B
2 5 4–1 As AB = AC ⇒ ∠B = ∠C
2 2 7–0 A + B + C = 180°
2 1 3–1 ∴ (109)10 = (1101101)2 A + B = 120° ⇒ C = 60° = B
2 6–1 ∴ A = 60° Ans: (60)
2 3–0
1–1 4. ∠BOC = 130° A
1
But ∠BOC = 90° + ∠A,
3 1 0 9 2
3 3 6–1 1
3 1 2–0 ∴130° = 90° + ∠A, O
∴ (109)10 = (11001)3 2
3 4–0
∠A = 80°
1–1 130°
B C
6 1 0 9 Ans: (80)
6 1 8–1
3–0
AD 1 AE 1 A
∴ (109) = (301)6 Choice (D) 5. = = =
AB 5 AC 5
13. 11001 DE 1

11001 BC 5 D E
× 11001 15
11001 ∴ DE = = 3cm
5
10101111
B C
∴ (11001)2× (111)2 = (10101111)2 Choice (C) Choice (B)

Triumphant Institute of Management Education Pvt. Ltd. (T.I.M.E.) HO: 95B, 2nd Floor, Siddamsetty Complex, Secunderabad – 500 003.
Tel : 040–27898195 Fax : 040–27847334 email : info@time4education.com website : www.time4education.com SM1001920.Sol/25
P 6 cm + 6 cm + BC = 18 cm. ⇒ BC = 6 cm.
6. PR = 62 + 82 = 10cm OB = OC = BC
1 ∴ Triangle BOC is equilateral.
= x PQ x QR Again ∠BOC = 2∠BAC (∴ the angle subtended by a chord
2 6 X
at the center of a circle is always twice the angle it
1
= x PR x QX subtends at any point on the circumference in the same
2 segment as that in which the center lies) ∴ ∠BAC = 30°.
= 6 x 8 = 10 x QX Choice (B)
⇒ QX = 4.8 cm. Q 8 R
13. ABIICD ⇒ ∠BAE and ∠EDC are alternate angles. They are
Ans: (4.8) equal
∠BAE = ∠EDC and ∠ABE = ∠ECD
7. A = 40° ∠ABE = ∠ADC (∴the angles in the same segment are
∠ACD = A + B equal.)
∠EAC = B + C Both ∠ABE and ∠ADC are angles subtended at the
B + C = 180 − A = 100° circumference by the segment AC. Now each of
∴ ∠EAC = 140° 180° − ∠AEB 180° − 100°
∴ ∠ACD = 280° − 100° = 180° ∠BAE and ∠EBA = = = 40°.
2 2
∴ A + B = 180°
Choice (B)
B + C = 140°
A + 2B + C = 280 ⇒ ∠B = 100° 14. QS is the diameter of the circle
∴ ∠C = 40° Choice (B) ∴ ∠QPS is the angle in a semicircle
∴ it is equal to 90°
8. Y ∠QSP + ∠PQS = 180° – ∠QSP = 90°.
X 2∠PQS + ∠PQS = 90°.
P 3∠PQS = 90°
Q ⇒ ∠PQS = 30°. Choice (A)
PX 2
= R
PQ 1 15. ℓ2 is a straight line.
PX. PY = PQ.PR ∴ p° + q° = 180° i.e., p° = 180° – q°
⇒ PR = 2(PY) ⇒ PQ + QR = 2(PX + XY) p° = q° + 60°
⇒ PQ + QR = 4PQ + 2XY 180° – q° = q° + 60°
QR = 3PQ + 2XY = 3 X 4 + 24 = 36 cm. Choice (B) 120° = 2q°
q° = 60°. Choice (C)
n(n − 3) 16. Let the number of sides of the polygon be N.
9. = 14 ⇒ n(n − 3) = 28
2 Let each of its interior angles and each of its exterior angles
⇒ a − 3n − 28 = 0
2 be i and e respectively.
∴ n = 7 or − 4 i = 2e -------- (1)
∴n=7 Ans: (7) i + e = 180°
2e + e = 180°
10. In a right angled triangle, the length of the median drawn to e = 60° ∴ i = 120°
the hypotenuse is equal to half the hypotenuse. 360°
The number of sides the polygon has = = 6.
In ∆PQR, ∠Q = 90°. ∴PR is a hypotenuse. 60°
PR [sum of the exterior angles of a polygon is 360°]
Required length =
2 Sum of its interior angles = (120°) (6) = 720.
Alternate method:
PR (in cm) = PQ2 + QR2
(1) ⇒ Sum of interior angles of the polygon = 2(sum of its
5 .4 2 + 7 .2 2 = ((1.8)(3) )+ ((1.8)(4) ) = 1.8 (5) = 9
2 2 exterior angles) = 2(360°) = 720°. Choice (C)

PR 17. A maximum of four tangents can be drawn to two circles.


∴ = 4.5 cm. Ans: (4.5) The four tangents that can be drawn to the two circles are
2
shown below:
(T1 – T4 are the tangents).
11. The shape of the quadrilateral can be any of the ones give
below. T1
Case 1: Case 2: T3
A B A B
120° 100° T4
100° 120°
4
T2
Ans: (4)
60° 80° 60° 80°
18. RQP and RST are secants
D C D C ∴(RQ) (RP) = (RS) (RT).
RP = RT (Given)
In case 1, ∠A + ∠D = ∠B + ∠C = 180° ∴ AB II CD. ∴ RQ = RS
∴ ABCD is a trapezium. ∴ RS = 4 cm.
In case 2, ∠A + ∠C = ∠B + ∠D = 180° RT = RS + ST = 8 cm. Ans: (8)
∴the opposite angles of ABCD are supplementary.
∴ it is a cyclic quadrilateral. 19. ∠PRS = 120°.
In any case, ABCD must be a trapezium or a cyclic ∠PQR + ∠QPR = ∠PRS (∴the exterior angle of a triangle
quadrilateral. Choice (D) is equal to the sum of the opposite interior angles).
2(∠QPR) + ∠QPR = 120°.
12. OB = OC (∴ the radii are equal). 3 ∠QPR = 120°.
∴ OB = 6 cm. The perimeter of triangle BOC = OB + OC + ∠QPR = 40°. Choice (C)
BC = 18 cm.
Triumphant Institute of Management Education Pvt. Ltd. (T.I.M.E.) HO: 95B, 2nd Floor, Siddamsetty Complex, Secunderabad – 500 003.
Tel : 040–27898195 Fax : 040–27847334 email : info@time4education.com website : www.time4education.com SM1001920.Sol/26
20. PQ and QR are tangents to the circle at P and R 26.
respectively. A
∴ ∠QPO = ∠QRO = 90° D
∠QPO + ∠POR + ∠QRO + ∠PQR = 360' P
∴ ∠POR + ∠PQR = 180°
∴ ∠POR + 30° = 180°
∠POR = 150° C B
∠POR = 2∠PSR
(Angle subtended by a chord at the centre of a circle is Here, PA ⋅ PB = PC ⋅ PD Choice (A)
twice the angle subtended by it at any point on the 27. PS is the bisector of ∠P
circumference in the same segment as which the centre PQ QS
lies) ∴ = --------- (1) P
PR SR
∴ ∠PSR = 75°. Choice (C)
STIIPR T
21. ∠P + ∠Q + ∠R = 180°------ (1) QT QS
∴ = --------- (2)
QS bisects ∠Q TP SR
∴ ∠SQR = ∠ Q /2. PQ QT QS Q S R
(1) and (2) ⇒ = = .
RS bisects ∠R PR TP SR
∴ ∠SRQ = ∠R/2
PT PR
∠SQR + ∠SRQ + ∠QSR = 180° ∴ = . Choice (D)
∠Q/2 + ∠R /2 + 2∠P = 180° TQ PQ
∠Q/2+ ∠R/2 + 2 ∠P = 180° 28. The ratio of the sides opposite to the angles A, B and C is
(1) ⇒ ∠Q + ∠R = 180° – ∠P sin A: sin B : sin C = sin 90° : sin 45° : sin 45°
(180° – ∠P)/2 + 2∠P = 180° B
∠P = 60°. Choice (D)
45°
22. QP is a tangent to the circle at P and QRS is a secant.
∴QP2 = QR . QS
Dividing both sides by QS2
2
 QP  QR
 QS  = QS
  90° 45°
QR
∵ <1 A C
QS
1 1
2 =1: : = 2 : 1: 1 Choice (B)
 QP  QP
∴   < 1 i.e. QS < 1 i.e QP < QS 2 2
 QS 
Only Choice (A) violates this condition. Choice (A) 29.
X
23. Triangle PQR is equilateral.
∴ ∠PQR = 60°.
QRST is a rectangle.
∴∠RQT = 90°.
∠PQT = ∠PQR + ∠RQT = 150°.
QU bisects ∠ PQT.
∠PQT
∴PQU = = 75°. Y P Z
2
According to Angle bisector theorem,
∠RQU = ∠PQU – ∠PQR = 15°
∠QUS = ∠QRU + ∠RQU = 90° + 15° = 105°
XY YP
= Choice (B)
(∵ the exterior angle of a triangle equals the sum of the XZ PZ
interior angles opposite to it). Choice (C) 30. Radius is perpendicular to the tangent at the point of
tangency:
24. ∠R = 180° – (∠P + ∠Q) = 90° OR bisects ∠ORQ. Therefore ∠ORQ = ∠QRS = 45°
∴ the triangle has angles 30°, 60° and 90°. In any triangle OQ = OR (∴ the radii are equal).
having angles 30°, 60° and 90°, the sides opposite to these ∴∠OQR = ∠ORQ
angles must be in the ratio 1 : 3 : 2. ∴∠OQR = 45°.
∴∠QOR = 180 – (∠OQR + ∠ORQ) = 180°– 90° = 90°
∴ in ∆ PQR, PR : QR = 1 : 3 . Thus ∠QPR = ½ ∠QOR = 45° Choice (A)
1
∴PR = QR = 2 3 cm. Choice (B) 31. P1, P2, and P3 are three lines such that P1 II P2.and P2II P3.
3 It follows that P1IIP3.
∴ L1 II L2, L2 II L3 ⇒ L1 II L3.
25. Given AB = 6 cm and BC = 8 cm. ∴ Choice (A) follows ---------- (1)
AC will be 10 cm ∴ L1 II L3 II L1, L1 ⊥ L4 ⇒ L2, L3 ⊥ L4.∴ Choice (B) and (C)
( Q AC2 = AB2 + BC2). follow ------- (2)
Also BD will be 10cm. (1) and (2) ⇒ Choice (D) follows. Choice (D)
As CE ⊥ BD and BCD is a right angled triangle, we have
32. Let the radius of the circle be r cm. Then BA = (r + 6) cm
1 1
x 6 x 8 = x 10 x CE ⇒ CE = 4.8cm BCD is a secant and AB is a tangent to the circle at A.
2 2 ∴ BA2 = BC.BD
As ∆DCE is right angled, CE2 + DE2 = CD2 i.e (r + 6)2 = 12 × (12 + 2r)
⇒ (4.8)2 + (DE)2 = 62 ⇒ r2 – 12r – 108 = 0
DE2 = (6 + 4.8) (6 − 4.8) ⇒ (r – 18) (r + 6) = 0 ⇒ r = 18
⇒ (DE)2 = 12.96 ⇒ DE = 3.6 cm Ans: (3.6) ⇒ BA = 24 cm Ans: (24)

Triumphant Institute of Management Education Pvt. Ltd. (T.I.M.E.) HO: 95B, 2nd Floor, Siddamsetty Complex, Secunderabad – 500 003.
Tel : 040–27898195 Fax : 040–27847334 email : info@time4education.com website : www.time4education.com SM1001920.Sol/27
33. The maximum number of diagonals which can be drawn in 41. ∠QOP = 2∠QPR = 2 (30°) = 60°. (∴ Angle subtended by a
N (N − 3) chord at the center of a circle is always twice the angle it
a regular polygon of N sides = . subtends at any point on the circumference in the same
2
segment)
12 (12 − 3)
Required number of diagonals = = 54. OQ = OR (∴ Radii are equal)
2
180° − ∠QOR
Ans: (54) ∴ ∠OQP = ∠QRO = = 60°
2
34. Let the number of sides the polygon has be N. ∴ Each angle of ∆QOR = 60°.
The polygon is regular ∴ ∆QOR is equilateral. ∴ Its perimeter = 3QR = 15 cm.
∴ Each interior angle is the same. Ans: (15)
∴Each interior angle = 135°
∴Each exterior angle = 180° – 135° = 45° 42. ∴∠PQT = ∠TRS ( Q PQ II RS ⇒ ∠PQT and ∠TRS are
360° alternate angles. ∴ They are equal) ––––––––– (1)
∴ = 45° ⇒ N = 8 Ans: (8) ∠QPT = ∠TRS ( Q Angles in the same segment are equal) -(2)
N
(1) and (2) ⇒ ∠PQT and ∠QPT ∴ ∠PQT = 30°.
35. The point of intersection of the perpendicular bisectors is Ans: (30)
called circumcentre. Choice (D)
43. BD is a diameter of the circle.
36. Two circles cannot intersect at more than two points ∴ ∠BCD is an angle in a semicircle
( Q Through any three or more points, a maximum of only ∴ It equals 90°
one circle can pass. ∴ Two circles cannot intersect at three ∠BCD + ∠DBC = 180° – ∠BCD = 90°
or more points). Ans: (2) ∠DBC – 20° + ∠DBC = 90°
2∠DBC = 110°
37.
P ∠DBC = 55°. Choice (B)

44. ℓ2 is a straight line.


∴ a° + b° = 180° i.e., b° = 180° – a°
2a° – 4b° = 90°
2a° − 90° a° − 45°
Q R ∴ b° = =
S 4 2
a° − 45°
PS = PQ – QS and PS = PR2 – RS2
2 2 2 2
180° – a° =
2
∴ PS2 = PQ2 – QS2 = PR2 – RS2 –––––– (1)
360° – 2a° = a° – 45
Choice (A) ⇒ PQ2 + QS2 = PR2 + PS2 which is not following
a° = 135°. Choice (D)
from (1).
Choice (B) is equivalent to (1) 45. OA = OC (∴ Radii are equal)
∴ Only Choice (B) must be true. Choice (B) ∴ ∠OAC = ∠OCA
38. PTS and PQR are secants. ∴ ∠OAC = 40°
∴ (PT) (PS) = (PQ) (PR) ∠OAC + ∠CAD = 90°, ( Q O is the center of the circle and
PT (PT + TS) = (PQ) (PR) AD is a tangent to the circle at A. The line joining a tangent
4 (4 + 8) = 3(PR) at the point of contact with the circle and its center is ⊥ to
48 = 3PR the tangent. (∴ ∠OAD = 90°))
PR = 16 Ans: (16) ∴ ∠CAD = 50°. Choice (A)
39. Ratio of corresponding sides of two similar triangles 46. ∠P + ∠Q + ∠R = 180°
= Ratio of their respective perimeters. ∴ ∠P + ∠R = 180° – ∠Q –––––––––(1)
For the given triangles, perimeters are equal.
∠P – ∠Q = ∠Q – ∠R
∴ Ratio of their perimeters = 1 : 1. ∴ Ratio of their
∠P + ∠R = 2∠Q
corresponding sides = 1 : 1.
(1) ⇒ 180° – ∠Q = 2∠Q
∴ The triangles are congruent and hence have equal
180° = 3∠Q
areas.
∴ Choice (A) and (B) follow. ∴ Choice (C) follows. 60° = ∠Q Choice (A)
Choice (C) 47. The maximum number of diagonals which can be drawn in
N (N − 3)
40. Case 1: a regular polygon of N sides = .
2
A B
N (N − 3)
144° 108° = 3N
2
N (N – 3) = 6N
36° 72° As N ≠ 0,
D C N–3=6⇒N=9 Choice (B)
In this case, ∠A + ∠D = ∠B + ∠C = 180° ∴ ABIICD.
∴ ABCD is a trapezium. 48. In triangle BCD, ∠C = ∠D (angles opposite to equal sides)
∠C = ∠D = 65°
Case 2:
A B ∠ABD = ∠C + ∠D (Sum of interior angles)
= 65 + 65 = 130°
108° 144°
∠BDA = 180° – {∠ABD + ∠BAD}
= 180° – (130 + 30) = 180 –160 = 20°. Choice (D)
36° 72°
49. A
D C

In this case, ∠A + ∠C = ∠B + ∠D = 180° ∴ Opposite


angles of ABCD are supplementary. ∴ It is a cyclic quadrilateral.
ABCD can be a trapezium or a cyclic quadrilateral. B D C
Choice (D)
Triumphant Institute of Management Education Pvt. Ltd. (T.I.M.E.) HO: 95B, 2nd Floor, Siddamsetty Complex, Secunderabad – 500 003.
Tel : 040–27898195 Fax : 040–27847334 email : info@time4education.com website : www.time4education.com SM1001920.Sol/28
AD2 = AB2 – BD2 and AD2 = AC2 – CD2 7. a = 16m
∴ AD2 = AB2 – BD2 = AC2 – CD2 Area of wall = 16 × 16
∴ AB2 – AC2 = BD2 – DC2 ---------- (1) Area of Tile = 0.4 × 2
Choice (A) must be true. 16 × 16
(1) ⇒ AB – DC = BD – AC
2 2 2 2 ∴ No. of tiles needed = = 320 Choice (C)
0.4 × 2
∴ Choice (B) does not follow. Choice (A)
50. Let the radius of first circle be ‘r1’ = 3 cm and that of second 8. l = 13ft, h = 120 ft
circle be ‘r2’ = 4 cm.
Given that their centres are at a distance of ‘d’ = 8 cm converse needed = C.S.A = π rl
When this distance is reduced by 1 cm, then (d – 1) = r1 + r2
r = l2 − h 2 = 7ft
The two circles now become externally touching circles.
∴ 3 common tangents can be drawn to them. ∴ C.S.A = 22/7 × 13 × 7 = 286 sqft. Ans: (286)
Choice (B)
9. r = 7cm ⇒ 2r = 14 cm = a 2
Exercise – 11
(Mensuration) (a is side of square)
⇒ a = 14/ 2 = 7 2
Solutions for questions 1 to 50:
Perimeter of the square = 4a = 4 × 7 2
1.
= 28 2 cm Choice (C)

13 13 10. Volume of the wall = 15m × 12m × 10cm


= 1500 × 1200 × 10cc
5 Volume of the brick = 24 × 25 × 10cc
1500 × 1200 × 10
10 ∴ Total bricks molded = = 3000
25 × 24 × 10
h= 132 − 52 = 12 Since 2/3 of the wall is already built, only 1/3rd of the
rd

1 bricks are needed = 1/3 (3000) = 1000 Ans: (1000)


Area = × 12 × 10 = 60 cm² Ans: (60)
2
11. Let the radius and the height of the cylinder be r and L
2. a = 14cm, b = 48cm, c = 50 cm respectively.
This is a right angled triangle 2πr L = 3 πr2
As a2 + b2 = c2 πr (2L – 3r) = 0.
∴ Area = ½ (a) (b) = 336 cm2 Choice (B) ∵ πr ≠ 0
⇒ 2L – 3r = 0 i.e. 2L = 3r.
3. Perimeter = 52cm = 4a A L 3
∴ a = 13cm = . Choice (D)
d = 10 cm = AC r 2
13 cm
∴ AE = 5 cm
E 12. Let the sides of the triangle and the square be s and a
∴ ED = 13 2 − 5 2 = 12cm B C respectively.
3 2
∴ Area of ABCD = 4 × ½ (AE) (ED) Area of the triangle = s.
4
1 2
=4× × 5 × 12 = 120 sqcm Area of the square = a .
2 D
3 2
Ans: (120) s = a2.
4
Perimeter of the triangle = 3s.
4. Let the side of the hexagon be H cm. Perimeter of the square = 4a.
6h = 24
h=4 3 2
(4a)2 = 16a2 = 16( s ) = 4 3 s2
4
3 3
Area of a regular hexagon = (Its side)2
2 Now, 4 3 s2 < 4 (2) s2 < 9s2
(4a)2 < (3s)2
3 3
∴ Required area = (4)2 = 24 3 cm2 ∴ 4a < 3s. Choice (C) follows. Choice (C)
2
Choice (C)
13. Let the length and the breadth of R be ℓ and b respectively.
Let the side of S be a.
5. Let the length and the breadth of the rectangle be 8x and
l +b
3x cm respectively. a= .
Length of the wire = Perimeter of the rectangle. 2
264 = 2(8x + 3x) l+b
Diagonal of S = 2 a =
264 = 22x 2
x = 12
Required area (in sq cm) = (8x) (3x) = 24x2 Diagonal of R = l 2 + b2
= 24(122) = (24) (144) = 3456 Ans: (3456)
( l 2 + b 2 )2 2(l 2 + b 2 )
6. Area to be leveled (in sq m) Required ratio = = which will
= Area of the outer circle – Area of the inner circle  l+b
2
(l + b 2 + 2lb)
2

= π(212) – π(14)2 = π [(212) – (142)] = π(441 – 196)  


 2 
22  
= (245) = 770 be always less than two but if ℓ = b, it will be equal to 1.
7
Cost of leveling (in Rs) = (4) (770) = 3080 Ans: (3080) Choice (C)

Triumphant Institute of Management Education Pvt. Ltd. (T.I.M.E.) HO: 95B, 2nd Floor, Siddamsetty Complex, Secunderabad – 500 003.
Tel : 040–27898195 Fax : 040–27847334 email : info@time4education.com website : www.time4education.com SM1001920.Sol/29
14. The maximum number of cubical boxes which can be 21. ABCD, TUVW and PQRS are rectangles.
Volume of the big box 30 3 30 ∴ TW = UV = AD and PS = QR = AB.
placed = = = ( )3 The area of TUVW = (TU) (TW) = (4 cm) (8 cm) = 32 sq cm.
Volume of each small box 53 5 The area of PQRS = (PQ) (PS) = (4 cm) (12 cm) = 48 sq cm
= 63 = 216. Ans: (216) The area common to TUVW and PQRS = (PQ) (RS)
= (4 cm) (4cm) = 16 sq cm.
15. Each cube has an edge of 8cm. The common area (in sq cm) = 32 + 48 – 16 = 64.
∴ the cuboid formed on joining both the cubes will have a Choice (D)
length of 16 cm, a breadth of 8 cm and a height of 8 cm. Its
lateral surface area (in sq cm) = 2(8) (16 + 8) = 384. 22. Let the radius of each ball be b cm.
Ans: (384) Let the radius of the rod be r cm.
The volume of each ball = 4/3 π b3 cubic cm.
16. Let the two sides forming right angle be 3k and 4k The total volume of the balls = 162(4/3 π) b3 cubic cm
respectively. = 216 π b3 cubic cm.
Now, area =
1
(3k )(4k ) = 54 The volume of the cylinder (in cubic cm) = πr2 (r) = π r3
(∴ radius of the rod = Its height)
2
⇒ 12k2 = 108 ⇒ k = 3 216π b3 = π r3
∴ r3 = 216b3 = (6b)3
∴ Hypotenuse = (3k )2 + (4k )2 = 5k = 15 cm. ∴ r = 6b i.e r/b = 6/1. Choice (C)
Choice (C)
23. AB + BC + CA = 18 cm (we have, AB < BC < CA.
17. The length of each rope = (the circumference of the From Choice (A)
cylinder on which it makes rounds) (the number of rounds it If BC = 4.5cm, AB < BC ⇒ AB < 4.5 cm
can make on that cylinder). In a triangle, the sum of any two sides is more than the
The length of R1 = (π 60) (49) cm. third side.
The length of R2 = (π 42) (70) cm. ∴BC + AB > CA
The length of R1 – the length of R2 = (π.60) (49) cm – BC + AB < 9 cm
(π.42) (70) cm. = 2,940 π cm – 2,940 πcm = 0 cm. ∴CA < 9cm.
Choice (D) But (1) ⇒ CA = 18 cm – (AB + BC) which is > 18 cm
– 9 cm, i.e., 9 cm
18. The third side of the triangle = 36 cm – (12 cm + 15 cm)= 9 cm. We have a contradiction.
∴the sides satisfy Pythagoras theorem. ∴ Choice (A) cannot be the length of BC.
∴the triangle is right–angled and has perpendicular sides Choice (B)
of lengths 12 cm and 9 cm. If BC = 9 cm , CA > 9 cm.
∴ Its area (in sq cm) ∴BC + CA > 18 cm.
= 1/2 (12) (9) = 54. Choice (B) This is not possible.
∴ Choice (B) cannot be the length of BC. Hence Choice
19. (D) follows. Choice (D)
A C B
24. Let the radii of C1 and C2 be r1 and r2 respectively.
Let the heights of C1 and C2 be h1 and h2 respectively.
O 2π r1 h1 4 r1 2
= and =
2π r 2 h2 9 r2 3
r1 h1 4 r1 2
∴ = and =
Let O be the center of the octagon. It would also be the r 2 h2 9 r2 3
center of the circle ∴ OA = OB. r1 h1 4
Let OC be the ⊥ bisector of AB. ------ (1) r 2 h2 h1 2
= 9 ⇒ = . Choice (A)
∠AOB
Then ∠AOC = . r1 2 h2 3
2 r2 3
∠AOC = 180° – (∠OAB + ∠OBA)
180° − (∠OAB + ∠OBA ) 25. Let the parallelogram and the triangle be ABCD and ABE
∴ ∠AOC =
2 respectively where AB is the common base of both.
∵ ∠A = ∠B (Octagon is regular). ------ (2) D E C
Each of these angles is 135°.
∴∠A/2 = ∠B/2 = 135°/2
135
180° − 2 ( )
∴∠AOC = 2 = 45°/2.
2 A B
(1) = ∠OCA = 90°.
AC/OA = Sin 45°/2. ABIICD and AB = CD. Again E lies on CD.
∴ AC = 8 sin 45°/2. Area of ABCD = (AB) (AD).
(1) = AB = 2AC = 16 sin 45°/2. Area of ABE = 1/2 (AB) (AD) (the height of ∆ABE = AD).
∴ perimeter = 8AB = 128 sin 45°/2. Choice (B) The ratio of the areas of ABCD and ABE =
(AB)( AD)
1
( AB) ( AD)
20. Let the shorter and the longer of the parallel sides be of 2
lengths s cm and l cm respectively. = 2 : 1. Choice (D)
s + l + 2(26) = 124.
s + l = 72. 26. The area of the track (in sq m) = π (142 – 72) = 22/7
(196 – 49) = 22/7 (147) = 22 (21) = 462.
Area (in sq cm) = 1/2(s + l ) (10) = 1/2 (72) (10) = 360.
The cost of levelling the track (in `) = (462) (6) = 2,772.
Choice (D) Choice (A)

Triumphant Institute of Management Education Pvt. Ltd. (T.I.M.E.) HO: 95B, 2nd Floor, Siddamsetty Complex, Secunderabad – 500 003.
Tel : 040–27898195 Fax : 040–27847334 email : info@time4education.com website : www.time4education.com SM1001920.Sol/30
27.
A 4π(2x )2  2x 
2
2
2
4
Required ratio = =   =   =
2
4π(3 x )  3x  3 9

β Choice (B)

32. Volume of the sheet = (Its area) (Its thickness) = (60,000 sq


m) (1.5 × 10–3 m) = 90,000 × 10–3 cubic m = 90 cubic m.
C B Choice (C)

Let C be the center of the cup’s base. 3 3 2


BC = 3 cm 33. Area of a regular hexagon of side a is given by a .
2
Tanβ = BC/AB
Let the sides of the first and second hexagons be 2x and 3x
3 respectively.
AB = cm.
tan β Ratio of the areas of the first and the second hexagons
1π 1π  3  3 3
Volume of the cup = (BC)2 (AB) = (3)2   (2x ) 2
3 3  tan β  2 4x 2 4
= = = . Choice (B)
2 9
9π 9π 9π 3 3 2 9 x
cubic cm = . Now = 24 i.e tanβ = i.e (3 x )
tan β tan β 24 2
3
tanβ = π
8 34. Area of isosceles triangle = b a2 − b2 ,
 3π  where the base is 2b and each equal side is a.
∴β = tan–1   Choice (A)
 8  Given 48 = b 102 − b2  (1)
28. The circumference of the wire’s initial circular form = its From the choices, 2b = 16 ⇒ b = 8, satisfies the equation (1).
present rectangular form’s perimeter. Also 2b = 12 ⇒ b = 6 satisfies eq (1). Choice (C)
Let its present sides be 7x cm and 4x cm respectively
Its present perimeter = 2(7x + 4x)cm = 22x cm. 35. Let the radii of C1 and C2 be 4x and 3x respectively.
Its initial circumference = 2π (31.5cm) = 63πcm Let the heights of C1 and C2 be 9y and 16y respectively.
22x = 63π 1
Volume of C1 = (4x)2 (9y)
 22  3
63   1
x=  7  =9 Volume of C2 = (3x)2 (16y)
22 3
⇒ x = 63. Ans: (63) 1 1
Required ratio = (4x)2 (9y) : (3x)2 (16y)
3 3
29. PQ + PR = PQ + QR ⇒ PR = QR -------- (1)
PQ + QR = 8 cm ----------- (2) x2y x2y
= (16) (9) : (9) (16) = (16) (9) : (9) (16) = 1 : 1
Perimeter = PQ + QR + PR = 12 cm. 3 3
(2) ⇒ 8 cm + PR = 12 cm. Choice (A)
∴PR = 4 cm.
(1) ⇒ QR = 4 cm. 36. Let the radius of each sphere be r cm
(2) ⇒ PQ = 4 cm. 4
Volume of each sphere = π r3 cubic cm.
PQ = QR = PR 3
∴ Triangle PQR must be equilateral. Choice (B) 4 3
Total volume of the spheres = N. πr cubic cm.
30. 3
Radius of the cylinder = Its height = 4r cm.
P Q
Its volume = π (4r)2 (4r) cubic cm.
4 3
N. πr = π (4r)2 (4r)
3
π ( 64 r 3 ) 64
N= = = 48. Ans: (48)
4 3 4
S R πr
3 3

Let PQRS be the rectangle. 37. Let the sides of the triangle and the square be s and a
Then ∠QPS = 90° respectively.
(Angle in a semicircle is 90°) Perimeter of the triangle = 3s.
∴ QS must be the diameter of the circle. Perimeter of the square = 4a
2 3s = 4a
5
QS = PQ 2 + PS 2 = 6 2 + 2 .5 2 = 36 +   4a
2 s=
3
 25  144 + 25 169 13 3 2 3  4a 
2
3
= 36 +   = = = = 6.5m.
 4  4 4 2
Area of the triangle = s =  3  =
4 4   4
Ans: (6.5)
 16 2  4 3 2
 a  = a
31. Let the radii of the first and the second spheres be 2x and  9  9
3x respectively. Area of the square = a2
Curved surface area of the first sphere = 4π (2x)2
Curved surface area of the second sphere = 4π (3x)2 3 <2

Triumphant Institute of Management Education Pvt. Ltd. (T.I.M.E.) HO: 95B, 2nd Floor, Siddamsetty Complex, Secunderabad – 500 003.
Tel : 040–27898195 Fax : 040–27847334 email : info@time4education.com website : www.time4education.com SM1001920.Sol/31
∴ 4 3 < (4) (2) 3 3 2
45. Area of a regular hexagon of side a is given by a
2
4 3 8
∴ < . 2
9 9 3 3  18  3 3
Area of the hexagon (in sq cm) =   = (9)
∴ Area of the triangle is <
8 2.
a ∴It is less than the area
2  6  2
9
27 3
of the square. Choice (C) = 13.5 3 Choice (A)
2
38. Let the length and the breadth of the rectangle be ℓ and b 46. Let the radius and the height of the cylinder be r and h
respectively. respectively.
Diagonal of the rectangle = l 2 + b2 2 Π r(r + h) = 4 ( Π r2)
(r + h) = 2r; r=h
Diagonal of the square = 4 2 cm. ∴r:h=1:1 Choice (D)

l 2 + b2 = 4 2 47. Let the heights of C1 and C2 be h1 and h2 respectively


Let the total surface areas of C1 and C2 be T1 and T2
As ℓ = 3 b, l2 + b2 = 3b2 + b2 = 4b2 = 2b respectively.
Let the radii of C1 and C2 be r1 and r2 respectively. Given
2b = 4 2 ⇒ b = 2 2 Choice (B) h
that 1 = 1 =
r 3
.
h2 r2 5
39. Maximum number of boxes which can be placed =
24 m ×18 m ×10 m T1 2Π r1 ( r1 + h1) r ( r1 + h1 ) 3 3
Volume of the big box
= = = 1 = ×  
Volume of each small box 4m×3m×2m T2 2Π r2 (r2 + h2 ) r2 (r2 + h 2 ) 5 5
24 18 10 9 a c a+c
= × × = 6 × 6 × 5 = 36 × 5 = 180. = (If = , each of these two equals .
4 3 2 25 b d b+d
Ans: (180) h1 r h +r
So, = 1 = 1 1 ). Choice (C)
h2 r2 h 2 + r2
40. Each cube has an edge of 40 cm.
∴ The cuboid formed on joining both cubes will have a 48. Let the sides of the triangle be a cm, b cm and c cm such
length of 80 cm, a breadth of 40 cm and a height of 40 cm. that a < b < c.
Its surface area (in sq cm) a + b + c = 12.
= 2 ((80) (40) + (80) (40) + (40) (40) a+b+c<c+c+c
= 2 (3,200 + 3,200 + 16,00) = 2 (8,000) = 16,000. 12 < 3 c i.e., 4 < c
Ans: (16,000) In a triangle, sum of any two sides must be more than the
41. Let the number of rounds it can make be N. third side.
Length of the rope = 70 (circumference of the first cylinder) ∴a+b>c
= 70 (π.56) cm. This also equals N (π.49) cm ∴ 12 – c > c i.e., 12 > 2 c i.e., 6 < c.
= 70 (π.49) cm = N (π.49) cm 4<c<6
70.56 Only Choice (B) violates this condition. Choice (B)
⇒N=
49 49.
7.10.7.8
⇒N= ⇒ N = 80 Choice (C)
49 2
2 2 2 2 2 2 2 2
42. Let the thickness be t m
40,000.t = 2
Length of the cuboid = 2(8) = 16 cm
2 1 Breadth of the cuboid = 2 cm
t= =
40,000 20,000 Height of the cuboid = 2 cm
1 1 1 Surface area = 2 (lb + lh + bh)
m= cm = mm = 0.05 mm = 2[(16 × 2) + (16 × 2) + (2 × 2)]
20,000 200 20
= 2(32 + 32 + 4) = 136 cm²
Ans: (0.05) Ans: (136)
43. 50. Let the distance between the parallel sides be h.
60 m
1
Area of trapezium = h (a + b)
A B 2
I J 1
40 m K L H 6m But Area of trapezium is given as × (a3 + b3)
2
J
C D 1 1 3
⇒ h(a + b) = (a + b3)
2 2
6m
⇒ h(a + b) = (a + b)(a2 − ab + b2); ⇒ h = (a2 − ab + b2)
Combined area of the roads = Area of ABCD + Area of
∴ the distance between the parallel sides = a2 – ab + b2
EFGH – Area of IJKL = [(60) (6) + (60) (6) – (6) (6)] Sq m
Choice (D)
= [360 + 240 – 36] sq m = 564 sq m (∴ As IJLK is a region
which occurs in ABCD as well as EFGH, its area will be a Exercise – 12
part of the areas of ABCD and EFGH). (Coordinate Geometry)
Its area is considered twice. ∴ Its area has to be subtracted
Solutions for questions 1 to 40:
once.
Cost of repairing (in `) = (564) (4) = 2,256. Choice (C) 1. The point of concurrence of the altitudes of a triangle is
orthocenter.
1 For a right angled triangle the orthocentre is the vertex
44. Area of the trapezium (in sq cm) = (12 + 18) (8) = 120
2 containing right angle.
Ans: (120) ∴ orthocentre = (3, 2) Choice (B)

Triumphant Institute of Management Education Pvt. Ltd. (T.I.M.E.) HO: 95B, 2nd Floor, Siddamsetty Complex, Secunderabad – 500 003.
Tel : 040–27898195 Fax : 040–27847334 email : info@time4education.com website : www.time4education.com SM1001920.Sol/32
x y 6 – k = ± 2; k = 6 m 2
2. The equation of a line in intercept form is + = 1.
a b k = 4 or 8 Ans: (4)
Given a : b = 1 : 2
9. Given 5x – 2y = 7 – (1)
a 1
= ⇒ 2a = b 7x + 5y = 2 – (2)
b 2 kx – y = 4 – (3)
x y The point of intersection of (1) and (2) is (1, –1)
∴equation of the line is + =1 Given the lines are concurrent,
a 2a
∴(1, –1) lies on the 3rd line.
2x + y k (1) – (–1) = 4
=1
2a k = 4 – 1; k = 3 Ans: (3)
2x + y = 2a
It is passing through the point (2, 4) 10. The slope of the line joining the points (2, 3) and (–1, 3) is
4 + 4 = 2a ⇒ a = 4 3−3
=0
∴ required equation is 2x + y = 8 Choice (C) − 1− 2
3. Given, 8x – y = 6 – (1) The slope of the required line is also ‘0’.
9x + 2y = 13 – (2) The equation of a line whose slope is ‘0’ and passing
Solving (1) and (2) we get through the point (1, 1) is y – 1 = 0 (x – 1)
x = 1 and y = 2 y – 1 = 0. Choice (C)
(x, y) = (1, 2) Choice (D)
11. The equation of a circle making intercepts on positive
4. If (x1, y1), (x2, y2) and (x3, y3) are collinear, then x–axis and positive y–axis of a and b respectively and
1 x1 − x 2 y 1 − y 2 −5 − 3 2 − k passing through (0, 0) is x2 + y2 – ax – by = 0.
=0⇒ =0 The equation of the required circle is
2 x2 − x3 y2 − y3 3+2 k−4
x2 + y2 – 2x – 6y = 0. Choice (A)
−8 2−k
=0 12. Given intercepts of the line are equal
5 k−4 ∴a=b
[–8 (k – 4) – 5 (2 – k)] = 0 x y
–8k + 32 – 10 + 5k = 0 ∴ equation of the line is + = 1
a a
– 3k + 22 = 0
x+y=a
3k = 22 ⇒ k = 22/3 Choice (D)
Since the above line is passing through (–1, 3),
5. Slope of 4x – 5y + 7 = 0 is –1 + 3 = a ⇒ a = 2
−4 4 ∴ required equation is x + y = 2 Choice (B)
=
−5 5
13. If the image of the point (x1, y1) w.r.t. the line ax + by + c = 0
h − x1 k − y1 −2 (ax1 + by1 + c )
Slope of a line which is perpendicular to 4x – 5y + 7 = 0 is
−5/4 and y intercept is 2. is (h, k), then = =
We know that the equation of a line whose y – intercept is c a b a2 + b2
and slope m is y = mx + c. h −1 k − 2 −2 (1− 4 + 4 )
∴ required equation is = =
1 −2 1+ 4
−5
y= x+2 h −1 k − 2 −2
4 = =
4y = –5x + 8 1 −2 5
5x + 4y – 8 = 0 Choice (C) −2 k − 2 −2
h–1= and =
6. Given equations are 6x + 7y = 5 – (1); 2x + 3y = 1 – (2); 5 −2 5
kx – 5y = 13 – (3) 2 4
The point of intersection of (1) and (2) is (2, –1) h=1– and k – 2 =
5 5
Since, the given lines are concurrent, the point (2, –1) lies
on (3) 3 14
h= and k =
∴k (2) –5 (–1) = 13 5 5
2k = 13 – 5 = 8 ⇒ k = 4 Ans: (4)  3 14 
Image is  ,  Choice (D)
7. x + 2y + λ = 0 – (1); µx – 8y – 12 = 0 – (2) 5 5 
Equations (2) and (1) represent the same line.
µ −8 −12 14.
= = . C (k, 4)
1 2 λ
µ = –4; λ = 3
∴ λµ = (3) (−4) = –12. Ans: (–12)
8. Area of the quadrilateral formed by the vertices
(x1 , y1) , (x2, y2), (x3, y3) and (x4, y4) is
1 x1 − x 3 y1 − y 3
2 x2 − x4 y2 − y4 A (6, 0) B (–2, 6)

1 2−3 5−k ∴ BC2 = AB2 + AC2


Area of the quadrilateral is =7 (–2 –k)2 + (6 – 4)2 = (–2 – 6)2 + (6 – 0)2 + (6 – k)2 + (0 – 4)2
2 − 6 −1 −4−3
4 + k2 + 4k + 4 = 64 + 36 + 36 + k2 – 12k + 16
−1 5 − k 4k + 8 = 152 – 12k
14 =
−7 −7 16k = 144 ⇒ k = 9

7 + 7(5 − k ) = 14 Alternate method:


Given A (6, 0), B (−2, 6), C (k, 4) form a right angled
42 − 7k = 14 triangle, right angled at A.
Slope of AB × slope of AC = –1
6−k = 2

Triumphant Institute of Management Education Pvt. Ltd. (T.I.M.E.) HO: 95B, 2nd Floor, Siddamsetty Complex, Secunderabad – 500 003.
Tel : 040–27898195 Fax : 040–27847334 email : info@time4education.com website : www.time4education.com SM1001920.Sol/33
6 4 24. The perpendicular distance from (x1, y1) to the line
× = −1
−8 k −6 ax1 + by1 + c
ax + by + c = 0 is
3 = k – 6 ⇒ k = 9. Ans: (9)
a2 + b2
15. The lines x = – 5 and y = 1 meet at (–5, 1) 3+ 4−k
i.e. in second quadrant. Choice (C) ∴ =1
32 + 42
16. The equation of a line which is parallel to x – 6 = 0 is in the
form x + k = 0 7−k
= 1 ⇒ 7−k = 5
It passes through (–5, 6) 5
∴ –5 + k = 0 7–k=±5
k=5 Let 7 – k = 5
Required equation is x + 5 = 0 Choice (B) Or 7 – k = – 5
k=2
17. Given two vertices of a triangle are (–5, 1) and (6, 2) k = 12
Let (x3, y3) be the third vertex and centroid is (1, 3) ∴ k = 2 or 12 Ans: (2, 12)
 −5 + 6 + x 3 1+ 2 + y 3 
∴  ,  = (1, 3)
 3 3  25. The distance between points (x1, y1) and (x2, y2) is
1+ x 3
= 1 and
3 + y3
=3
(x 2 − x1 )2 + (y 2 − y1 )2
3 3 Let the given points be A (3λ – 1, λ – 3) and
1 + x3 = 3 and 3 + y3 = 9 B (3λ – 5, λ + 1)
x3 = 2 and y3 = 6
Third vertex is (2, 6) Choice (C) (3λ − 5 − 3λ + 1)2 + (λ + 1− λ + 3)2 = 16 + 16 = 32 = 4 2
Choice (D)
18. Let A (2, 1) B (1, 4) and C (–2, 3)
AB = (1− 2)2 + (4 −1)2 = 10 26. Let A (0, 3) and B (0, –3)
In an equilateral triangle one of the vertices should have
BC = (− 2 −1)2 + (3 − 4)2 = 10
irrational coordinates.
From the options
CA = (− 2 − 2) + (3 −1)
2 2
= 20 (3 3 , 0) (0, 3 3 ) may be the third vertex but (0, 3 3 )
(BC)2 + (AB)2 = 10 + 10 = 20 = (CA)2 (0, 3) (0, –3) do not form a triangle
∴ The given vertices form a right angled isosceles triangle. ∴ third vertex is (3 3 , 0) Choice (A)
Choice (D)
27. Given slope of a line is – 1
19. Centroid of the triangle formed by the vertices i.e., tan θ = – 1
 x + x + x 3 y1 + y 2 + y 3  ⇒ θ = 135°.
(x1, y1 ), (x2, y2) and (x3, y3) is  1 2 , . The line is passing through the origin
 3 3 
∴ the line passes through second quadrant and fourth
 −5 −10 + 0 6 + 1−13  quadrant. Choice (B)
Centroid =  ,  = (–5, –2) Choice (B)
 3 3  28. The ratio in which the y-axis divides the line segment
joining the points (x1, y1) and (x2, y2) is −x1 : x2
20. Let y = mx + c – (1) The ratio that y-axis divides the line joining points
(1) passes through (1, 7) i.e., 7 = m + c – (2) (3, 6) and (9, 2) is −3 : 9
(1) passes through (–2, –8)
∴y-axis divides in the ratio 1 : 3 externally
∴ – 8 = – 2m + c
Choice (A)
2m – c = 8 – (3)
Solving (2) and (3), we get 29. The equation of a line which is parallel to 7x – 14y + 17 = 0 is
m = 5 and c = 2 7x – 14y + k = 0.
c – m = 2 – 5 = –3. Ans: (–3) It is passing through (3, –4)
7(3) −14(−4) + k = 0
21. The roots of the equation x2 – 8x + 15 = 0 are 3 and 5. k = −21−56 = −77
If m = 3 and c = 5, ∴ required line is 7x − 14y – 77 = 0
then required equation is y = 3x + 5. If m = 5 and c = 3, x – 2y – 11 = 0
then the required equation is y = 5x + 3 Choice (C) ∴ 2y = x – 11
22. Let A (1, 3), B (2, 3) and C (1, 5) 1  −11 
y= x +  
AB = 1 BC = 1+ 4 = 5 , CA = 2 2  2 
∴ BC2 = AB2 + CA2 −11
∴ y – intercept = Ans: (–5.5)
∆ ABC is a right angled triangle right angled at A. 2
For a right angled triangle circumcentre is the midpoint of 30. According to the problem, the given two lines are
the hypotenuse perpendicular to each other.
The vertices of the ends of the hypotenuse are (2, 3) and
−3
 2 +1 5 + 3  Slope of the first line m1 = and
(1, 5) Midpoint =  ,  5
 2 2 
−k
Slope of the second line m2 =
3  2
Circumcentre =  , 4  Choice (C)
2  Since m1 m2 = – 1
 −3   −k  3k
23. Given lines are not parallel.  5   2  = – 1; 10 = – 1
∴ The lines are intersecting each other   
∴ shortest distance between these two lines is zero. −10
Ans: (0) 3k = – 10 ⇒ k = Choice (D)
3
Triumphant Institute of Management Education Pvt. Ltd. (T.I.M.E.) HO: 95B, 2nd Floor, Siddamsetty Complex, Secunderabad – 500 003.
Tel : 040–27898195 Fax : 040–27847334 email : info@time4education.com website : www.time4education.com SM1001920.Sol/34
31. The equation of the line passing through the points 37. The vertex opposite to (2, 4) is
y −y (6 + 4 – 2, 6 + 8 – 4) = (8, 10) Choice (B)
(x1, y1) and (x2, y2) is y – y1 = 2 1 (x – x1)
x 2 − x1 38. The ratio in which x-axis divides the line joining (x1, y1) and
4+3 (x2, y2) is −y1/y2
Required equation is y + 3 = (x – 2) Required ratio = −8/16 = −1/2 = 1 : 2 externally
− 1− 2
Choice (B)
−7
y+3= (x – 2) 39. Let the required point be P.
3
 mx 2 − nx1 my 2 − ny 1 
3y + 9 = – 7x + 14 P =  , 
7x + 3y – 5 = 0 Choice (B)  m−n m − n 
 5(10 ) − 7(8) 5( −11) − 7( −9) 
32. Given line is 3 x–y+6=0 =  , 
 5−7 5−7 
− 3
Slope of the line = = 3  50 − 56 − 55 + 63 
−1 =  ,  = (−6/−2, 8/−2) = (3, −4)
 −2 −2 
∴ tan θ = tan 60°
Inclination of the line is Choice (D)
π 40. Since the straight line meets the positive x-axis and
θ = 60° = Choice (D)
3 negative y-axis at distances of 3 and 4 units respectively
from the origin, the equation of the line in intercept form is
33. Given line equation is 6x – 5y – 30 = 0 x y
6x – 5y = 30 + = 1 ⇒ 4x – 3y = 12 Choice (D)
3 −4
6x 5y
− =1 Exercise – 13
30 30
(Trigonometry)
x y
+ =1 Solutions for questions 1 to 40:
5 −6
x – intercept is 5 3
y – intercept is – 6 1. Given: cosθ = ; sinθ = 1 − cos2 θ = 1 − 9 / 25
5
∴ product of the intercepts = – 30 Ans: (–30)
5
∴cosecθ = Ans: (1.25)
4
34. Diagonal of the square = PR = ( 5 − 2) + ( 6 − 3 )
2 2

2. sin 60° . cos30° + cos60° .sin30°


= 18 units. = sin (60° + 30°) = sin90° = 1 Ans: (1)
1
area of a square = (diagonal)2 3. 4 4
4(sin 60° + cos 60°) – 3(cos 45° – sin 90°) 2 2

2
 4 4  
( )
2
3   1  
= 4   1  − 12 
1 2
Its area = 18 = 9 sq.units Ans: (9)  2  +  
2  − 3
2       

  2
 
35. Given lines are 15x + 8y – 5 = 0 – (1) and 9 1 1  5 3 8
30x + 16y – 7 = 0 – (2) =4  +  − 3  2 − 1 = 2 + 2 = 2 = 4
7  16 16   
(2) can be written as 15x + 8y – = 0 – (3) Ans: (4)
2
Distance between the parallel lines ax + by + c1 = 0 and 4. Given: 2 cos2A = sin30° + cos245°
2
c 2 − c1 1  1 
ax + by + c2 = 0 is i.e., 2cos2A = + 
a2 + b2 2  2 

 −7  2cos2A = 1
 + 5 
 2  1
Distance between the given lines is ⇒ cos A = ⇒ A = 45°
15 2 + 8 2 2
3
3
∴ cosec245° = ( 2) = 2 2
Ans: (2)
= 2 = Choice (A) 5. 5 sin θ + 2 cos θ = 3
2 2
17 34 A
⇒ 3 sin2θ + 2 sin2θ + 2cos2θ = 3
36. When x + 4y – 16 = 0 intersects the X-axis, y = 0. ⇒ 3 sin2θ + 2 = 3 3
∴ x + 4(0) – 16 = 0 i.e., x = 16. ⇒ 3 sin2θ = 3 – 2 = 1 1
1 θ
When it intersects the Y-axis, x = 0
⇒ sinθ = B C
0 + 4y – 16 = 0 i.e., y = 4 3
Y 2 2
 2
(0, 4) ∴ cot θ = 
2  =2 Ans: (2)
 1 
A  
4 π
(16, 0) 6. Given: r = 63 cm, θ = 60° =
X 3
O 16 B π
As l = rθ = 63 ×
1 1 3
Area of triangle AOB = (OA)(OB) = ( 4)(16)
2 2 22
= 21 × = 66 cm Ans: (66)
= 32 sq.units Ans: (32) 7
Triumphant Institute of Management Education Pvt. Ltd. (T.I.M.E.) HO: 95B, 2nd Floor, Siddamsetty Complex, Secunderabad – 500 003.
Tel : 040–27898195 Fax : 040–27847334 email : info@time4education.com website : www.time4education.com SM1001920.Sol/35
1 1 1
7. Given cos(x – y) = , sin y = = × cos45°. cos45°
2 2 2 2
x – y = 60°, y = 30° 1 1 1 1
∴ x – 30° = 60° ⇒ x = 90° = × × = Choice (D)
∴ the values are 30° and 90° Choice (B) 2 2 2 2 4 2

17. (1 – sinθ) (1 + sinθ) (1 + tan2θ)


8. Given: tan (A –B) = 2 – 3 = tan 15°
= (1 – sin2θ) (1 + tan2θ) = cos2θ . sec2θ = 1 Ans: (1)
⇒ A – B = 15°
3 1 1
Also, sinB = = sin 60° 18. +
2 2(1 + cos θ) 2(1 − cos θ)
π 5π c 1 1 1 
⇒ B = 60° ⇒ A = 75° = 75 × = 12 =  + 
180 2  1 + cos θ 1 − cos θ 
Choice (A)
1  1 − cos θ + 1 + cos θ 
=  
9. Given: sin(A + B) = 1 = sin 90° 2  (1 + cos θ)(1 − cos θ) 
⇒ A + B = 90°
1 2  1
Also cot (A – B) = 3 = cot 30° =   = = cosec2θ Choice (C)
2  sin 2 θ  sin2 θ
⇒ A – B = 30°
∴ A = 60°, B = 30°
19. Given: cosA = 1/2 and sec B = –5/3
∴ A = 60° Ans: (60)
2 5
59 4
10. Given tan θ + 2sec θ = 3
2 2

16 A B
59 1 3
tan2θ + 2(1 + tan2θ) = (∵ sec2θ = 1 + tan2θ) 3 3
16 ∴ sinA = and cot B =
2 4
9
tan2θ = 2
16  3
⇒ 4 sin2A + 8cotB = 4   + 8(3/4)
3  2 
tanθ = ± Choice (C)  
4
3
11. Let f(x) = 3(sinx – cosx)4 + 6(sinx + cosx)2 + 4(sin6x+ cos6x) = 4   + 6 = 9 Ans: (9)
Put x = 0 ⇒ sin x = 0, cosx = 1 4
f(x) = 3(0 – 1)4 + 6(0 + 1)2 + 4(0 + 1) 20. tan (960°) = tan (10 × 90° + 60)
= 3 + 6 + 4 = 13 Ans: (13)
= tan [2 × 90° + 60°] = tan 60° = 3 Choice (B)
12. Given: cosθ + secθ = 5
Squaring on both sides
cos2θ + sec2θ + 2cosθ . secθ = 25 1 − cos θ 1 − cos θ (1 + cos θ)
21. = ×
cos2θ + sec2θ + 2 = 25 1 + cos θ 1 + cos θ (1 + cos θ)
⇒ cos2θ + sec2θ = 23 Ans: (23)
13. Given: secθ – tanθ = p ––– (1)
1 − cos 2 θ sin 2 θ
= =
We know that sec2θ –tan2θ = 1 (1 + cos θ) 2
(1 + cos θ)2
⇒ (secθ + tanθ) (secθ – tanθ) = 1
⇒ (secθ + tanθ) p = 1 sin θ (1 − cos θ)
= ×
⇒ secθ + tanθ = 1/p ––– (2) (1 + cos θ) (1 − cos θ)
Adding (1) and (2)
sin θ(1 − cos θ) 1 − cos θ
1 p2 + 1 = =
sin θ
2 sec θ = p + = sin 2 θ
p p
1 cos θ
= −
p2 + 1 2p sin θ sin θ
secθ = ⇒ cosθ = Choice (B)
2p p2 + 1 = cosecθ – cotθ Choice (C)

1 1 5
14. sec2θ . cosec2θ = ⋅ 22. Given: sinθ =
cos 2 θ sin 2 θ 13
1 sin2 θ + cos2 θ
= =
cos 2 θ ⋅ sin 2 θ cos2 θ sin2 θ 13
1 1 5
= + = sec θ + cosec θ
2 2
Choice (B)
cos 2 θ sin 2 θ
θ
15. sin3θ . cos2θ can be either positive or negative depending 12
on the value of θ. Choice (D)
12 13 13 12
16. sin30° . cos45° . sin135° . cos315° ⇒ cosθ = ; secθ = ;cosecθ = ; cotθ =
13 12 5 5
1 1 13 cos θ − 6 sec θ
= ⋅ . sin (90° + 45°) . cos (360° – 45°) =
2 2 10 cos ecθ + 5 cot θ

Triumphant Institute of Management Education Pvt. Ltd. (T.I.M.E.) HO: 95B, 2nd Floor, Siddamsetty Complex, Secunderabad – 500 003.
Tel : 040–27898195 Fax : 040–27847334 email : info@time4education.com website : www.time4education.com SM1001920.Sol/36
13 × 12 / 13 − 6 × 13 / 12 2
= a  2b 
10 × 13 / 5 + 5 × 12 / 5 ⇒   = 1 + 2 
c  c 
11
= Choice (C) a2 4b
76 ⇒ = 1+
c2 c

23. Given cosα =


5 a2 c 2 + 4bc
13 ⇒ =
2
c c2
We know that sin2α + cos2α = 1 ⇒ a2 – c2 = 4bc Choice (C)
2
 5  144
sin α = 1 –   =
2
30. Given: x = sinθ + cosθ
 13  169
y = sinθ – cosθ
12 Squaring and adding
sinα = (α is acute) x2+y2 = (sinθ + cosθ)2 + (sinθ – cosθ)2
13
= 2(sin2θ + cos2θ) = 2
sin α 12 i.e., x2 + y2 = 2
tanα = = Choice (D)
cos α 5
 12  31. sin 30° + cos120°
2  = sin 30° + cos (90° + 30°)
2 tan α  5  = − 120
tan2α = = Choice (A) = sin 30°– sin 30° = 0
1 − tan2 α  12 
2 119 Ans: (0)
1−  
 5 
4
32. Given 1 – cos2θ + cos4θ – cos6θ + . . . .=
5
 cos ecα + cot α  cos ecα + cot α 
24. x =  cos ecα − cot α  cos ecα + cot α  . sin α 1
=
4
  
1+ cos2 θ 5
(cos ecα + cot α )2 5
= . sin α 1 + cos2θ =
cos ec 2α − cot 2 α 4
 cos ecα + cot α  cos2θ =
1
 sinα (∵ cosec α – cot α = 1)
2 2
= 
 1  4
= 1 + cosα Choice (B) 1
cosθ =
2
25. Minimum value of 5 sinx – 12 cosx + 14 θ = 60°. Choice (C)
= 14 – 5 2 + (12) 2 33. cosecθ = 1/2 ⇒ sinθ = 2
But sinθ ∈ [–1, 1]. Choice (D)
= 14 – 25 + 144 = 14 – 169 = 14 – 13 = 1
Ans: (1)
3 −1 3 +1
+
26. Since time taken is 3 times, angle turned will also be 3 cos 75° + sin 75° 2 2 2 2
34. =
times cos 75° − sin 75° 3 −1 3 +1
54 9π −
∴ Angle turned = 3 × 54° = 3 × ×π = 2 2 2 2
180 10
Choice (A) 2 3
= =– 3 Choice (C)
−2
27. Given: A + B = 45°
⇒ tan (A + B) = tan 45° = 1 3
35. Given: sin A = cos B =
tan A + tan B 2
=1
1 − tan A ⋅ tan B
⇒ tanA + tanB = 1 – tanA.tanB 3
sin A = ⇒ A = 60°
⇒ tanA + tanB + tanA tanB = 1 Ans: (1) 2
28. Given: sinθ – cosθ = 0 3
cos B = ⇒ B = 30°
⇒ sinθ = cosθ 2
⇒ tanθ = 1 and θ is in third quadrant π
⇒ θ = 225° 2A + B = 2(60°) + 30°=150°= 5 Choice (D)
∴ sinθ = sin (225°) 6
1 36.
= –sin45° = − A
2
2 2
∴ = = −2 2 Choice (B)
sin θ − 1 / 2 15 m

29. Given: sinθ and cosθ are the roots of the equation
θ
cx2 –ax + 2b = 0 B C
5 3m
a 2b
⇒ sinθ + cosθ = . and sinθ . cosθ = 15
c c tanθ = = 3 = tan60°
(sinθ + cosθ) = sin θ + cos θ + 2sinθcosθ
2 2 2 5 3
∴θ = 60° Choice (C)

Triumphant Institute of Management Education Pvt. Ltd. (T.I.M.E.) HO: 95B, 2 Floor, Siddamsetty Complex, Secunderabad – 500 003.
nd

Tel : 040–27898195 Fax : 040–27847334 email : info@time4education.com website : www.time4education.com SM1001920.Sol/37


37. cosα = 12/13 A x+y x−y
⇒ sinα = 5/13 4. Given: x ↓ y = and x ↑ y =
2 2
sinα = y/20
2 2
⇒ y/20 = 5/13 y
20 m  x+y  x−y 
y = 100/13 m (x ↓ y)2 – (x ↑ y)2 =   –  2 
 2   
α
C B 4 xy
x = = xy Choice (A)
4
Choice (C)
5. Given: (x y) = min (x, y)
(5 3) = min (5, 3) = 3 and
38. In ∆ABC tan45° = AC/BC A x ∆ y = max (x, y)
⇒ BC = 100 m 7 ∆ 5 = max (7, 5) = 7
In ∆ADC tan 60° = AC/CD 45° 60°
(5 3) (7 ∆ 5) = 3(7) = 21 Ans: (21)
100
CD = 6. 2 2
Given: x y = x – xy + y and x ÷ y = x + y
3 100 m (x y) (x ÷ y) = (x2 – xy + y2) (x + y) = x3 + y3
100 Choice (A)
BD = 100 + 45° 60°
3
B C D 7. Given: x ∆ y = x3 + y3 and x ∇ y = x2 – xy + y2
 3 + 1
= 100  m Choice (A) x∆y x3 + y3 ( )
(x + y ) x 2 − xy + y 2 = x + y
 3 
= =
x∇ y x − xy + y
2 2
x 2 − xy + y 2
Choice (C)
x
39. In ∆ABC, tan60° = A
x − 20 a+b
8. Given: a × b = and a → b = a2 – b2
3=
x a −b
x − 20 x+y 2
(x y) (x → y) = (x – y2) = (x + y)2 Choice (B)
3 x − 20 3 = x x−y
3 x − x = 20 3
9. Given: a b c = a2 + b2 + c2
20 3 a $ b $ c = ab + bc + ac
x=
( 3 − 1)
60° 45°
(x y z) + 2 (x $ y $ z)
B x − 20 m C D = x2 + y2 + z2 + 2 (xy + yz + xz) = (x + y + z)2
= 10 3 ( 3 + 1)
Choice (D)
x

= 10(3 + 3 ) m
10. Given: (a? b) = a + b + ab
Choice (C) (5? 3) ? 2 = (23? 2)
= 23 + 2 + 23(2) = 71 Ans: (71)
40.
Solutions for questions 11 and 12:
A
11. f(25, 22, 18) = max(25, 22, 18) = 25
g(38, 32, 36) = min(38, 32, 36) = 32
h(16, 18, 24) = L.C.M (16, 18, 24) = 144
I (g(38, 32, 36), h(16, 18, 24), f(26, 22, 18)
α = I (32, 144, 25)

B C
32 + 144 + 25 201
20 m = = = 67 Choice (D)
3 3
tanα = 4/3 = AB/20
⇒ AB/20 = 4/3 12. I (12, 6, 24) = Average (12, 6, 24)
cosα = 3/5 = BC/AC 12 + 6 + 24
= = 14
3/5 = 20/AC 3
AC = 100/3 f(12, 18, 16) = max (12, 18, 16) = 18
BA + AC = 80/3 + 100/3 = 60 m Choice (B) g(24, 18, 16) = min (24, 18, 16) = 16
h(f(12, 18, 16), I(12, 6, 24), g(24, 18, 16))
Exercise – 14 = h(18, 14, 16)
(Operator Based Questions) = L.C.M (18, 14, 16)
= 1008 Choice (B)
Solutions for questions 1 to 10:
Solutions for questions 13 and 14:
Given: a ∆ b = (a + b) (a – b)
1.
7 ∆ 5 = (7 + 5) (7 – 5) = 24 Ans: (24) 13.
(a$b)2 + (aψb)2 − 4(a ∧ b)
(aψb)2
a+b
2. Given: a $ b =
a −b
(a + b)2 + (a − b)2 − 4(ab)
5+3 (a − b)2
5$3=
5−3
=4 Choice (D)
=
( )
2 a2 + b2 − 4a b

3. Given: x y = x2 – xy + y2
(a − b)2
12 11 = 122 – 12 × 11 + 112 2(a − b )2
= =2 Ans: (2)
= 144 – 132 + 121 = 133 Ans: (133) (a − b)2
Triumphant Institute of Management Education Pvt. Ltd. (T.I.M.E.) HO: 95B, 2nd Floor, Siddamsetty Complex, Secunderabad – 500 003.
Tel : 040–27898195 Fax : 040–27847334 email : info@time4education.com website : www.time4education.com SM1001920.Sol/38
14.
((a∃b) + (aΕb))3 + ((a∃b) − (aΕb))3 + 24(a ∧ b)2 (a∃b) (n + 1) (2n + 1) = 630
from options n = 17 satisfies the above equation.
(a∃b)3 Ans: (17)
=
6. GM (9, 81, 36, 4)
(a 2
+ b2 + a2 − b2 ) + (a
3 2
+ b2 − a2 + b2 )
3
+ 24(ab )2 (a + b ) 1

(a 2
+b )
2 3 = (9 × 81 × 36 × 4) 4
1
(2a ) + (2b ) + 24a b (a + b )
2 3 2 3 2 2 2 2
= (9 × 9 × 9 × 9 × 4 × 4) 4

(a + b )
=
2 2 3 1
= (94 × 24)
8(a + b + 3a b (a + b ))
4 = 18 Ans: (18)
6 6 2 2 2 2

(a + b )
= 7. The geometric mean of 4, 42, 43, . . .., 49
2 2 3
9
4 . 4 2 . 4 3 ... 4 9
8(a + b )
2 2 3
9.10
(a + b )
= =8 Ans: (8)
2 3
41+ 2 + 3 + . . . +9
9
2
= 4 2.9 = 45 = 1024 Choice (A)

Solutions for question 15: Σx i2


Σx i
8. Given, = 289 and = 15
15. a * b = a2 + ab + b2 20 20
a∆b=a–b
2
(a × b) ÷ (a ∆ b) Σx i 2  Σ x i 
= (a × b) (a ∆ b) ∴ S.D = −  
= (a2 + ab + h2) (a – b)
n  n 
= a3 – b3 Choice (B) 2
Σx i 2  Σ x i 
= − 
20  20 
Exercise – 15
(Statistics)
Solutions for questions 1 to 20: = 289 − (15 )2
1. We know that the correct mean = 289 − 225
= wrong mean +
∑ correct observatio n − ∑ wrong observatio n = 64 = 8 Choice (B)
n 9. We know that
= 40 +
(25 + 36 ) − (23 + 42) (GM)2 = AM × HM
20 (16)2 = 32 × HM
4 16 × 16
= 40 – = 40 – 0.2 = 39.8 Choice (C) HM = =8 Ans: (8)
20 32

2. Let the two observations be a, b The AM of a, b, 8, 12, 15 10. QD. (standard result) Choice (C)
a + b + 8 + 12 + 15 11. The standard deviation of two positive numbers ‘a’ and ‘b’
is = 10 a + b + 35 = 50 ⇒ a + b
5 a −b
= 15 → (1) The variance is 11.6 is
2
(10 − a)2 + (10 − b)2 + (10 − 8)2 + (12 − 10)2 + (15 − 10 )2
= 11.6 Given, |a – b| = 32
5 32
(10 – a)2 + (10 – b)2 + 4 + 4 + 25 = 58 (10 – a)2+ (10 – b)2 ∴ S.D = = 16 Choice (A)
2
= 25 → (2)
Solving (1) and (2) we get a = 5; b = 10. ∴The two 12. Given series is 3, 7, x, y, 10, 10, 10
observations are 5, 10 Choice (C)
3 + 7 + x + y + 10 + 10 + 10
3. The given observations in the ascending order are 12, 14, Mean = = 8 ⇒ x + y = 16
7
15, 18, 20, 21, 23
As the series is non descending and all the terms are
∴ The middle value is 18 integers, the possibilities for x+ y = 16 are (7, 9), (8, 8)
∴ Median is 18. Ans: (18) ∴ y can be 9 or 8 Choice(C)
4. AM (33, 43, . . . . 203)
13. We know that the harmonic mean of x1, x2, . . . . ., xn is
33 + 43 + − − − + 203
= n
18 1 1 1
+ +−−− +
13 + 23 + 33 + 43 + .... + 20 3 − 1 3 − 23 x1 x 2 xn
=
18 5 3 10 9
∴ the harmonic mean of , , and is
=
(210 )2 − 9 = 9 × (4900 − 1) 3 2 9 4
18 18 4
=
3 2 9 4
=
4899
= 2449.5 Choice (B) + + +
5 3 10 9
2
4
5. The mean of squares of first ‘n’ natural numbers is =
54 + 60 + 81 + 40
(n + 1)(2n + 1) 90
6 4× 90 72
(n + 1)(2n + 1) = 105
=
235
=
47
Choice (A)

Triumphant Institute of Management Education Pvt. Ltd. (T.I.M.E.) HO: 95B, 2nd Floor, Siddamsetty Complex, Secunderabad – 500 003.
Tel : 040–27898195 Fax : 040–27847334 email : info@time4education.com website : www.time4education.com SM1001920.Sol/39
14. We know that, below 100 there are 25 prime numbers. y : 7, 5, 3, 1
∴ the 26th prime number is 101 and first prime number is 2 ∴ The number of integral solutions of the given equation is 4.
∴ the required range = 101 – 2 = 99 Ans: (99) Ans: (4)
15. The maximum value = 158 2. Given: 3x + 4y = 56
The range = 48 56 − 3 x
∴minimum value = maximum value – range. y= ------- (1)
4
= 158 – 48 = 110 Ans: (110)
The positive integral values of x, y that satisfy the above
16. The variance of the given series is 35 equation are listed below.
x : 4, 8, 12, 16 from (1)
∴ The standard deviation of the given series is 35 y : 11, 8, 5, 2
35 Of the above (x, y) = (4, 11) satisfies the condition x < y
Hence the standard deviation of x1, x2, x3, - - - - xn is The required value of y = 11 Choice (B)
5
Now, the standard deviation of 3x1, 3x2, 3x3, - - - 3xn 82 + 3 y
3. Given: 4x – 3y = 82 ⇒ x =
35 4
= 3× Clearly y has to be even. When y = 2, then x = 22
5
Ans: (22)
9 63
∴ Variance of 3x1, 3x2, . . . . ., 3xn is × 35 = 75 + 3 y
25 5 4. Given: 5x – 3y = 75 ⇒ x =
Choice (A) 5
As y < 0, one of the values of y is –5. The possible values
17. If one of the observations is 0, then the product of all of y and the corresponding values of x are listed below
observations will be 0
y: –5 –10 – 15 – 20
∴ The GM of the observations is ‘0’ Choice (B)
x: 12 9 6 3
18. The arithmetic mean of 8, 6, 3, 4, 9, 12 is The number of possible solutions is 4. Choice (D)

8 + 6 + 3 + 4 + 9 + 12 42 5. Given: 21x + 9y = 120 or 7x + 3y = 40 ------ (1)


= =7 x > 0, y > 0 from (1) y < 14 and x < 6
6 6
40 − 7 x
Σ xi − M y=
Mean deviation = 3
n x : 1, 4
y : 11, 4 are the only possible solutions. As x ≠ y, (1, 11)
8−7 + 6−7 + 3−7 alone is the solution Choice (B)

4 − 7 + 9 − 7 + 12 − 7 88 − 7 y
= 6. Given: 12x + 7y = 88 ⇒ x =
6 12
1+ 1 + 4 + 3 + 2 + 5 y = 4, x = 5 is the only positive integral solution.
= Ans: (1)
6
16 7. Let the number of pens with Tinku be x and with Rinku be y.
= =8/3 Choice (D) 21x + 17y = 324 -------- (1)
6
Dividing both sides by 17 (least coefficient) we have
19. S.D. (x, x, x, x, x, x, x + 7) = S.D (0, 0, 0, 0, 0, 0, 7)  4x   324 
Remainder =   = R  17  = 1 → (1)
(By subtracting ‘x’ from each observation)
 17   
 
()
2 By trail, we notice x = 13 satisfies (1). Also, it is clear that
0 + 0 + 0 + 0 + 0 + 0 + 72  7  
2
∑ xi 2 
= −   Q S.D = − x  x < 16 and y < 20
7 7  n 
  ∴ x = 13 and y = 3 is the only solution. Choice (B)
= 7 −1 = 6 8. Let the number of type Ι and ΙΙ books purchased be x and y
∴ variance = (S.D)2 = 6 Ans: (6) respectively
29 − 5 x
20. The series 1, 2, 3, . . . . . , n is an 5x + 7y = 29 ⇒ y =
7
A.P. with d = 1.
By trial, we notice x = 3, y = 2 is the only solution.
n2 − 1 ∴ The number of type Ι books purchased is 3.
∴S.D. = Choice (A)
12 Choice (C)

Exercise – 16 9. Let the number of type Ι and type ΙΙ balloons sold be x and
(Special Equations) y respectively
x > 0, y > 0 and 5x + 3y = 120
Solutions for questions 1 to 15: 120 − 5 x
⇒y=
1. Given: 2x + 3y = 21 3
21− 2x By trial, x = 3 y = 35
y= ------- (1) The remaining solutions of x, y are listed below
3
As x and y are integers, x = 0, y = 7 is one set of solutions. x: 3 6 9 12 15
As the denominator on the RHS of (1) is 3, the values of x y: 35 30 25 20 15
that satisfy (1) form an AP with common difference 3. The
possible values of x and the corresponding values of y are As he sold an equal number of type Ι and type ΙΙ balloons,
listed below. x = 15 and y = 15
x : 0, 3, 6, 9 ∴ Total he sold 15 + 15 = 30 balloons

Triumphant Institute of Management Education Pvt. Ltd. (T.I.M.E.) HO: 95B, 2nd Floor, Siddamsetty Complex, Secunderabad – 500 003.
Tel : 040–27898195 Fax : 040–27847334 email : info@time4education.com website : www.time4education.com SM1001920.Sol/40
Alternate method: Exercise – 17
From (1) as x = y (Quadratic Equations)
⇒ 8x = 120 ⇒ x = 15
Solutions for questions 1 to 40:
∴ x = 15, y = 15
1. x2 + 11x + 30 = 0
The total number of balloons sold = x + y = 30
⇒ x2 + 6x + 5x + 30 = 0
Ans: (30)
⇒ x (x + 6) + 5 (x + 6) = 0
10. Let the number of marbles Alok and Ashok had be x and y ⇒ (x + 6) (x + 5) = 0
respectively ⇒ x = −6 or −5. Choice (B)
5x – 3y = 22 2
2. Given y1 and y2 are the roots of y – 7y + k = 0
22 + 3 y
x=  −7 
5 y1 + y2 = –   = 7 ---- (1)
 1 
By trial, when y = 1, x = 5. Possible values of x, y are listed
below given
y : 1, 6, 11, 16, 21, …….. y1 – y2 = 3 ----- (2)
x : 5, 8, 11, 14, 17, ……. the product of roots = solving (1) and (2), y1 = 5 and y2 = 2
As x + y = 30 x = 14 and y = 16 y1y2 = k
∴ k = 10. Ans: (10)
∴ The number of marbles Alok had is 14 Choice (D)
3. Let the other root be α
11. Given: 4x + 9y = 72 −(−8)
4+ 5 +α= =8
72 − 4 x 1
y=
9 α= 4− 5
By trial and error x = 0, y = 8 is one of the solutions. The
other solutions are x = 9, y = 9 and x = 18, y = 0 The other root is 4 – 5 . The product of the roots = m
∴ The possible non negative solutions are 3.
Choice (A)
( )(
m = 4 + 5 4 − 5 = 42 − ) ( 5) 2

= 16 – 5 = 11. Ans: (11)


12. Let the number of scales, pencils and erasers purchased 4. Let the roots of the equation y2 + ky + 32 = 0 be p and 2p
be s + 2, p + 2, e + 2 respectively p + 2p = – k and p. 2p = 32
∴ 15s + 5p + 1.5e = 84 – 43 = 41 3p = – k and 2p2 = 32
2p2 = 32 ⇒ p2 = 16 ⇒ p = ± 4
∴ 30s + 10p + 3e = 82.
Since 3p = – k ⇒ k = – 3p ⇒ k = ± 12. Choice (D)
If p = 8, (s, e) has no possible value.
If p=7, 10s + e = 24 5. Given that α and β are the roots of x2 – 8x + 9 = 0.
(s, e) can be (2, 4) α + β = 8 and α.β = 9
∴ p can be 7. α β α 2 + β2 (α + β) 2 − 2αβ 8 2 − 2(9)
+ = = =
Maximum value of p is 7. Maximum number of pencils that β α αβ αβ 9
Ramesh could have purchased is (max (p + 2) i.e) 9.
Ans: (9) 46
= Choice (C)
9
13. A + B + C = 20 6. A quadratic equation in x whose sum of the roots is s and
20000A + 15000B + 10000C = 3,00,000 product of the roots is p is x2 – sx + p = 0
Since, each department has at least 5 employees, The equation whose sum of the roots is –4 and the product
A – 5 ≥ 0, B – 5 ≥ 0 and C – 5 ≥ 0 of the roots is 9 is x2 – (–4)x + 9 = 0
∴ The maximum number of employees in B is 10. ⇒ x2 + 4x + 9 = 0. Choice (D)
Choice (D) 7. Let the roots of the equation py2 + qy + r = 0 be mk and nk
−q r
14. Let the number of books of Maths, Physics and Chemistry mk + nk = and (mk) (nk) =
p p
purchased be x, y and z respectively
Given 250x + 200y + 225z = 4500 ------ (1) −q r
m+n= and mn =
He purchased minimum of 6 books in each subject kp pk 2
250(x – 6) + 200(y – 6) + 225(z – 6) = 4500 – 4050 = 450
With `450 he can purchase, two more Chemistry books or r
mn pk 2  r   k 2p 2 
1 Maths book and 1 Physics book.
= =     = pr .
The total number of books he purchased = 6 + 6 + 8 = 20 (m + n)2 2  pk 2   q2  q2
 −q    
or 7 + 7 + 6 = 20. In either case, total number of books he  
purchased is 20. Choice (C)  kp 
Choice (B)
15. Given: x + y + z = 11 ---- (1) and
2x + 3y + 5z = 36 ----- (2) 8. Let the roots of py2 + qy + r = 0 be 3k and 4k
2x + 3y + 5(11 – x – y) = 36 (eliminating z using (1) & (2)) −q r
∴3k + 4k = and (3k) (4k) =
3x + 2y = 19 p p
19 − 3 x −q r
y= ∴ 7k = and 12k2 = ------- (1)
2 p p
By trial, when x = 1, y = 8 2
Possible values of x, y, z are −q  −q r
k= substitute the k value in (1) = 12   =
x : 1, 3, 5 7p  7p  p
 
y : 8, 5, 2
z : 2, 3, 4 12q2 r
Given y > 5 x = 1, y = 8 and z = 2 is the only possible =
49p 2 p
solution.
x+z=1+2=3 Ans: (3) 12q2 = 49pr. Choice (A)

Triumphant Institute of Management Education Pvt. Ltd. (T.I.M.E.) HO: 95B, 2 Floor, Siddamsetty Complex, Secunderabad – 500 003.
nd

Tel : 040–27898195 Fax : 040–27847334 email : info@time4education.com website : www.time4education.com SM1001920.Sol/41


9. Given α and β are the roots of the equation px2 + qx + r = 0 14. Let the equation be py2 + qy + r = 0
−q r −q r
α+β= and αβ = Sum of its roots = and product of its roots =
p p p p
−q According to the problem
2
1 1 β+α p −q  −q r
+ = = =   = 4
α β αβ r r  p  p
 
p
q2 r
1 1 1 p = 4 ⇒ q2 = 4pr
. = = p2 p
α β αβ r
q2 – 4pr = 0
1 1  1 1 ∴ Discriminant = 0. Ans: (0)
The equation whose roots are , is x2 –  + 
α β α β
15. Given : α and β are the roots of x2 + bx + c = 0 ⇒ α + β = −b
1
x+ =0 and αβ = c
αβ αβ + α + β + α + β − αβ = 2 (α + β) = –2b
 −q  p (α + β + αβ) (α + β − αβ) = (α + β)2 – (αβ)2
x2 –   x + = 0 = b2 – c2
 r  r
∴ The quadratic equation whose roots are αβ + α + β and
⇒ rx2 + qx + p = 0 α + β – αβ is
x2 – (sum of the roots)x + (product of the roots) = 0
Alternate method: ⇒ x2 + 2bx + b2 − c2 = 0. Choice (C)
The quadratic equation in x whose roots are reciprocals of
the roots of a given quadratic equation in x can be obtained 16. The quadratic equation whose roots are twice the roots of a
1 x
by replacing x by in given equation. given equation is obtained by substituting in place of x.
x 2
2 2
 1  1 x x
∴ Required equation = p  ∴ The required equation is   –9   + 18 = 0
 + q  x  + r = 0 i.e. 2 2
 
x  
⇒ x2 – 18x + 72 = 0 Choice (B)
p q p + qx + rx2
2
+ + r = 0 i.e. =0
x x x2 17. The roots are [l + (m − n)] and [l − (m − n)]
rx2 + qx + p = 0. Choice (D) x2 − [l + m − n + l − m + n]x + [l + m + n] [l − (m − n)] = 0
⇒ x2 − 2lx + l2 − (m − n)2 = 0 Choice (C)
4ac − b 2
10. Maximum value of ax2 + bx + c is 18. Given that the sum of the roots of 3x2 – bx – 4 = 0 is 1.
4a
b
4( −11)(12) − 22 2 ∴ =1
∴ Maximum value of 12 + 22x – 11x2 = 3
4( −11) ⇒ b = 3. Ans: (3)
−44(12 + 11)
= = 12 + 11 = 23. Ans: (23)
− 44 19. If the roots of a quadratic equation are the reciprocals of
each other, its constant term must be equal to the
11. We know that equation if ax2 + bx + c = 0 have equal roots coefficient of x2. For this reason, as the roots of the
then b2 – 4ac = 0 equation ax2 + bx + c = 0 are reciprocals of each other,
given x2 + 2(m + 3) x + 81 = 0 has equal roots a = c must hold true. Choice (B)
∴ Its discriminant = [2(m + 3)]2 – 4(1) (81) = 0
[2 (m + 3)]2 = (4) (81) ⇒ (m + 3)2 = 81 20. For any quadratic equation, if the sum of its roots is zero,
then coefficient of x must be 0. For this reason, the sum of
⇒m+3=±9
the roots of ax2 + bx + c = 0 being zero implies that b = 0.
⇒ m = 6 or –12. Choice (C)
Choice (B)

12. In the equation (r – q) y2 + (p – r) y + (q – p) = 0 21. Let the roots of the equation be k and 3k.
Sum of the coefficients = r – q + p – r + q – p = 0 −b
∴ 1 is a root of the equation The sum of its roots = = 4k
Since the equation has equal roots the other root is also 1 a
∴ Product of the roots = 1 c
The product of its roots = = 3k2
( q − p) a
∴ = 1⇒ 2q = p + r
(r − q) The ratio of the square of the sum of its roots and the
Choice (C) 2
b
 
13. Let the roots of the equation be α and β. The arithmetic product of its roots =   =
a (4k )2
α+β c 3k 2
mean of α and β = and a
2
Given geometric mean α and β = αβ b2 16
=
ac 3
α+β
= 10 and αβ = 8 ⇒ 3b2 = 16ac Choice (D)
2
∴ α + β = 20 and αβ = 64 22. As ac is positive, a and c have the same sign. The
The quadratic equation whose roots are α and β is discriminate of the equation is b2 – 4ac = ac – 4ac = –3ac.
x2 – (α + β) x + αβ = 0 ⇒ x2 – 20x + 64 = 0. As ac is positive, the discriminate is negative.
Choice (A) ∴ The roots are complex. Choice (A)

Triumphant Institute of Management Education Pvt. Ltd. (T.I.M.E.) HO: 95B, 2nd Floor, Siddamsetty Complex, Secunderabad – 500 003.
Tel : 040–27898195 Fax : 040–27847334 email : info@time4education.com website : www.time4education.com SM1001920.Sol/42
23. K (K + 1)(K + 2)(K + 3) = 120 α10 – (β – 1)10 → (3)
K (K + 3)(K + 1)(K + 2) = 120 From (1), β = 1 – α.
(K2 + 3K)(K2 + 3K + 2) = 120 ------(1) Substituting β = 1 – α in (3), we get
Let K2 + 3K = x α10 – β20 = α10 – (1 – α – 1)10
x2 + 2x = 120 = α10 – (–α)10
⇒ x2 + 2x – 120 = 0 = α10 – α10 = 0
(x + 12)(x – 10) = 0 ∴The value of α10 – β20 = 0.
x + 12 = 0 or x – 10 = 0
x = – 12 or x = 10 Alternative solution 2:
As K2 + 3K is positive, x is positive. Given that α and β are the roots of x2 – x + 1 =0
∴ x = 10 Ans: (10) Consider, (x + 1) (x2 – x + 1) = 0 ⇒ x3 + 1 =0 → (1).
The roots of (1) are –1, α and β.
24. Let the cost of each apple be a. Therefore α and β satisfying (1).
160 ⇒ α3 = –1 and β3 = –1
Then the number of apples bought for =
a α10 – β20 = (α3)3 (α) – (β3)6 (β2) = –α – β2
If the cost of each apple was `2 less, i.e., ` (a − 2) (As x2 – x + 1 =0, α + β = 1 and β2 – β + 1 = 0 ⇒ α + β = 1
and β2 = β – 1)
160 ∴–α – β2 = –α – β + 1
Amar would have bought apples.
a−2 = –(α + β ) + 1 = –1 + 1 = 0
∴ α10 – β20 = 0. Ans: (0)
160 160
Given, − =4
a−2 a 11 13
27. + =2
x − 13 x − 11
a − a + 2
⇒ 60  =4 11(x – 11) + 13 (x – 13) = 2 (x – 13) (x – 11)
 a − 2a 
2
x2 – 36x + 288 = 0
a2 – 2a – 80 = 0 (x – 24) (x – 12) = 0
⇒ (a – 10) (a + 8) = 0 ⇒ x = 24 or x = 12
a = 10 or – 8 a b
Note: Whenever an equation is of the form +
As a ≠ – 8, a = 10. Ans: (10) x −b x −a
25. Let us say each boy received b chocolates. Each girl a+b
= 2, its roots will be a + b and . Choice (D)
received b2 chocolates. 2
360 = 90 b + 45 b2
45 (b2 +2b – 8) = 0 28. (x + 2)2 = x2 +4x + 4
⇒ (b + 4) (b – 2) = 0 (x – 2)2 = x2 – 4x + 4
b – 4 = 0 or b – 2 = 0 ∴(x + 2)2 + (x – 2)2 = 2x2 + 8 = 40
⇒ b = – 4 or 2 x2 = 16 ⇒ x = ± 4 Choice (A)
As b is positive, b = 2. Ans: (2)
29. If any root of a quadratic equation is in the form a – b , the
26. x2 – x + 1 = 0
other root must be in the form a + b . For this reason, as
1± 1− 4 1± 3 i
x= = 3 – 3 is a root of the given quadratic
2 2
1+ 3 i equation, the other root must be 3 + 3 . The sum of the
1− 3 i
α= ;β = roots = p = 6
2
2
2
 1− 3 i  1+ 3 i 2 − 2 3 i
The product of the roots = q = (3)2 – ( 3) 2
=6
β2 =   = ∴ p + q = 12 Ans: (12)
 2  4
  30. The maximum/minimum value of a quadratic expression
1− 3 − 2 3 i 4ac − b2
= ( Q i2 = –1) ax2 + bx + c is given by
4 4a
− 2 − 2 3 i − (1+ 3 i) If a > 0, the expression has its minimum value. If a < 0, the
= = expression has its maximum value.
4 2 a = 10, b = 8 and c = 5.
β2 = – α a > 0 it will have a minimum value. Its minimum value is
(β2)10 = (– α)10 = β20 = α10
4(10 )(5 ) − 82 17
⇒ α10 – β20 = 0.
4(10 )
given by = Choice (A)
5
Alternative solution 1:
31. As the roots are real and equal, the discriminate equals 0
Given,
∴ (3k – 12)2 – 4(k – 4) (9) =0
α, β are the roots of the equation x2 – x + 1 =0
9(k – 4) [(k – 4) – 4] = 0
−b ⇒ 9 (k – 4) (k – 8) = 0
Sum of the roots =
a k – 4 = 0 or k – 8 =0
⇒ α + β = 1 → (1) If k – 4 = 0, the equation will not be quadratic.
c ∴ k – 4 ≠ 0. ∴ k – 8 = 0
Product of the roots = ∴k=8 Ans: (8)
a
⇒ αβ = 1 → (2) 32. The required equation can be obtained by replacing x by
Q β is a root of the equation x2 – x + 1 =0
1
β2 – β + 1 = 0 in the given equation. We get 55x2 – 21x + 2 = 0.
x
⇒ β2 = β – 1
Alternate way of obtaining the equation is simply swapping
α10 – β20 = α10 – (β2)10 the coefficient of x2 and the constant term. Choice (C)

Triumphant Institute of Management Education Pvt. Ltd. (T.I.M.E.) HO: 95B, 2nd Floor, Siddamsetty Complex, Secunderabad – 500 003.
Tel : 040–27898195 Fax : 040–27847334 email : info@time4education.com website : www.time4education.com SM1001920.Sol/43
33. The equation each of whose roots are k times the roots of (c) α3 + β3 = (α + β)3 − 3αβ(α + β) = 8 − 3 × 1 × 2 = 2
the equation ax2 + bx + c = 0 is given by ax2 + kbx + k2 c = 0 Ans: (2)
As K = 3, the required equation is
2x2 + 3(–11)x + 9(12) = 0 1 1 α β
(d) + + +
i.e., 2x2 – 33x + 108 = 0 Choice (C) α2 β2 β α
34. The required equation can be obtained by replacing x by α 2 + β 2 + α 3 β + αβ 3
x – 1 in the given equation as x2 – 15 x + 35 = 0. =
α 2β 2
Choice (D)
α + β + αβ( α 2 + β2 )
2 2
2 + 1× 2
35. (a) x2 − 3x + 2 = 0 = = =4 Ans: (4)
x2 − 2x − x + 2 = 0 ( αβ)2 1
x(x − 2) − 1(x − 2) = 0
1 1 α β
(x − 1) (x − 2) =0 (e) + + +
x = 1, 2 Choice (C) α3 β3 β2 α2
(b) x2 + 3x + 2 = 0 α 3 + β 3 + α 4 β + αβ 4
x2 + 2x + x + 2 = 0 =
(αβ) 3
x(x + 2) + 1(x+ 2) = 0
(x +1) (x + 2) =0 α + β + αβ(α 3 + β 3 )
3 3
2+1× 2
= = =4
x = −1, −2 Choice (D) (αβ) 3 1
(c) x2 − 2x − 3 = 0
x2 − 3x + x − 3 = 0 Ans: (4)
x(x − 3) + 1(x − 3) = 0
(x − 3) (x + 1) =0 40. Let the roots of the Q.E be α, 2α
x = 3, −1 Choice (B) 45 9
Sum = 3α = =
(d) x2 + 2x − 3 = 0 25 5
x2 + 3x − x − 3 = 0
3 6
(x + 3) (x − 1) =0 ⇒ α = , 2α =
x = −3, 1 Choice (C) 5 5

( ) ( )( )
3 6 k
Product = × = ⇒ k = 18 Ans: (18)
36. (a) Q.E. = x 2 − 3 + 2 2 + 3 − 2 2 x + 3 + 2 2 3 − 2 2 = 0 5 5 25
⇒ x − 6x + 1 = 0
2
Choice (C)
Exercise – 18
(b) x2 − [ (a + b ) + (a − b )] x + (a + b )(a − b ) = 0 (Inequalities and Modulus)
⇒ x2 − 2ax + a2 − b = 0 Solutions for questions 1 to 40:
⇒ x2 − 2ax + a2 − b = 0 Choice (D)
1. When x>1/2, |2x-1| = (2x-1)
37. (a) b2 − 4ac = (−3)2 − 4 × 4 × (−1) = 25 Then |2x – 1| > 3 – x becomes
Greater than 0 and a perfect square. 2x – 1 > 3 – x ⇒ 3x > 4
Roots are rational and unequal. Choice (B) ⇒ x > 4/3.
(b) b − 4ac = (−11) − 4 × 4 × 5 = 41 > 0 and not a perfect
2 2 When x < 1/2, |2x – 1| = – (2x – 1),
square. Then |2x – 1| > 3 – x becomes
∴ roots are irrational. Choice (C) ⇒ – (2x – 1) > 3 – x ⇒ 1 – 2x > 3 – x
⇒ 1 – 3 > 2x – x ⇒ –2 > x ⇒ x < –2.
(c) b2 − 4ac = (4)2 − 4 × 1 × 4 = 0 Since x > 0, x > 4/3 Choice (B)
∴ roots are real and equal Choice (A)
2. |8x − 5| = 3
2 ⇒ 8x − 5 = 3 or 8x − 5 = −3
38. (a) 5x2 − 2x + =0
3 8x = 8 or 8x = 2
( −2) 2 2 1
Sum = − = ⇒ x = 1 or x = =
5 5 8 4
2 1 
2 ∴ Solution set =  , 1 Choice (C)
Product = 3 = Choice (D) 4 
5 15
3. |2x − 3| < 1
22
(b) Sum = = 2 ⇒ −1 < 2x − 3 < 1
11 2 ⇒1<x<2 Choice (D)
5 2 5 3x − 6
Product = = Choice (C)
11 2 11 4. ≤ 12
2

p 2 − q2 |x−2|
(c) Sum = =p − q ≤4
p+q 2
|x – 2| ≤ 8
1 1
+ –8 ≤ x – 2 ≤ 8
p q (p + q) 1 –6 ≤ x ≤ 10
Product = = = Choice (B) Choice (D)
p + q pq(p + q) pq
x+4
5. ≥0
x−3
39. α + β = 2, αβ = 1
Multiplying both the numerator and the denominator by
1 1 α+β 2
(a) + = = =2 Ans: (2) x − 3,
α β αβ 1 ( x − 3) ( x + 4)
≥ 0 ⇒ (x − 3) (x + 4) ≥ 0
(b) α2 + β2 = (α + β)2 − 2αβ = 4 − 2 = 2 Ans: (2) ( x − 3 )2

Triumphant Institute of Management Education Pvt. Ltd. (T.I.M.E.) HO: 95B, 2nd Floor, Siddamsetty Complex, Secunderabad – 500 003.
Tel : 040–27898195 Fax : 040–27847334 email : info@time4education.com website : www.time4education.com SM1001920.Sol/44
(∵ (x – 3)2 is always positive) x = −5 or x = 4
The critical points are –4 and 3. −1
∵x> ⇒ x = 4 only satisfy the given equation.
3
 x 
−∞ ∞ −1
−4 3 Case 2: When x < , |3x + 1| = −(3x + 1)
3
When x ∈ (−4, 3) the given inequation is not satisfied and ⇒ |x + 3x + 1| = 9
at x = 3, the function is not defined ⇒ |4x + 1| = 9 ⇒ 4x + 1 = 9 or 4x + 1 = −9
The values of x for which the inequation is not satisfied are ⇒ 4x = 8 or 4x = − 10
−4 < x ≤ 3. Choice (A) −10 −5
x = 2 or x = =
6. x2 − 5x + 6 < 0 4 2
⇒ (x − 2) (x − 3) < 0 −1 −5
Since x < ⇒ x= only satisfy the given condition.
x  x 3 2
−∞ ∞ ∴ The total number of distinct solutions are two.
2 3 Ans: (2)
The critical points are 2, 3. When x = 0, the inequation is
not satisfied. 12. Since for all values of x,
⇒ when x ∈ (2, 3) the inequation is satisfied. |x + 2| and |x − 2| both are positive
∴ Solution set is 2 < x < 3 Choice (C) ∴ |x + 2| + |x − 2| ≠ 0 for all values of x.
∴ There is no solution Choice (D)
7. y |y − 4| = 12
When y ≥ 4, y – 4 is positive ∴ |y – 4| = y – 4 13. Given x |x| = 9.
y (y – 4) = 12 When x > 0, |x| = x.
y2 – 4y – 12 = 0 x(x) = 9 ⇒ x2 = 9
y = 6 or –2 but y ≥ 4; so y = 6. ⇒ x = ± 3.
When y < 4; y –4 is negative Since x ≥ 0; ∴ x = 3.
Then |y – 4| = -(y – 4). When x < 0, |x| = –x
–y(y – 4) = 12 x(–x) = 9 ⇒ x2 = – 9
y2 – 4y + 12 = 0. Which is not possible.
There is no value of y that satisfies the above equation. ∴ solution set = {3}. Ans: (3)
The solution is {6}. Choice (A)
14. Given 3x − 4 > 11 − 2x
8. Given x2 − 15x < − 54 ⇒ 3x + 2x > 11 + 4 ⇒ 5x > 15
⇒ x2 − 15x + 54 < 0 ⇒ (x − 9) (x − 6) < 0 ⇒ x ∈ (6, 9) the
15
above inequation is true. Choice (B) ⇒ x> ⇒x>3 Choice (B)
5
x+5 x+5
9. Given ≥ −2 ⇒ +2≥0 9x 2 − 9x + 2
x−4 x−4 15. Given <0
x + 5 + 2x − 8 2x 2 + 1
⇒ ≥0 2
Since 2x + 1 is always positive
x−4
9x2 − 9x + 2 < 0
3x − 3 ( x − 1) ⇒ 9x2 − 6x − 3x + 2 < 0
≥0 ⇒ 3 ≥0
x−4 x−4 ⇒ 3x (3x − 2) − (3x − 2) < 0
( x − 1)( x − 4) ⇒ (3x − 1) (3x − 2) < 0
≥0
( x − 4 )2 1 2
The critical points are  , .
(x – 1) (x – 4) ≥ 0. 3 3
Critical points are 1 and 4 x  x
−∞ ∞
 x 
−∞ ∞ 1/3 2/3
1 4
When x = 0, the inequation is not satisfied.
When x = 0 the inequation is true. 1 2
When x ≤ 1 or x > 4 the inequation is true. ⇒ x ∈ , 
3 3
∴ When x = 2, 3 and 4 the inequation is not true.
Choice (B) 1 2
∴ Solution set =  ,  Choice (D)
3 3
10. Given x2 − 13 |x| + 42 = 0
When x > 0, |x| = x
⇒ x2 − 13x + 42 = 0 16. Given x − 9 < 10 and |x − 4| > 10
⇒ (x − 6) (x − 7) = 0 ⇒ x = 6 or 7 ⇒ x − 9 < 10 and x − 4 > 10 or x – 4 < –10
When x < 0, |x| = −x ⇒ x < 19 --------- (1)
x2 + 13x + 42 = 0 and x > 14 or x < − 10 + 4
(x + 6) (x + 7) = 0 x > 14 or x < −6 --------- (2)
x = − 6 or −7 From (1) and (2) we get x < −6 or x ∈(14, 19).
∴ The total number of distinct solutions is 4. ∴ The solution set is x ∈ (−∞, −6) ∪ (14, 19).
Ans: (4) Choice (A)

11. Given |x − |3x + 1| = 9 17. Given |x|2 + 2 |x| + 1 = 0


−1 Since |x| and |x|2 are always positive
Case 1: When x > , |3x + 1| = 3x + 1. ⇒ |x|2 + 2|x| + 1 is never equal to zero.
3
∴ There is no solution. Choice (C)
⇒ |x − 3x − 1| = 9 ⇒ |−2x − 1| = 9
⇒ −2x − 1 = 9 or −2x − 1 = −9 ⇒ 2x = − 10 or 2x = 8
Triumphant Institute of Management Education Pvt. Ltd. (T.I.M.E.) HO: 95B, 2nd Floor, Siddamsetty Complex, Secunderabad – 500 003.
Tel : 040–27898195 Fax : 040–27847334 email : info@time4education.com website : www.time4education.com SM1001920.Sol/45
1 x+6
≥0 ;
(x + 6)(x − 4) ≥ 0
18. Given < 0 . When both the numerator and 25. Given
x2 − 9 x−4 (x − 4)2
denominator are multiplied by x2 – 9, we have ⇒ (x + 6) (x − 4) ≥ 0 [∵ (x – 4)2 ≥ 0]
x2 − 9 The critical points are −6 and 4
<0
(x 2 − 9)2 
 
As (x2 − 9)2 is always positive, we have
x2 − 9 < 0 i.e (x − 3) (x + 3) < 0
The critical points are −3 and 3.
Only when x ∈ (−3, 3), the above equation is satisfied. –6 4
∴ The solution set = (−3, 3). Choice (B)
When x = 0, the inequality is not satisfied.
∴ When x ∈ (−6, 4], the inequation is not satisfied.
19. Since |x − 10| is always positive, maximum value of 25 − |x − 10|
Choice (C)
is 25. Ans: (25)
26. x2 – 7x + 12 > 0 ⇒ (x – 4) (x – 3) > 0
20. Given x ≥ 2 and |3x − 5| − |x − 1|
∴ The critical points are 3 and 4
When x ≥ 2, both 3x − 5 and x − 1 are positive.
∴ |3x − 5| − |x − 1| = 3x − 5 − x + 1 = 2x − 4 Choice (D)   
2
21. Given x + 5x + 6 < 0
⇒ (x + 3) (x + 2) < 0
The critical points are −2 and −3 3 4

   When x = 0, then the inequality is satisfied.


∴ When x < 3 or x > 4, the inequality is satisfied.
Hence, the solution set is R – [3, 4]. Choice (B)
−3 −2
27. Given 5x2 + 39x − 8 < 0
At x = 0, the inequation is not satisfied. 5x2 + 40x − x − 8 < 0
∴ The solution set is (–3, –2). Choice (A)
⇒ 5x (x + 8) − 1 (x + 8) < 0
1
(x + 8) (5x − 1) < 0 ⇒The critical points are −8 and
22. Given 3x − 2 > 2x − 3 5
x>2−3
x > −1 --- (1)   
x − 3 < 4 − 3x
x + 3x < 4 + 3 ⇒4x < 7
–8 1
7 5
x< --- (2)
4
 7 When x = 0, the inequation is satisfied.
∴ From (1) and (2), the solution set is  − 1,  .
 4  1
∴ From the number line, when x ∈  − 8,  , the
Choice (D)  5
inequation is satisfied. Choice (A)
x+3 x+3
23. Given >2 ⇒ −2>0 28. Given | 7x – 9 | = 15
x −1 x −1
7x – 9 = ±15
x + 3 − 2x + 2 −x + 5 x−5 7x – 9 = 15; 7x – 9 = –15 ⇒7x = 24; 7x = 6
≥0⇒ ≥0 ⇒ ≤0
x −1 x −1 x −1 24 −6  24 −6 
(x − 5)(x − 1) ≤ 0 ⇒ (x − 1) (x − 5) ≤ 0 x= ;x= ∴x=  ,  Choice (B)
7 7 7 7 
(x − 1)2
The critical points are 1 and 5 29. Given ||x + 5| − 9| = 7
Case 1:
   x > −5 ⇒ x + 5 > 0. ⇒ |x + 5| = x + 5.
So, ||x + 5| − 9| = 7 ⇒ |x + 5 − 9| = 7 ⇒ |x − 4| = 7
x−4=±7
1 5 x = 7 + 4; x = −7 + 4
⇒ x = 11 or x = −3 which agree with x > –5.
When x = 0, the inequality is not satisfied
Case 2:
∴ From the number line, x ∈ [1, 5]
But when x = 1, the denominator will be zero. x < −5
∴ x ∈ (1, 5]. Choice (C) |x + 5| = −(x + 5)
∴ ||x + 5| − 9| = 7 ⇒ |−x − 5 − 9| = 7
⇒ |x + 14| = 7 ⇒x + 14 = ± 7
24. Given 4x + 3 > 3x – 1
x = −7 or x = −21 which agree with x < –5
4x – 3x > – 1 – 3 ⇒ x > – 4. --- (1)
∴ The number of solutions is 4. Ans: (4)
5x – 9 > 6x – 5 ⇒ 5x – 6x > 9 – 5
–x > 4 ⇒ x < – 4 --- (2) 30. Given |2x + 5| ≥ 7
From (1) and (2), We know that if |x| ≥ a, then x ≤ −a or x ≥ a
There is no value of x which satisfies the given
So, |2x + 5| ≥ 7 or ⇒ 2x + 5 ≤ −7 or 2x + 5 ≥ 7
simultaneous equations.
2x + 5 ≤ −7 or 2x + 5 ≥ 7 ⇒ 2x ≤ −12 or 2x ≥ 2
∴ The solution set is φ. Choice (D)
⇒x ≤ −6 or x ≥ 1
∴ The solution set is (−∞, −6] ∪ [1, ∞). Choice (B)
Triumphant Institute of Management Education Pvt. Ltd. (T.I.M.E.) HO: 95B, 2nd Floor, Siddamsetty Complex, Secunderabad – 500 003.
Tel : 040–27898195 Fax : 040–27847334 email : info@time4education.com website : www.time4education.com SM1001920.Sol/46
31. Given |3x + 5| ≤ 11 38. Given 5x – 8 < 2x + 9
We know that, |x| ≤ a ⇒ −a ≤ x ≤ a ⇒ 3x < 17;
So, |3x + 5| ≤ 11 ⇒ −11 ≤ 3x + 5 ≤ 11 ⇒ 17
x< ; → (1)
−16 3
−11 − 5 ≤ 3x ≤ 11 − 5 ⇒ −16 ≤ 3x ≤ 6 ⇒ ≤ x ≤ 2.
3 4x + 7 > 7x – 8
Choice (D) ⇒ 15 > 3x
⇒ x < 5 → (2)
32. Given x |x − 5| = 6 The common solution for (1) and (2) is x < 5. Range of x is
Case 1:
(– ∞, 5) Choice (B)
x > 5 ⇒ |x − 5| = x − 5
∴ x (x − 5) = 6 ⇒ x (x − 5) = 6 x 2 − 5 x + 10
⇒ x2 − 5x − 6 = 0 ⇒ x2 − 6x + x − 6 = 0 39. ≤1
3 x 2 − 5 x − 22
⇒ x (x − 6) + (x − 6) = 0 ⇒ (x − 6) (x + 1) = 0
x = −1, 6 But x > 5. ∴ x = 6 x 2 − 5 x + 10
⇒ –1≤0
Case 2: 3 x 2 − 5 x − 22
x < 5 ⇒ |x − 5| = −(x − 5) ∴ x |x − 5| = 6 ⇒ x (5 − x) = 6
x 2 − 5 x + 10 − 3 x 2 + 5 x + 22
5x − x2 = 6 ⇒ x2 − 5x + 6 = 0 ⇒(x − 3) (x − 2) = 0 ⇒ ≤0
⇒ x = 2, 3 3 x 2 − 5 x − 22
∴ The solution set A = {2, 3, 6}. Choice (C)

−2x 2 + 32
≤0⇒
(
− 2 x 2 − 16 )
≤0
33. Given x2 − 13 |x| + 36 = 0 3 x − 5 x − 22
2 (3x −11)(x + 2)
|x|2 − 13 |x| + 36 = 0
|x|2 − 9 |x| − 4 |x| + 36 = 0 ⇒
(x 2
− 16 )(3 x − 11)(x + 2)
≥0
(|x| − 9) (|x| − 4) = 0 (3x −11)2 (x + 2)2
|x| − 9 = 0 or |x| − 4 = 0 ⇒ (x2 – 16) (3x – 11) (x + 2) ≥ 0
|x| = 9 or |x| = 4 ⇒ x = ± 9 ; x = ± 4 11
∴ The number of solutions is 4. Ans: (4) Critical points are – 4, – 2, ,4
3
34. Given |x + 1| + |x − 3| = 6 i ii iii iv v
Case 1: ✓ × ✓ × ✓
when x > 3; • • •
|x + 1| = x + 1, |x − 3| = x − 3 −∞ −4 −2 11/3 4 ∞
∴ |x + 1| + |x − 3| = 6 ⇒ x + 1 + x − 3 = 6
⇒ 2x − 2 = 6 ⇒ 2x = 8 ⇒ x = 4
When x = 0, the above inequation is satisfied
Case 2:
∴ I, iii, v regions together form the solution.
When x < −1,
|x + 1| = − (x + 1) ; |x − 3| = − (x − 3)  11 
Solution set is (– ∞, – 4) ∪  − 2,  ∪ (4, ∞)
∴ |x + 1| + |x − 3| = 6 ⇒ − (x + 1) − (x − 3) = 6  3
⇒ − 2x + 2 = 6 ⇒ 2x = − 4 ⇒ x = −2 Choice (C)
Case 3:
40. x2 – 12x + 42 > – 5
When −1 < x < 3, ⇒|x + 1| = x + 1 ; |x − 3| = 3 − x
⇒ x2 – 12x + 42 + 5 > 0
∴ |x + 1| + |x − 3| = 6 ⇒ x + 1 + 3 − x = 6 ⇒ 4 = 6
⇒ x2 – 12x + 47 > 0
which is not possible
⇒ x2 – 12x + 36 + 11 > 0
∴ The solution set is {−2, 4}. Choice (C)
⇒ (x – 6)2 + 11 > 0
35. Given f(x) = 18 − |3x + 9| (x – 6)2 is always positive.
Since |3x + 9| ≥ 0 for all x ∈ R, ∴ For all values of x, (x – 6)2 + 11 is positive
18 − |3x + 9| ≤ 18 ∴ solution is x ∈ R Choice (B)
i.e., f(x) ≤ 18 Exercise – 19
∴ The maximum value of f(x) is 18. Ans: (18) (Sequences and Series)
x+5 Solutions for questions 1 to 50:
36. ≤2
x −3
1. Let the first term and common difference be a and d
x+5 respectively
⇒ –2≤0
x −3 a + 6d = 34 ---------- (1)
a + 11d = 59 --------- (2)
x + 5 − 2x + 6 11 − x
⇒ ≤0⇒ ≤0 Solving (1) and (2) :
x −3 x −3 a = 4, d = 5

(11− x )(x − 3) ≤ 0 ⇒ (11 – x) (x – 3) ≤ 0
T29 = a + 28d = 144 Ans: (144)
(x − 3)2 2. As there are eight arithmetic means between the numbers
Critical points are 3, 11 7 and 43, there are ten terms in arithmetic series including
7 and 43.
I II First term = 7, Last term = 43
⇒ 43 = 7 + (10 – 1) d ⇒ d = 4
✓ × ✓
• • Fourth arithmetic mean from 43 = fifth arithmetic mean from
–∞ 3 11 ∞ 7 = sixth term of the series = 7 + (6 – 1)4 = 27
When x = 0, the inequation is satisfied. Regions I and II
∴Fourth arithmetic mean = 27 Ans: (27)
satisfy the inequation.
∴ solution set is (– ∞ , 3) ∪ [11, ∞) Choice (D) 3. 8 X T8 = 17 x T17
⇒ 8(a + 7d) = 17(a + 16d)
37. 4x + 3 ≥ 3x – 12
⇒ a = –24d
4x – 3x ≥ – 15
T25 = a + 24d = –24d + 24d = 0 Choice (B)
x ≥ – 15 Choice (D)
Triumphant Institute of Management Education Pvt. Ltd. (T.I.M.E.) HO: 95B, 2 Floor, Siddamsetty Complex, Secunderabad – 500 003.
nd

Tel : 040–27898195 Fax : 040–27847334 email : info@time4education.com website : www.time4education.com SM1001920.Sol/47


4. GM² = AM x HM x
12 5 ⇒a=
GM2 = x ( x − 1)
5 2 As 0 < b < 1
⇒ ( ab ) = 6 ⇒ ab = 6 (as the numbers are positive)
2
y=
1
(1 − b)
⇒b=
( y − 1)
y
Choice (A)
As 1/a < b/a < 1
n
5. Sn = (2a + (n − 1) d) b b2
2 = 1+ + + ..........
a a2
11
= − 176 = (2a + 10 ( −3) ) 1 1
2 = +
 b  (1 − 1 / y ) 
⇒ a = –1 Ans: (–1) 1 −  1 − 
 a   ( x /( x − 1) ) 
6. 7(a + 20d) = 12(a + 15d)
⇒ 5a = –40d ⇒ a = –8d 1
=
S17 = 17/2 [ 2a + 16d]  ( y − 1) ( x − 1) 
= 17/2[–16d + 16d] = 0 Choice (C) 1 − 
 xy 
7. Sn of AP = n x Arithmetic mean xy
⇒ 1836 = n x 51 ⇒ n = 36 Choice (B) = Choice (C)
( x + y − 1)
8. Tn = 9n + 2
T1 = 9 x 1 + 2 = 11 3 1
T2 = 9 x 2 + 2 = 20 17. a= ,r =
4 4
∴Common difference = 9 Choice (A)
S6 = a
(1− r ) n

Sum of n terms = [a + l )]
n (1− r )
9.
2
3   1  
6
n , 1 −   
⇒ (11 + 44 ) = 330 ⇒ n = 12
2
Ans: (12)
4  4 
=  
10. nth term = 311 – n  1
1 − 
6th term = 311 - 6 = 35 = 243 Choice (B)  4
11. nth term = 62n – 3 1 4095
First term = 62x1 – 3 = 1/6
= 1− = Choice (C)
212 4096
Second term = 62 x 2 – 3 = 6
Common ratio = 6/1/6 = 36 Choice (A) 18. 1 + 4 + 42 +…..4n > 2006
12. Geometric mean = n Pr oduct of n terms  4 n + 1 − 1
1   > 2006
⇒ 90 = 5 (216 ) (54) ( x ) (30) (27)  4 − 1 
⇒ x = 625 Ans: (625) 4n + 1 – 1 > 6018
⇒ 4n+1 > 6019 ⇒ n + 1 > 6
13. As the terms are in geometric progression The least value of n satisfies the above inequation is 6.
64 = (3 − 5 ) x (3 + 5 )
n n
. Squaring both sides,
Ans: (6)

64² = (9 – 5)n ⇒ 2ab


19. Harmonic mean of a and b is
46 = 4n ⇒ n = 6 Choice (C) a+b
Required harmonic mean is
14. a = 3
2 × 15 × 25 2 × 15 × 25
a = = 18.75 . Choice (D)
S∞ = =4 15 + 25 40
1− r
3 20. a = 11, d = –4, T18 = a + 17d
⇒ = 4 ⇒ r = 1/4. Choice (B) = 11 + 17(–4) = –57 Ans: (–57)
1− r

15. 5a/2 .5a/6 . 5a/18 . . . . . . . . . . ∞ = (125)1/2 21. Given 2x + 1, 3x + 4 and 7x – 2 are in A.P
a a a  2(3x + 4) = 2x + 1 + 7x – 2
 + + + ........ ∞  ⇒ 6x + 8 = 9x – 1
=5  2 6 18  = 53 / 2 ⇒ 9 = 3x
a a a 3 ⇒ x = 3. Choice (A)
= + + + ........ ∞ =
2 6 18 2
22. Let the first term and the common difference of the
a/2 3 arithmetic progression be a and d respectively
= =
[ 1 − (1 / 3) ] 2 Normal method:
3a 3 nth term tn + a = (n – 1) d
= =
4 2 ∴ 5th term = a + 4d and 11th term = a + 10d
⇒a=2 Ans: (2) Given a + 4d = 14 ------ (1)
a + 10d = 44 ------ (2)
16. As a > 1, 1/a < 1 and 1/a > 0 Solving (1) and (2)
1 a We get = a = –6, d = 5
x= =
(1 − 1 / a) (a − 1) 8th term = a + 7d = –6 + 7(5) = 29

Triumphant Institute of Management Education Pvt. Ltd. (T.I.M.E.) HO: 95B, 2nd Floor, Siddamsetty Complex, Secunderabad – 500 003.
Tel : 040–27898195 Fax : 040–27847334 email : info@time4education.com website : www.time4education.com SM1001920.Sol/48
Alternate method: 1
Since 5th term, 8th term and the 11th terms are in arithmetic 1 1 1 4 1
Consider + + + ........ = =
progression 4 16 64 1 3
1−
∴ 2(8th term) = 5th term + 11th term = 14 + 44 = 58 4
(∵ If a, b and c are in arithmetic progression, 2b = a + c) 1
+
1
+
1
+ ........
1

∴8th term = 29. Ans: (29) ∴Q4 16 64 = Q 3 = P (Q P3 = Q). Choice (B)


23. Let the first term and the common difference of the 1 1
29. Given m = 1 + + + …..
arithmetic progression be a and d respectively p2 p4
Given
6(a + 5d) = 9(a + 8d) ----- (1) 1 p2
m= = and
6a + 30d = 9a + 72d 1 2
p −1
0 = 3a + 42d 1−
p2
0 = a + 14d
∴ 15th term = 0 Ans: (0) 1 1 1 q2
n=1+ + = n=
2 4 2
24. Let the first term and the common difference of the p p 1−
1 q −1
arithmetic progression be a and d respectively q2
Given a + 2d = 20 ---- (1) and a + 6d = 3a ⇒ 6d = 2a
1 1 p2 − 1 q2 − 1 (p 2 − 1)q2 + ( q2 − 1)p 2
⇒ 3d = a Now + = + =
2 2
Substitute the value of a is (1), 3d + 2d = 20 m n p q p 2 q2
∴ 5d = 20
d = 4 ⇒ a = 12 2p 2 q2 − q2 − p 2 2p 2 q2 − (p 2 + q2 )
= =
2 2
n p q p 2 q2
Sum of the first 15 terms = [2a + (n – 1)d]
2
2p 2 q2 − p 2 q2 p 2 q2
15 = = = 1 (Q P2 + q2 = p2.q2).
2 2
= [2(12) + (14) (4)] p q p 2 q2
2
Choice (A)
15
= (80) = 600. Choice (A) 30. x, y and z are in arithmetic progression
2
∴ 2y = x + z ---- (1)
x, y, z + x are in geometric progression
2
25. Given a = 18 and ar = 12 ⇒ r = . ∴ (y)2 = x(x + z)
3 2
x+z
2 256
7 (1) ⇒   = x(z + x)
∴ t = ar = 18   =
8 7
Choice (C)  z 
3 243
⇒ x + z = 4(x)
26. Geometric mean of N terms = Nth root of their product ⇒ z = 3x
∴ Geometric mean of the given 7 terms ∴x:y:z
7 1+ 2 + 3 + 4 + 5 + 6 + 7
= x : 2x : 3x
7
= 3.3 2.33.3 4.35.3 6.37 = 3 i.e. 1 : 2 : 3
Alternately, from options it can be observed that only
( )
1 28
7 Choice (A) satisfies this condition. Choice (A)
= 3 28 = 3 28 7 = 37 = 34 = 81
31. Let the three terms in G.P be a/r, a, ar
Alternate method: Product = a/r × a × ar = 1000
If a geometric progression has an odd number of terms, ⇒ a = 10
their geometric mean = The middle term. The given terms sum = 10/r + 10 + 10r = 35
are in geometric progression ⇒ r = 2, 1/2
∴ Their geometric mean = Middle term = 4th term = 34 = 81 ∴ the three terms are 5, 10, 20 Choice (A)
Choice (C) 32. We know that the geometric mean of any two positive
27. Given Xp = ( 2) p numbers a and b = ab .
n ∴ Geometric mean of 6 and y = 6y
∑ xp = x1 + x2 + ….. + xn
p=1
Given 6 y = 12
= 2+ ( 2 ) + ….. + ( 2 )
2 n
Squaring on both sides, 6y = 144. ⇒ y = 24. Ans: (24)

2  ( 2 ) − 1
 n  1 1 1
n
  33. Let ps = qt = ru = k. Then p = , q = k and r = k u . ks t
∑= = 15 2 + 30 Given p, q and r are in geometric progression
p =1 2 −1
∴ q2 = pr

( )n 
2  2 − 1  1
2
 1  1

2 −1
 = 15 2 ( 2 + 1)  t
k 
 
 
= k s 
 
 u
k 
 
     
( ) − 1 = 15 2 ( 2 + 1) ( 2 − 1)

2  2

n 2
k t = ks
1 1
+
u

( 2 ) – 1 = 15
n
Equating the powers of k on both sides =
2
t
=
1
s
+
1
u
( 2 ) = 16 ⇒ 2 = ( 2 ) ∴ n = 8.
n 4
8
Choice (B) 2
=
u+s
t s.u
1 1 1 1 1 1
+ + + ........ s.u t
28. Q 4 . Q 16 . Q 64 ………. = Q 4 16 64 ∴ = . Choice (D)
u+s 2
Triumphant Institute of Management Education Pvt. Ltd. (T.I.M.E.) HO: 95B, 2nd Floor, Siddamsetty Complex, Secunderabad – 500 003.
Tel : 040–27898195 Fax : 040–27847334 email : info@time4education.com website : www.time4education.com SM1001920.Sol/49
34. Given p, q and r are in harmonic progression 44. Sn = 11x2 + 8x
1 1 1 ∴ Sn = 11n2 + 8n
We know that , and are in arithmetic progression nth term = Sn – Sn – 1
p q r
11n² + 8n – [11(n – 1)² + 8(n – 1) = 22n – 3
---(1) = 22 (n – 1) + 19
If a, b and c are in arithmetic progression, ka, kb and kc are = 19 + 22 (n –1)
also in arithmetic progression where k is any value Common difference = 22 Ans: (22)
 1  1
∴ (1) ⇒ p + q + r   , (p + q + r)   and (p +q + r) 45. Let the three terms in A.P be a – d, a, a + d
p
  q sum = 3a = 21 ⇒ a = 7
 1 Also, (7 – d )2 + 49 + (7 + d )2 = 165
  are in arithmetic progression ⇒d=±3
r
∴the numbers are 4, 7, 10 Choice (C)
q+r p+r p+q
∴ 1+ , 1+ and 1 + are in arithmetic
p q r 46. Let the first term and the common difference of the A.P be
progression a and d respectively.
q+r p+r p+q 7th term = a + 6d = 13 → (1)
∴ , and must be in arithmetic 18th term = a + 17d = 35 → (2)
p q r
Solving (1) and (2) we get a = 1, d = 2
progression. Choice (C) 23rd term = a + 22d = 45 Ans: (45)
35. The number of ants in the parallel rows are as follows
47. Let the first term and the common difference of the A.P be
2, 4, 8 ….
a and d respectively. Third term = a + 2d = 30 → (1)
It is a GP of a = 2, r = 2
Given, t7 = 3t1
S12 = 2(212 – 1)/1 = 8190 Choice (D)
a + 6d = 3a
36. Let the three numbers be a – d, a, a + d a = 3d
Given sum of the numbers = 12 substitute the value of a in (1)
i.e., a – d + a + a+ d = 3a = 12 3d + 2d = 30
⇒ a =4 ∴ 5d = 30; d = 6
Again, a + d = 3 (a – d) ∴ a = 18
4d = 2a ⇒ d = 2 Sum of the first 15 terms =
∴ The three numbers are 2, 4, 6. 15 n
S15 = (2(18) + (14) (6)) ( Q s n = (2a + (n − 1)d)
Their product is = 2 × 4 × 6 = 48 Choice (D) 2 2
15
37. a = 110, r = 1/11. = (120) = 900 Choice (C)
Sum to infinite terms = 110/1 – (1/11) = 121. Choice (C) 2

38. a = 8, d= 5 48. a = ¼ , r = 4
Let the number of terms be n T8 = ar7 = ¼ × 47 = 46 = 212 Choice (C)
n/2 [16 + (n – 1)5] = 645
49. The series is a G.P, a = 1/64, r = 2.
⇒ 5n² + 11n – 1290 = 0
T16 = ar15 = 1/26 × 215 = 29 = 512 Ans: (512)
⇒ 5n2 + 86n – 75n – 1290 = 0
⇒ n = 15 and –86/5 50. Let the common ratio be r. Then, 2801 = 2401r – 1 / r – 1
As n cannot be negative, n = 15. Choice (B) ⇒r=7 Ans: (7)
39. Let the three terms in A.P be a – d, a , a + d Exercise – 20
Sum = a – d + a + a + d = –30 ⇒ a = –10 (Functions)
Also, (a – d + a) – (a + a + d) = 8 ⇒ d = –4
Solutions for questions 1 to 55:
∴the three terms in A.P are –6, –10, –14 Choice (D)
40. Let the first term and the common difference of the A.P be 1. Given, A ⊂ B and n (A) = 6, n(B) = 10
a, d respectively. So, n(B – A) = n(B) – n(A) = 10 – 6 = 4
T4 = a + 3d = 5 ∴ the number of subsets of (B – A) = 2 n(B – A) = 24 = 16
T16 = a + 15d = 13 Ans: (16)
⇒ a = 3; d = 2/3 2. From choice (A), A = {2} ⇒ A is not a void set
2 From choice (B), B = {1} ⇒ B is not a void set
⇒ T6 = 3 + × 5 = 19/3
3 From choice (C), C = { } ⇒ C is a void set Choice (C)
last term, Tn = 3 + ( n – 1)2/3 =21 ⇒ n = 28
Choice (B) 3. If A ∩ B = φ, then A ∆ B will have maximum number of
elements.
41. The two digit numbers which leave a remainder of 2 when In this case, n (A ∆ B) = n(A) + n(B) = 8 + 12 = 20
divided by 6 are 14, 20, 26, …. 98 Ans: (20)
Tn = 14 + (n – 1)6 = 98 ⇒ n = 15
15 4. As n(A) = 8 and n(B) = 4, the set A ∪ B will have minimum
Sn = [14 + 98] = 840. Ans: (840) number of elements if B ⊂ A.
2
Q Accordingly we have n(A ∪ B) = 8 Ans: (8)
42. Let the four terms in A.P be a – 3d,a – d, a + d, a + 3d.
Sum = a – 3d + a – d + a + d + a + 3d = 30 5. Given, A ∩ B = φ
⇒ a = 15/2 So, A ∆ B = A ∪ B
Also, (a – d)(a + d) – ( a – 3d)(a + 3d) = 18 ∴n(A ∆ B) = n(A ∪ B) = n(A) + n(B)
⇒ d = ± 3/2 ∴ choice (A) is true.
∴the four terms are 3, 6, 9, 12 Choice (D) A– ∪ B– = (A ∩ B)– = µ – (A ∩ B) = µ
Choice (B) is true.
43. Let the first term and common difference be a and d Since A ∩ B = φ, A – B = A and B – A = B
respectively then, 3t2 = t8 – t4 ⇒ (A – B) ∩ (B – A)= φ
⇒ 3(a + d) = a + 7d – (a + 3d) ⇒ 3a = d ∴ choice (C) is also true. Choice (D)
⇒ a : d = 1:3 Choice (B)
Triumphant Institute of Management Education Pvt. Ltd. (T.I.M.E.) HO: 95B, 2nd Floor, Siddamsetty Complex, Secunderabad – 500 003.
Tel : 040–27898195 Fax : 040–27847334 email : info@time4education.com website : www.time4education.com SM1001920.Sol/50
6. (A ∪ B)– ∩ B = (A– ∩ B– ∩ B = A– ∩(B– ∩ B) 17. Dual of the statement,
= A– ∩ φ = φ Choice (B) A' ∩ (A ∩ B)= φ is A' ∪ (A ∪ B) = µ Choice (B)

7. The number of elements in the power set of a set A having 18. Given, A = {1, 2, {6, 5, 3,}, 4, {7, 8}}
n elements is 2n. ∴n(A) = 5 Ans: (5)
Now, 64 = 28 19. We know that if a set A contains n elements, then the
1 = 20 number of subsets of A that contain exactly ‘r’ elements is
But 127 cannot be expressed in the form 2n for n ∈ N. n
Cr.
Choice (A) Here n(A) = 7; r = 3
∴ the number of subsets of A that contain exactly
8. Given, A = {3, 6, 9, 12, 15, 18, 21, 24, 27, 30, 33, 36, 39, 7×6×5
42, 45, 48} 3 elements is 7C3 = = 35 Ans: (35)
B = {9, 18, 27, 36} 3 × 2 ×1
∴ A – B = {3, 6, 12, 15, 21, 24, 30, 33, 39, 42, 45, 48}.
20. Given, n(A) = 10, and n(B) = 15 and n(A ∪ B) = 20
Choice (C)
We know that,
9. Given, n(A ∩ B) = n(A) + n(B) – n(A ∪ B)
= 10 + 15 – 20 = 5
5n2 + 6n + 12 12 ∴n(B – A) = n(B) – n(A ∩ B)
p= = 5n + 6 +
n n = 15 – 5 = 10 Ans: (10)
When n = 1, 2, 3, 4, 6 and 12 only p will be a natural
number 21. We know that, if A ⊆ B, then A – B = φ Choice (A)
n = 1 ⇒ p = 23, n = 4 ⇒ p = 29 22. Idempotent law. Choice (C)
n = 2 ⇒ p = 22, n = 6 ⇒ p = 38
n = 3 ⇒ p = 25 and n = 12 ⇒ p = 67 23. The maximum number of elements in B = the number of
A = {23, 29, 67} elements in A ∪ B = 20 Ans: (20)
∴ n(A) = 3 Ans: (3)
24. Given, n(A) = 4 and n(B) = 3
The total number of relations that can be defined from
10. Given, A = {1, 2, 3, 4, 5}
A to B is 24 × 3 = 212
The total number of subsets of A is 25 = 32
The number of functions that can be defined from A to B is
Number of subsets of A which contain exactly one element
34 = 81
= 5.
∴the number of relations which are not functions is
∴ the total number of subsets of A which contain at least
212 – 81 = 4096 – 81 = 4015 Ans: (4015)
two elements = 25 – 5 – 1 = 26 Ans: (26)
25. We know that, n(A ∪ B) = n(A) + n(B) – n(A ∩ B)
11. Given, ⇒ n(B) = n(A ∪ B) + n (A ∩ B) – n(A) = 9 + 3 – 4 = 8
A = {3, 4, 5, 6}
B = {6, 7, 9, 10} ∴the number of relations possible from B to A is 2n( A ).n (B )
C = {5, 7, 10, 12}
∴ (A ∪ B) ∩ (A ∪ C) = A ∪ (B ∩ C)
( )
= 28 × 4 = 232 = 2 4 = 168
8
Choice (C)

= {3, 4, 5, 6} ∪ {7, 10} = {3, 4, 5, 6, 7, 10}  a  g(a)


Choice (D) 26. g  =
 b  g(b)
12. Given, A = {1, 2, 3, 4, 5, 6} 1 1
Given g(6) = g(3) =
B = {4, 5, 6, 7, 8, 9, 10} 36 3
We know that, (A – B) ∪ (B − A) ∪ (A ∩ B) = A ∪ B 1
∴n((A – B) ∪ (B – A) ∪ (A ∩ B)) = n(A ∪ B)  6  g(6) 36 1 1
Now, n(A ∪ B) = n(A) + n(B) – n (A ∩ B) = 6 + 7 – 3 = 10 g(2) = g   = = = ×9=
 3  g(3) 1 36 4
Ans: (10)
9
13. Since n(A) = 5 and n(B) = 3, A ∩ B can have a maximum of  
2 g ( 2 )
1
3 elements. Ans: (3) g   = = 4 =9
 3  g(3) 19 4

14. Given, A ⊆ B ⊆ C
g(3 )
( )
1
Then, B ∪ (C – A) = C Choice (B) g 3 = = 9 =4
2 g(2) 1 9
4
15. We have, (A ∩ B)' = A' ∪ B' (Demorgan’s law)
 2   3  65
A ∪ (B ∩ C) = (A ∪ B) ∩ (A ∪ C) (Distributive law) g   − g  = Choice (C)
A – (B ∩ C) = (A – B) ∪ (A – C) (Demorgan’s law).  3   2  36
Choice (D) 27. h(x + y) = h(x). h(y)
h(6) = h(3 + 3) = h(3). h(3)
16. Given, the total number of men, n(S) = 36 15625 = [h(3)]2
2 125 = h(3) Choice (A)
From the data, n(A) = × 36 = 24
3 28. f(x + 1) = 3 f(x) – f(x – 1)
1 f(2) = 3 f (1) – f (0) = 3
n(B) = × 36 = 6 f(3) = 3 f (2) – f (1) = 7
6
f(4) = 3f(3) – f (2) = 18
1 f(5) = 3f(4) – f(3) = 47
n(A ∩ B) = × 36 = 4
9 ∴ f(5) – f(4) = 29. Ans: (29)
∴ n(A ∪ B) = n(A) + n(B) – n (A ∩ B)
= 24 + 6 – 4 = 26 x−2 x−2 x−2 1
29. f|x| = = = =±
So, the number of men who belong to A or B is 26. 2x − 4 2( x − 2) ± 2( x − 2) 2
∴ the number of men who belong to neither of the groups
1
is 36 – 26 = 10. Ans: (10) |f(x)|= Choice (B)
2
Triumphant Institute of Management Education Pvt. Ltd. (T.I.M.E.) HO: 95B, 2nd Floor, Siddamsetty Complex, Secunderabad – 500 003.
Tel : 040–27898195 Fax : 040–27847334 email : info@time4education.com website : www.time4education.com SM1001920.Sol/51
30. Standard result. Choice (B) 40. Given, f(x + 2) = x2 + 7x – 5. Put x = –2
Then, f(0) = (–2)2 + 7(–2) –5 = 4 – 14 – 5 = –15
31. By the definition of a function, all the three choices Ans: (–15)
represent a function. Choice (D)
41. Given, f(x) = ax2 – bx – a
2
32. Given, n(A) = 5 and n(B) = 4.  − 1  − 1  − 1
The number functions that can be defined from A to B ∴ f   = a  − b  – a
= 45 =1024  x   x   x 
Since n(A) > n(B), the number of one – one functions is a b a + bx − ax 2
zero. = 2
+ –a=
∴ every function defined will be a many-one function. x x x2
Hence, the number of many-one functions = 1024.
Choice (A) =–
(ax 2
− bx − a ) =– f (x )
Choice (B)
x2 x2
33. A constant function from a set A to a set B will be an onto
function only if n(B) = 1. Choice (C) 2x + 3
42. Given, f(x) =
3x + 2
x Let f(x) = y
34. Given, f(x) =
1+ x 2x + 3
⇒ = y ⇒ 2x + 3 = 3xy + 2y
x 3x + 2
 x  1+ x x 2y − 3
∴ fof(x) = f (f (x)) = f   = = ⇒ x(2 – 3y) = 2y – 3 ⇒ x =
 1 + x  x 1 + 2x 2 − 3y
1+
1+ x 2y − 3 2x − 3
fofof(x) = f(fof(x)) ∴ f–1 (y) = and hence f–1 (x) =
2 − 3y 2 − 3x
x Choice (D)
 x  1 + 2x x
= f   = =
 1 + 2x  x 1+ 3x 43. Given, f(x) = log (2x + 3) + 3 x − 5
1+
1 + 2x Consider log (2x + 3)
We know that log x is defined only when x > 0
x
Generalising for ‘n’, we get fofo.....f ( x ) =
14243 −3
1 + nx ∴ 2x + 3 > 0 ⇒ x >
n times 2
Choice (B)  −3 
3x + 2 ∴ the domain of log (2x + 3) is  , ∞ 
35. f(x) =  2 
5x − 3
fof(x) = f(f(x)) Consider 3x − 5
 3x + 2 
3 +2 We know that f (x ) is defined only when f(x) ≥ 0
 3x + 2  5x − 3 
= f =  5 5 
 5 x − 3  5 3 x + 2  − 3 ∴ 3x – 5 ≥ 0 ⇒ x ≥ i.e., x ∈  , ∞  → (2)
 5x − 3 
3 3 
9 x + 6 + 10 x − 6 19 x ∴ from (1) and (2), the domain of log (2x + 3) + 3 x − 5 is
= = =x
15 x + 10 − 15 x + 9 19 5 
∴fof(x) = x  , ∞  Choice (C)
fo[fo(f(x))] = f(x) 3 
fofofofofof(x) = x
⇒ fofofofofof(6) = 6. Choice (2) 44. Given, f(x) = 7x + 3
Let f(x) = y ⇒ x = f-1 (y)
2
36. Given, f(x, y) = x + 2y f(x) = y
∴ f(3, –4) = (3)2 + 2(–4) = 1 ⇒ 7x + 3 = y
⇒ f(2, f(3, –4) = f(2, 1) = 22 + 2(1) = 6 Choice (C) 7x = y – 3
37. Given, f(x) = 3x2 + 4x y −3
x=
g(x) = 3x + 4 7
∴ gof(x) = g [f(x)] = g (3x2 + 4x) = 3 (3x2 + 4x) + 4 y −3 x −3
= 9x2 + 12x + 4 = (3x + 2)2 Choice (C) ∴ f-1 (y) = and hence f-1(x) =
7 7
x -1 −4 − 3 -1 10 − 3
38. Given, f(x) = and g(x) = x – [x] So, f (−4) = = – 1, f (10) = =1
x 7 7
We know that the range of f(x) is {–1, 1} ∴ f-1 ({4, 10}) = {–1, 1} Choice (B)
Case (1) let f(x) = −1. Then gof(x) = g(f(x)) 2
= g (–1) = –1 – (–1) = 0 45. Given, f(x) = x + sin x
Case (2): let f(x) = 1. Then gof(x) = g(f(x)) = g(1) = 1 – [1] = 0 f(–x) = (–x)2 + sin (–x) = x2 – sin x
∴the range of gof(x) = {0} Choice (A) ∴ f(x) + f(–x) = 2x2 which is an even function
∴ f(x) + f(–x) is an even function. Choice (A)
39. If f : A → B is a bijective function, then A and B should
contain same number of elements. 46. If n(A) = m and n(B) = 2, then the number of onto functions
Let n(A) = n(B) = m that can be defined from A to B is 2m – 2
Then the number of bijective functions that can be defined Here n(A) = 4, n(B) = 2
from A to B is m ! ∴the number of onto functions is 24 – 2 = 16 – 2 = 14
∴ m ! = 120 ⇒ m ! = 5! ⇒ m = 5. Ans: (5) Choice (B)

Triumphant Institute of Management Education Pvt. Ltd. (T.I.M.E.) HO: 95B, 2nd Floor, Siddamsetty Complex, Secunderabad – 500 003.
Tel : 040–27898195 Fax : 040–27847334 email : info@time4education.com website : www.time4education.com SM1001920.Sol/52
47. Given, f = {(1, 3), (4, 2), (2, 1), (3, 4)} 2. y = –f(–x) is obtained by reflecting the graph y = f(x) in both
Domain of f = {1, 2, 3, 4} the X-axis as well as in the Y-axis. In general the order of
fof(1) = f (f(1)) = f(3) = 4, (1, 4) the reflections is important. But in this special case, we get
fof(2) = f(f(2)) = f(1) = 3, (2, 3) the same image whether we reflect first in the x-axis or first
fof(3) = f(f(3)) = f(4) = 2, (3, 2) in the y-axis. Choice (C)
fof(4) = f(f(4)) = f(2) = 1, (4, 1)
3. The graph in fig(2) is obtained by shifting the graph in fig(1)
∴ fof = {(1, 4) (2, 3), (3, 2), (4,1)}. Choice (A)
towards left horizontally for 3 units and down by one unit
48. A function f: A → B is a one – one function, if distinct g(x) = f(x+3) –1 Choice (C)
elements in A are assigned to distinct elements in B.
4. The graph in Fig 2 stretches from x = –6 to x = –2
In Choice (A), 2 and 4 are assigned to 6. So it is not
The graph in Fig 1 stretches from x = 0 to x = 4
one – one.
In Fig 2, to obtain y at x = –2, we must consider x = 0 in Fig 1.
In Choice (C), 3 is not assigned to any element in B,
In Fig 2, to obtain y at x = –6, we must consider x = 4 in Fig 1.
∴ Choice (C) is not a function. In general, to obtain y at x = a we must consider x at –a–2
In Choice (D), 2 and 3 are assigned to 3. So, it is not in Fig 1.
one – one Choice (B) Also, we have to add 3
49. If the range = codomain, then f must be an onto function. i.e, g(x) = f(–x–2) +3 Choice (D)
Choice (D)
5. The graph in fig(2) is obtained from the graph in fig(1) by
50. Given, f(x) = 4x + 5 and g(x) = x + 2 shifting the graph of fig(1) vertically down for 8 units
We know that (fg) (x) = f(x) . g(x) g(x) = f(x) – 8 Choice (B)
∴(fg) (2) = f(2) . g(2)
= (4(2) + 5) . (2 + 2) = 13.4 = 52. Ans: (52) 6. Choice (A) : (1, 2), 2(1) – 3(2) ≤ 5
–4 ≤ 5 True
51. Given, f(x) = x + 3 and g(x) = x + 5 2
Choice (B) : (–3, 1), 2(–3) –3(1) ≤ 5
We have, –9 ≤ 5 True
fog(x) = f(g(x)) Choice (C) : (3, 2), 2(3) –3(2) ≤ 5
= f(x2 + 5) = x2 + 5 + 3 = x2 + 8 Choice (A) 0 ≤ 5 True Choice (D)
2x + 1 7. The graph of y = g(x) is obtained from f(x) by reflecting 1st
52. Given, f(x) = in the Y-axis and then in X-axis (or vice-versa)
x−2
g(x) = –f(–x) Choice (C)
Let f(x) = y
2x + 1 8. The shaded region is denoted by
x
+
y
≤1
⇒ =y −2 −3
x−2
or 3x + 2y + 6 ≥ 0
⇒ 2x + 1 = xy – 2y (2, –5) : 6 – 10 + 6 ≥ 0 True
⇒ 2x – xy = –2y – 1 (–4, 1) : –12 + 2 + 6 ≥ 0 False
⇒ x(2 – y) = – 2y – 1 (2, –7) : 6 – 14 + 6 ≥ 0 False Choice (A)
−2 y − 1
⇒x= 9.
2−y
Y
1 + 2y
⇒ f–1(y) =
y−2
2x + 1 C(0, 3)
∴ f–1 (x) = = f(x) Choice (B)
x−2 D(–3, 0) B(3, 0)
X
0
53. Given, n(A) = 3 and n(B) = 5
∴ number of one – one functions from A to B A(0, –3)
= n(B)Pn(A)
= 5P3 = 60 Ans: (60)
The graph of x + y ≤ 3 is represented by the shaded
54. Given, f : A → B is a bijective function. So, f(A) = B
A = {2, 3, 4, 5} region in the figure
f(x) = 3x – 2 1
The required area = × 6 × 6 = 18 sq. units Ans:18
∴f(2) = 3(2) – 2 = 4 2
f(3) = 3(3) – 2 = 7
f(4) = 3(4) – 2 = 10 Solutions for questions 10 to 15:
f(5) =3(5) – 2 = 13
∴ B = {4, 7, 10, 13} Choice (D) 10. The graph of f(x) + 2 is choice (D). Choice (D)

log (3 − x ) 11. A curve represents a function if no line parallel to y-axis


55. Let f(x) =
x2 − 4 meets the graph of the curve at more than one point.
So, the graph in choice (A) only represents the graph of a
log (3 – x) is defined only when 3 – x > 0 i.e., 3 > x function. Choice (A)
But when x = ± 2, 1 is not defined. 12. f(x – 5) is the graph obtained when f(x) is moved 5 units
x2 − 4 right side to the origin. Choice (C)
∴ the domain of f(x) is (– ∞, 3) – {–2, 2}
Choice (B) 13. f(x) – 4 is the graph obtained when f(x) is moved 4 units
down to the origin. Choice (B)
Exercise – 21
(Graphs) 14. The graph of x = |y – 3| is choice (D). Choice (D)
Solutions for questions 1 to 9: 15. The equation of the line is x – y = 3
1. y = f(–x) is obtained by reflecting the graph y = f(x) in Y-axis Since (0, 0) is in the graph required region is represented
Choice (B) by x – y ≤ 3. Choice (D)
Triumphant Institute of Management Education Pvt. Ltd. (T.I.M.E.) HO: 95B, 2nd Floor, Siddamsetty Complex, Secunderabad – 500 003.
Tel : 040–27898195 Fax : 040–27847334 email : info@time4education.com website : www.time4education.com SM1001920.Sol/53
Exercise – 22  1 1 
(Indices and Surds) 7. x4 – x3 +2x2 + x + 1 = x2  x 2 − x + 2 + + 2 
 x x 
Solutions for questions 1 to 40:  1  1 
= x2  x 2 + 2 − 2 + 4 −  x −  
 x  x 
1. The expression on the left hand side of the equation
equals,  1  1 
= x2   x −  2 + 4 −  x −  
( xy + 1) ( xy − 1)
x x y
 xy + 1   xy − 1 
y
 x  x 
    2 2
 y   x  yx xy = x (y + 4 – y) Choice (B)
=
24n − 22.4n−3
x
( xy + 1) ( xy − 1)
y y x
 xy + 1   xy − 1 
 x    8.
   y  xy yx 42n
= (xy + 1) x–y
(xy – 1) y–x
 1
24n 1 − 
 xy + 1 
x−y 24n − 24n × 2−1  2 1
= = = Ans: (0.5)
=   = zx – y 24n 2 4n 2
 xy − 1 
9. If a number is multiplied by 10 –K, the decimal place in it
xy + 1
∴z= Choice (C) would move K places to the left.
xy − 1 In order to satisfy the given condition, the decimal place in
123456 cannot move by more than 5 places to the left.
∴The maximum value of K is 5. Ans: (5)
(64)2.5 × (256 )1.75 = (2 6 )2 × (2 8 )4
5 7
215 × 214 10. Let 3a = 5b = 15c = K.
2. = = 218
(16)0.25 × 32 2 (2 4 )4 × (25 )2
1
2 × 210
1
3 = K a ; 5 = K b ;15 = K
1 1
c

We know 3.5 = 15
∴ m = 18 Ans: (18) 1 1 1
⇒ K a .K b =K c
3 3 3 3 3 3
3. 4.5 – (2.2 + 2.3 ) = (2.2 +2.3) – (2.2 + 2.3 ) 1 1 1
+
= (2.2)3 + (2.3)3 + 3(2.2) (2.3) ⇒ Ka b =K c
(2.2 + 2.3) – (2.2)3 – (2.3)3
= 3(2.2) (2.3) (2.2 + 2.3) 1 1 1 a+b 1
⇒ + = ⇒ =
= (6.6) (2.3)
(9) = (3.3) (20. 7) a b c ab c
2 ⇒ ac + bc = ab
= (3 + 0.3) (20.7) = 62.10 + 6.21 = 68.310 Choice (B) ⇒ ac – ab = –bc. Choice (B)
5 2 3
4. 93x = 729 × 81x + 4 8 2 (27) 5 6 2
(32)3x = 36 × (34)x + 4 11. 5 2 4
= 2a . 3b
36x = 36 + 4x +16 (36) 3 9 3 16 3

(2 ) (3 ) (2 × 3)
6x = 6 + 4x + 16 5 2
3 2 3 5 3
x = 11 Ans: (11) 2
= = 2a . 3b
5. a b c
Let a = b = c = d = K (K > 1) d (2 . 3 ) (3 ) (2 )
2 2
5
3 2
2
3 4
4
3

1 15 6 3 3
a=K a 2 2 3 5 .2 2 3 2
= 10 4
= 2a . 3b
1 10 16
b=K b 23 3 3 33 2 3

1 15 3 10 16 6 3 10 4
+ − − + − −
c=K c 2 2 3 3
2a. 3b = 2 35 2 3 3
1 1 45 + 9 − 20 − 32 36 + 45 −100 − 40
+
ac = K a c
2a . 3b = 2 6 3 30
1 1
+ −59
bd = K b d 1
2a . 3b = 2 3 3 30
1 1 1 1
+ +
a c =Kb d 1 −59
As ac = bd, K ∴a = ; b=
3 30
1 1 1 1
∴ + = + 1 59 10 + 59 69 23
a c b d a–b= + = = = . Ans: (2.3)
3 30 30 30 10
Alternate method: 12. a = 327
b= 333
As a > 0, b > 0, c > 0 and d > 0 such that aa = bb = cc = dd, it c = 333
is possible only when a = b = c = d. As 33 lies between 33 and 34, c lies between (33)3 and (34)3
1 1 1 1 i.e., 39 and 312
∴ + = + Choice (C) The ascending order of a, b and c is cab. Choice (B)
a c b d
1 1 1
6. 4x + 10 = 64y – 2 = 166 13. (81) 3 × (81) 9 × (81) 27 × ...... ∞
4x + 10 =43(y–2) = 412 1 1 1
x + 10 = 12 3(y–2) = 12 + + +......∞
x=2 y–2=4 = (81) 3 9 27

y=6 1/ 3
∴xy = 26 = 64 Ans: (64) = (81) 2 / 3 = (81)1/2 = 9 Ans: (9)

Triumphant Institute of Management Education Pvt. Ltd. (T.I.M.E.) HO: 95B, 2 Floor, Siddamsetty Complex, Secunderabad – 500 003.
nd

Tel : 040–27898195 Fax : 040–27847334 email : info@time4education.com website : www.time4education.com SM1001920.Sol/54


14. HCF of the powers of a, b, c and d = 25 152 < 35 < 113 < 74
If a, b, c and d are expressed with this HCF as their outer ∴ The descending order is bcad. Choice (B)
powers, we have
a = (26)25
23. x = 4+ 5
b = (34)25
c = (63)25 x–4= 5 ⇒ (x − 4)2 = ( 5 )2
d = (52)25
⇒ x – 8x + 16 = 5
2
52 < 26 < 34 < 63
⇒ x2 – 8x = –11
∴d<a<b<c Choice (B)
Again squaring on both sides
x4 – 16x3 + 64x2 = 121. Choice (C)
1
15. The HCF of the powers of a, b, c and d is
500
24. x = 10 +3 11
If a, b, c and d are expressed with this HCF as their
1 1
powers, we have, =
1 x 10 + 3 11
a = (22) 500 Rationalising its denominator, we have,
1
1 10 − 3 11
b = (1010)
1
500 =
(
x 10 + 3 11 (1 − 3 11) )
= 10 − 3 11

(55) 500 1
c= ∴x + = 20 Choice (D)
1 x
d= (2020) 500
2
Suppose K is a real number greater than 1. Then it follows 1  1
that the larger the value of K, the larger is the value of KK. 25. x2 + =  x +  – 2 - - - - - (1)
x2  x
For this reason, 2020 > 1010 > 55 >22
∴d > b > c > a 1 2
Choice (B) =
x 7 − 45
16. The given equation can be written as
1
1 Rationalising the denominator of , we have,
 5  x + 5  2 1029 3(343 ) x
 
 7 
 =

=
375 3(125 ) 1 2 7 + 45 ( 2 7 + 45 ) ( ) = 7+ 45

x+5
 =
(
x 7 − 45 7 + 45
=
)(
4 ) 2
−3 1
5 2 5 ∴x+
  =   =7
7 7 x
x+5 1
x2 + 2 = 47 from (1) Choice (C)
= –3 ⇒ x = – 11 Choice (C) x
2

9 2 ( 6 + 5 )( 6 + 5 ) ( 6 + 5 ) 2

( ) ×( ) ( 6 _ 5 )( 6 + 5 ) = 1
3
(729 )
2 26. x = = 11 + 2 30
1⋅5 2⋅25  39 
× (81) 36 2 34 4 3 ×3
9 9
17. = = 6 6 = 6 
(3 ) × (3 )
3 
( 6 − 5 )( 6 − 5 )
1⋅2
(243 ) × (27 ) 2 6
5 5 3 2 3 ×3  
36 = 3x ⇒ x = 6. Ans: (6)
y=
( 6 + 5 )( 6 − 5 )
18. 164x = (24)4x = 216x =
( 6 − 5 ) = 11- 2 30
2

256 × 128x + 4 = 28 × (27)x + 4 1


= 28 + 7x + 28 = 27x + 36 xy = 1
∴ 216x = 27x + 36 x2 + xy + y2 = (x + y)2 – xy
= 16x = 7x + 36 = 222 – 1 = 483 Ans: (483)
⇒ x = 4. Ans: (4)
(
1 3+ 5+ 8 )
( 3+ )( 3 + )
19. 8x + 2 = 6410 27. x =
= (82)10 = 820 5− 8 5+ 8
⇒ x + 2 = 20 ⇒ x = 18
16 y + 3 = (42)y + 3 = (43)10
3+ 5+ 8 3+ 5+ 8
( 3 + 5 ) −( 8)
⇒ 2 (y + 3) = 30 ⇒ y = 12 = 2 2
=
∴x − y = 6. Ans: (6) 2 5

1( 3 − 5 + 8 )
20. Given 3x+2 + 3x–3 = 732
⇒ 3x . 32 + 3x . 3–3 = 732
( )(
⇒ 3 x −3 35 + 1 = 732 )
y=
( 3 − 5 − 8 )( 3 − 5 + 8 )
⇒ 3x–3 (244) = 732 3− 5+ 8 3− 5+ 8
( 3 − 5) −( 8)
⇒ 3x–3 = 31 ⇒ x = 4 Ans: (4) = 2 2
=
− 2 15
21. a2 = 0⋅0001 = 10−4
a10 = (a2)5 = (10−4)5 = 10−20. Choice (D) − 3+ 5 − 8
=
+ 2 15
22. HCF of the exponents of a, b, c and d is 4.
Expressing a, b, c and d with this HCF, 3+ 5+ 8− 3+ 5 1
x+y= = Choice (C)
we have, a = (35)4, b = (74)4, c = (113)4 and d = (152)4 2 15 3
As 35 = 243, 74 = 2401, 113 = 1331 and 152 = 225

Triumphant Institute of Management Education Pvt. Ltd. (T.I.M.E.) HO: 95B, 2nd Floor, Siddamsetty Complex, Secunderabad – 500 003.
Tel : 040–27898195 Fax : 040–27847334 email : info@time4education.com website : www.time4education.com SM1001920.Sol/55
1 ( 3 + 2) p = 40 and q = 2 or vice versa
28.
x
=
( 3 − 2 )( 3 + 2 ) ∴ p + q = 40 + 2 Choice (D)

= 3+ 2

x–
1
=–2 2
35. 196 + 2 195 = ( 195 ) + ( 1) + 2 195
2 2
1

( 195 + 1)
x
2
= 195 + 1
( )
4 =
 1 4
 x −  = − 2 2
 x
= 196 − 2 195 = ( 195 ) + ( 1) − 2 195
2 2
1
(2 2 ) 4

( 195 − 1)
= 64 Choice (B) 2
= = 195 –1
29. x = 14 + 10 2
∴ 196 + 2 195 − 196 − 2 195 = 2 Ans: (2)
x
=7+5 2
2
2 7−5 2 ( ) 36. 9 + 2 3 + 2 5 + 2 15
=
(
x 7+5 2 7−5 2 )( ) =5 2 –7
= 1 + 3 + 5 + 2 3 . 1 + 2 5 .1 + 2 5 .3
x 2
+ = 10 2 = 2 50
(1 + )
Choice (C)
2 x 2
= 3+ 5 = 1+ 3 + 5 Choice (D)
1 1
3
30. 6× 2 = 63 × 22 37. a2 = 16 + 2 48

( ) ×( )
1 1 2
b = 19 + 2 48
= 62 6 23 6
c2 = 26 + 2
= (6 × 2 ) =
1 48
2 3 6 6
288 Choice (D)
d2 = 14 + 2 48
31. Let the value of the given expression be x. The irrational parts of a2, b2, c2 and d2 are the same.
Comparing their rational parts, it can be seen that
We then have 420 + x = x 26 > 19 > 16 > 14
Squaring both sides and rewriting the equation obtained, ∴ c2 > b2 > a2 > d2
we have ∴c>b>a>d Choice (B)
x2 – x – 420 = 0
(x – 21) (x + 20)= 0
( ) ( )= 3
15 3K − 2 + 30 3K −1 K −2
(15 + 30 × 3)
x = 21 or – 20
As the square root is positive, x = 21. Ans: (21)
38.
15K − 2 ( )
3K − 2 5K − 2
105 21
 2n − 1 n − 1   2n − 1 n −1  = = Choice (D)
x − y2 x + y2  5K − 2 5K − 3
32.   
n n
x2 − y2 −4
−4  1 1 1 1 
 p − 4 pq   4 
 2n − 1 
2  2n − 1 
2
   p ×p4 − p4 × q4 
  − y  
 n n 39.
 4 pq − q 
=
 1 1 1 1 
x  x2 − y2    p4 × q4 − q4 × q4 
= n n
= n n
=1 Ans: (1)
 
x2 − y2 x2 − y2 −4
 1 1 1 
 p 4 p 4 − q 4   −4
a + 1 1 − a (a + 1)2 − (a − 1)2 4a     1
 4


33. + = = 2   
=   p q
a − 1 1+ a a2 − 1 a −1  =  = Choice (A)
 1 
( ) ( )  
1 1 1 p
   q4 
4 6 + 7 4 6 + 7  q 4 p 4 − q 4    
=
( 6 + 7) −1
=   
2
6 + 7 + 2 42 − 1   

4 ( 6 + 7 ) 2( 6 + 7 )
= =
12 + 2 42 6 + 42 81 − 77
40. x=

=
2 ( 6 + 7) = 2
Choice (D) 1
2
2 (
2 81 + 77 )
= =
81) (− 77 )
6 6+ 7 6 2
x 81 − 77 ( 2

34. Let the square root of 42 + 240 be p + q = ( 81 + 77 )


42 + 320 = 2( 81 + 77 ) 81 + 77
p + q = =
Squaring both sides, 81 − 77 2
1
42 +2 80 = p + q + 2 pq ⇒x+ = 9,
x
Equating the rational and irrational parts on both sides, 2
p + q = 42 - - (1) 1  1
x2 + =  x +  − 2 i .e., 92 − 2 = 79 Ans: (79)
pq =80 - - (2) x 2
 x
Solving (1) and (2),
Triumphant Institute of Management Education Pvt. Ltd. (T.I.M.E.) HO: 95B, 2nd Floor, Siddamsetty Complex, Secunderabad – 500 003.
Tel : 040–27898195 Fax : 040–27847334 email : info@time4education.com website : www.time4education.com SM1001920.Sol/56
Exercise – 23 (iii) log7(2744)2 – log764
(Logarithms)  ( 2744 )2 
= log7  
 64 
Solutions for questions 1 to 25:  
1. (i) log³√21024 = log73432 = 6 log77 = 6 Choice (D)
= log2 1/3210 log 25 log 27 log 16
= 10 x 3log22 (iv) × ×
log 3 log 64 log 125
= 30 Choice (C)
2 log 5 3 log 3 2 log 4
× ×
(ii) log 113 / 2 + log 64 / 3 log 3 3 log 4 3 log 5
114 62
= 4/3 Choice (C)
3 2
= log1111 + log66 2 3 3
8 3 4. log58 + log525 – log5320 + log532
3 2 25  8 × 25 2 × 32 3 
= + = Choice (B) = log5   = log55 = 1 Choice (B)
8 3 24  320 3 
 
(iii) log18 –1(2187)–1/2 5. log10(x – 2) + 3log1010 = log1080 + log10(11x + 5)
7 ⇒ log10[(x – 2) x 103] = log10[80 (11x + 5)]
− ⇒ (x – 2) 1000 = 80 (11x + 5)
= log 3 –43–7/2 = 2 log33 = 7 Choice (D)
−4 8 ⇒ 25x – 50 = 22x + 10 ⇒ x = 20 Ans: (20)

(iv) log
71 / 2
(7 )
3 9
+ log
72
(7–4)11 + log
74
728
6. log10(7x + 8) – 2log1010
= log10(x + 5) – log1025
7 27 + log 7 −44 + log 7 28  7x + 8  x+5
= log
71/ 2 72 74
log10   = log10  25 
 100   
=54 – 22 + 7 = 39 Choice (C)
(7 x + 8 ) ( x + 5 )
⇒ =
log 1728 100 25
2. (i) = log25x
log 144 ⇒ 7x + 8 = 4 (x + 5)
3 ⇒ 3x = 12
log 12
⇒ = log25x ⇒x=4 Ans: (4)
log 12 2
7. log5252 + log5252/3 + log5252/9 + …………. ∝
3/2 = log25x ⇒ x = 253/2 = 125 Choice (D)
log554 + log554/3 + log554/9 + …………. ∝
(ii) log[(x + 1) (x – 1)] = log15 4 4
⇒ log(x² – 1) = log15 = 4log55 + log55 + log55 + …………. ∝
3 9
⇒ x² – 1 = 15 ⇒ x = ±4
4 4
As log of negative numbers do not exist, x is 4. =4+ + + …………. ∝
Choice (B) 3 9
(iii) log5 + log(2x – 1) – log7 = log(x + 1) 4
= =6 Choice (C)
 5(2x − 1)  1
⇒ log   = log(x + 1) 1−
 7  3
 5( 2x − 1)  8. log(169)2 – log(143)3 + log(1100) – log(1300) + log(121)
⇒   = (x + 1)
 7  (169 ) 2 × 1100 × 121
= log = log(1) = 0 Choice (A)
⇒ 3x = 12 ⇒ x = 4 Choice (C) (143 ) 3 × 1300
7
(iv) log8(x – 139) = 11/3
x7 – 139 = (8)11/3 9. 0.81log82048 + 0.0081log82048 + …………….. + ∝
⇒ x7 = 2187 = 37 ⇒ x = 3 Choice (A) = log8(2048)0.81 + log8(2048)0.0081 + log8(2048)0.000081 +
…………….. + ∝
(v) log728 + log7(4x + 2) = log7(5x – 7) + 2 = log8[(2048)0.81 x (2048)0.0081 x ……….. ∝)]
⇒ log7 [28 (4x + 2)] = log7 [(5x – 7) x 7²] = log8(2048)0.81 + 0.0081 + ………….∝
⇒ 28 (4x + 2) = 7² (5x – 7) = log8(2048)0.818181…………..∝
⇒x=3 Choice (B) = log8(2048)9/11
= log8(211)9/11 = log8512 = 3 Choice (C)
log 34 × log 53
(vi) = log322x3
log 55 × log 33 10. log2(log2(log3(log12(1728)27)))
= log2(log2(log3(log121281)))
4×3 = log2(log2(log381))
= log322x3
5×3 = log2(log2(log334))
⇒ 2x = (32)4/5 ⇒ x = 2
3
Choice (D) = log2(log24)
= log22 = 1 Ans: (1)
3. (i) 4log2 + 5log3 – log144 – 3log6
 24 × 35 
= log   11. log(693)2 + log891 – log6561 + log2401 – log(49)3 – log( 1331 )2
 144 × 6 3 
   ( 693 )2 × 891 × 2401 
= log(1/8) = – log8 = –3log2 Choice (D) = log  
 6561 × 493 × 1331 
 
(ii) 3log21 + 3log20 – 3log180 – 3log7 + 3log3
= log213 + log203 – log1803 – log73 + log33 (9 × 7 × 11) 2 × 81 × 11 × 7 4
=
 213 × 20 3 × 3 3  3 8 × 7 6 × 113
= log   = log1 = 0 Choice (D) = log1 = 0 Choice (A)
 (180) 3 × 7 3 
 
Triumphant Institute of Management Education Pvt. Ltd. (T.I.M.E.) HO: 95B, 2nd Floor, Siddamsetty Complex, Secunderabad – 500 003.
Tel : 040–27898195 Fax : 040–27847334 email : info@time4education.com website : www.time4education.com SM1001920.Sol/57
log47 1296 − log47 216 21. log1331x × log25614641 × log2431024 = log35
12. = log625x
log47 92 + log47 24 log x log 14641 log 1024
= × × = log35
log 1331 log 256 log 243
 1296 
log 47   log x 4 log 11 10 log 2
 216  = log x ⇒ × × = log35
⇒ 625 3 log 11 8 log 2 5 log 3
log 47 (9 2 + 2 4 )
log x 4 10 1 log 5
log47 6 ⇒ × x x =
⇒ = log625x 3 8 5 log 3 log 3
log47 1296
⇒ logx = log53
⇒ log12966 = log625x
⇒ x = 125 Ans: (125)
⇒ 1/4 log66 = log625x
⇒ log625x = 1/4 log a log b log c
⇒ x = (625)1/4 = 5 Ans: (5) 22. Let = = =k
m−l m−n l−n
⇒ loga = k(m – l)
log 5 5 5 × log 3 3 7 × log 4 4 5 ⇒ a = 10km – kl
13.
log 6 6 5 × log (114 ) 5 Similarly
113 b = 10km – kn
5 log 5 5 × 7 log 3 3 × 5 log 4 4 and c = 10kl – kn
= b2 = (10km – kn)2 = 102km – 2kn
20
5 log 6 6 × log11 11 From the among the options, the one satisfying is
3 a2 x c2 = 102km – 2kl . 102kl – 2kn
5 × 7 × 5 × 3 21 = 102km – 2kl + 2kl – 2kn
= = Ans: (5.25) = 102km – 2kn = b2 Choice (D)
5 × 20 4
23. logy[(x – 5) (x² + 5x + 25)] = 3
14. log3125p X log925 X log343243 X log249 = 4 ⇒ logy(x3 – 53) = 3
⇒ log 5 p X log 2 5 2 X log 3 3 5 X log 2 7 2 = 4 ⇒ x3 – 53 = y3
5 3 7
⇒ x3 – y3 = 125 Ans: (125)
1 2 5
⇒ log5 p X . log3 5 X . log7 3 X 2 . log 2 7 = 4  x3 + 1
5 2 3 24. log216   – log216(x² – x + 1) = 1
 x  3
⇒ log5p X log3 5 X log7 3 X log2 7 = 6  
⇒ log2p = 6 ⇒ x = 26 = 64 Choice (B)  x 3 + 1  1
⇒ log216  =
 x( x 2 − x + 1)  3
 
1 1
15. + ( x + 1) 1
lognn + lognm + lognl log l + log m + log n ⇒ log216 =
l l l x 3
+
1 x +1
⇒ = (216)1/3 = 6
logmm + logml + logmn x
1 1 1 ⇒ x = 1/5 Choice (D)
= + + 3
lognlmn log lmn logmlmn 25. log21 – log7 + log(81/205) + log82 – log(3 /5) – log9
l
 21 × 81 × 82 × 5 
= loglmnn + loglmnl + loglmnm = log   = log2 Choice (A)
 205 × 7 × 33 × 9 
 
= loglmn lmn = 1 Choice (C)
Exercise – 24
16. log13[(x – 7) (x² + 7x + 49)] = 2 (Permutations and Combinations)
⇒ log13(x3 – 73) = 2 Solutions for questions 1 to 4:
⇒ x3 – 343 = 169 ⇒ x3 = 512 = 83
⇒x=8 Choice (D) 1. We know that nCr = nCn − r,
∴25C12 = 25C13, and 25C19 = 25C6
17. Given number is 36100. ∴ value of the required expression = 0 Ans: (0)
Applying log, the number becomes 100 (log 36).
2. a
100 (log 22 x 32) 1
100 [2 log2 + log3] = 100 [2(0.3010 + 0.4771)] 2 b
A B C
= 100 (1.5562) = 155.62 3 c
∴The number has 156 digits. Choice (D) 4
A person can travel from A to B using any of the four
 2x 108 
18. log  ×  = log32 highways 1, 2, 3, 4. Having reached B, he can travel to C
 3 48  using any of the three highways a, b, c i.e., a person can
2x 108 reach C from A via B using the highways in any of the
⇒ × =9⇒x=6 Ans: (6) following possible ways.
3 48
1, a ; 1, b ; 1, c ; 2, a ; 2, b ; 2, c ;
3, a ; 3, b ; 3, c ; 4, a ; 4, b ; 4, c ;
19. 8192 log = 213 log 6561 = 213/8log26561 Hence there are a total of 12 ways. Ans: (12)
256 6561 28
n
log 13 / 8 3. Pr = 120, nCr = 20
= 21 2( 6561) = (38)13/8 = 313 Choice (C) (nCr) r! = nPr; r! = 120/20 = 6 ⇒ r = 3
n(n - 1) (n - 2)
3125 3125 × 5 n
C3 = 20 ⇒ = 20 ; n(n−1) (n − 2) = 120
20. log (3125 )3125 = log555 = log55 3.2.1
5 25 25 25
From choices, n = 6 satisfies the above equation.
= 625 Ans: (625)
Ans: (6)
Triumphant Institute of Management Education Pvt. Ltd. (T.I.M.E.) HO: 95B, 2nd Floor, Siddamsetty Complex, Secunderabad – 500 003.
Tel : 040–27898195 Fax : 040–27847334 email : info@time4education.com website : www.time4education.com SM1001920.Sol/58
4. We want to consider 3 flags out of 6 different coloured Solutions for questions 13 and 14:
flags. As the order of the colours play a role in forming the
signals, ∴6P3 = 6 × 5 × 4 = 120 different signals can be 13. 5 persons can be seated (arranged) in 7 chairs in 7P5 ways.
generated. Ans: (120) Choice (C)

Solutions for questions 5 to 8: 14. Each ring can be positioned in 10 ways. Hence, the three
rings can be positioned in 103 ways. We have to try out all
The given word is MEADOWS and it contains 4 consonants and these to be sure of opening the lock.
3 vowels.
∴ The required number is 103. Choice (A)
5. If the word starts with letter ‘A’, then remaining 6 letters can
be arranged in the remaining 6 positions in 6! = 720 ways. Solutions for questions 15 and 16:
∴ The required number of words = 720 Choice (B) 15. Since one specified man is selected we have to choose
6. The number of words when the word starts with M = 6! 3 out of 9 which can be done in 9C3 = 84 ways.
The number of words when the word starts with M and Choice (C)
ends with W = 5!
∴ The required number of words = 6! – 5! = 5 × 5! = 600 16. Since one specified man is excluded, we have to choose
Choice (D) 4 out of 9 which can be done in 9C4 = 126 ways
Choice (C)
7. A E O + __ __ __ __
Solutions for questions 17 to 19:
As all the vowels are to be together, we treat the three
vowels as one unit. Now, there are 4 other letters, these 17. We have to choose 3 out of 6 Maharastians and 2 out of
4 letters and the unit of vowels can be permuted in 5! ways, 4 Punjabis. i.e. 6C3 × 4C2 = 20 × 6 = 120 Choice (C)
while the vowels can be permuted among themselves in
3! ways. 18. The committee consists of 2, 3 or 4 Punjabis.
The required number of words = 5! × 3! = 120 × 6 = 720 Hence, the total number of ways of forming the committee
Choice (B) = 4C2 × 6C3 + 4C3 × 6C2 + 4C4 × 6C1
= 120 + 60 + 6 = 186 Choice (A)
8. As no two vowels are together, we need to have a
consonant present in between two vowels which act as a 19. The committee consists of 1 or 2 Maharastrians.
separator for vowels. As there is no condition on Hence, the total number of ways = 6C1 × 4C4 + 6C2 × 4C3
consonants, we first arrange them and this can be done in = 6 + 15 × 4 = 6 + 60 = 66 Choice (B)
4! ways. Now, there are 5 possible positions for the
3 vowels as indicated below. Solutions for questions 20 to 22:
VCVCVCVCV
In these 5 positions 3 vowels can be arranged in 5P3 ways. 20. The number of ways to select two men and three women
∴ The required number of words = 4! × 5P3 = 5C2 ×6C3
= 24 × 120/2 = 1440 Choice (C) 5×4 6×5×4
= × = 200 Choice (B)
Solutions for questions 9 and 10: 2 ×1 3×2

9. As no two boys are together we need the presence of a girl 21. The number of ways to select three men and two women
between two boys such that one man and one woman are always selected
The seven girls can be arranged in 7! ways. _ G _ G _ G _ = Number of ways of selecting two men and one woman
G _ G _G _ G _. The 5 boys can now be arranged in 8 from four men and five women.
available gaps in 8P5 ways. ; ∴The total number of 4×3
= 4C2 × 5C1 = ×5
arrangements are 7! × 8P5. Choice (C) 2 ×1
10. The total number of letters = 9 and two of them are E, one I = 30 ways Choice (C)
and one O. There are 5 odd places. The vowels can be
4! 5 × 4 × 3 × 2 22. Total number of persons in the committee = 5 + 6 = 11
arranged in these 5 places in 5C4 × = = 60 ∴ Number of ways of selecting a group of eight persons = 11C8
2! 2
11× 10 × 9
ways. Now the consonants can be arranged in 5! = 120 = 11C3 = = 165 ways Choice (A)
ways. Total number of arrangements = 60 x 120 = 7200. 3×2
Ans: (7200)
Solutions for questions 23 to 38:
Solutions for questions 11 and 12:
23. The boy can select one trouser in nine ways.
11. As the numbers formed are divisible by 5, they have to end
The boy can select one shirt in 12 ways.
with either 0 or 5. The number of ways in each case is
∴The number of ways in which he can select one trouser
shown below
__ __ __ __ __ 0 = 5! = 120 and one shirt is 9 × 12 = 108 ways. Ans: (108)
__ __ __ __ __ 5 = 4 × 4! = 96 (∵ 1st place cannot 24. The number of letters in the given word is four.
be zero. It can be filled in 4 ways.) The number of three letter words that can be formed using
Hence, the total number of 6 digit numbers = 120 + 96 = 216 these four letters is 4P3 = 4 × 3 × 2 = 24. Ans: (24)
Choice (D)
12. As the number formed is even, they have to end with 25. Total number of letters = 8
0, 2, or 4. The number of ways in each case is shown Using these letters the number of 8 letter words formed is
8
below P8 = 8!. Choice (B)
__ __ __ __ __ 0 = 5! = 120
26. There are five letters in the given word.
__ __ __ __ __ 2 = 4 × 4! = 96 (∴ first place is a Consider 5 blanks --------
non zero digit) The first blank and the last blank must be filled with N and
__ __ __ __ __ 4 = 4 × 4! = 96, A and the remaining three blanks can be filled with the
∴ Total number of six digit even numbers is 120 + 96 + 96 remaining 3 letters in 3! ways.
= 216 + 96 = 312 Choice (C) ∴ The number of words = 3! = 6. Ans: (6)

Triumphant Institute of Management Education Pvt. Ltd. (T.I.M.E.) HO: 95B, 2nd Floor, Siddamsetty Complex, Secunderabad – 500 003.
Tel : 040–27898195 Fax : 040–27847334 email : info@time4education.com website : www.time4education.com SM1001920.Sol/59
27. The word MEADOWS has 7 letters of which 3 are vowels. The above 5 items can be arranged in 5! ways and AAEI
__ V __ V __ V __ 4!
As the vowels have to occupy even places, they can be can be arranged among themselves in ways.
arranged in the 3 even places in 3! i.e., 6 ways. While the
2!
consonants can be arranged among themselves in the ∴ Number of required ways of arranging the above letters
remaining 4 places in 4! i.e., 24 ways. Hence the total ways 4!
= 5! ×
are 24 × 6 = 144 Ans: (144) 2!
120 × 24
28. n items of which p are alike of one kind, q alike of the other, = = 1440 ways Ans: (1440)
r alike of another kind and the remaining are distinct can be 2
n!
arranged in a row in ways. Solutions for questions 39 and 40:
p! q! r!
The letter pattern ‘MESMERISE’ consists of 10 letters of 39. Two men, three women and one child can be selected in
which there are 2M’s, 3E’s, 2S’s and 1I and 1R. 4
C2 × 6C3 × 5C1 ways
9! 4×3 6×5×4
Number of arrangements = Choice (A) = × ×5
(2 ! )2 3 ! 2 ×1 3×2
= 600 ways Ans: (600)
29. The total number of stations is 20
From 20 stations we have to choose any two stations and 40. The number of ways of selecting three men, two women
the direction of travel (i.e., Hyderabad to Bangalore is and three children is = 4C3 × 6C2 × 5C3
different from Bangalore to Hyderabad) in 20P2 ways. 4×3×2 6×5 5×4×3
= × ×
∴ 20P2 = 20 × 19 = 380. Choice (D) 3 × 2 ×1 2 ×1 3 × 2 ×1
= 4 × 15 × 10 = 600 ways Ans: (600)
30. Since each ring consists of six different letters, the total
number of attempts possible with the three rings is Exercise – 25
= 6 × 6 × 6 = 216. Of these attempts, one of them is a (Probability)
successful attempt.
∴ Maximum number of unsuccessful attempts Solutions for questions 1 to 35:
= 216 – 1 = 215 Choice (C)
31. We can initially arrange the six boys in 6! ways. 1. The first 5 natural numbers are 1, 2, 3, 4 and 5.
Having done this, now there are seven places and six girls Out of these, 3 numbers can be selected simultaneously in
5
to be arranged. This can be done in 7P6 ways. C3 = 10 ways
The combinations which give a sum of 9 are: (2, 3, 4) and
Hence required number of ways = 6! × 7P6
(1, 3, 5).
Choice (B)
2 1
Hence, the required probability = = Ans: (0.2)
32. The given digits are six. 10 5
∴ The number of four digit numbers that can be formed
using six digits is 6P4 = 6 × 5 × 4 × 3 = 360 2. P(getting at most 3 heads)
Ans: (360) 10
C0 + 10C1 + 10C2 + 10C3
=
33. The given digits are 1, 2, 3, 5, 7, 9 210
A number is even when its units digit is even. Of the given 1+ 10 + 45 + 120
digits, two is the only even digit. =
Units place is filled with only ‘2’ and the remaining three 210
places can be filled in 5P3 ways. 176 11
= = Choice (A)
∴ Number of even numbers = 5P3 = 60. Ans: (60) 1024 64

34. Six members can be selected from ten members in 3. When a fair dice is rolled, the possible outcomes are 1, 2,
10
C6 = 10C4 ways (∴ nCr = nCn–r). Choice (B) 3, 4, 5 and 6.
Out of these, the outcomes 2, 3, 4, 5 and 6 are favourable
35. Total number of balls = 9 + 3 + 4 = 16 outcomes.
Two balls can be drawn from 16 balls in 16C2 ways. 5
Choice (C) Hence, the required probability = Choice (C)
6
36. The word contains five consonants three consonants and 4. The total number of outcomes when two dice are rolled
three vowels can be selected from five consonants in together is 6 × 6 = 36
5
C3 ways, two vowels can be selected from three vowels in The favourable combinations are:
3
C2 ways. (1, 1), (2, 4) and (4, 2)
∴ 3 consonants and 2 vowels can be selected in 5C2 ⋅ 3C2 Hence, the required probability
ways i.e., 10 × 3 = 30 ways. Ans: (30) 3 1
= = Choice (B)
37. We can select one boy from 20 boys in 20 ways 36 12
We can select one girl from 25 girls in 25 ways
∴ we can select a boy and a girl in 20 × 25 ways 5. We know that, the probability of a dice shows up an even
i.e., = 500 ways. Ans: (500) 1
number is .
2
38. In the word, "MATERIAL" there are three vowels A, I, E. So, when three dice are rolled together, P(at least one dice
If all the vowels are together, the arrangement is MTRL shows up an even number)
‘AAEI’ = 1 – P (None of the dice shows an even number)
Consider AAEI as one unit. The arrangement is as follows. 1 1 1 7
=1– × × = Ans: (0.875)
MTRLAAEI 2 2 2 8

Triumphant Institute of Management Education Pvt. Ltd. (T.I.M.E.) HO: 95B, 2nd Floor, Siddamsetty Complex, Secunderabad – 500 003.
Tel : 040–27898195 Fax : 040–27847334 email : info@time4education.com website : www.time4education.com SM1001920.Sol/60
6. The total number of ways of selecting a card from a pack of So, there are a total of 15 favourable cases.
cards = 52 15 5
We have 9 numbered cards in each suit numbered from 2 to 10 Hence, the required probability = =
Of these, the numbers 2, 3, 5 and 7 are prime numbers.
6 × 6 × 6 72
So, we have 4 favourable cards in each suit. Choice (C)
Hence, the required probability
13. Let A be the event that the number is divisible by 8.
4×4 4
= = Choice (D) 12
52 13 ∴n(A) = 12 and P(A) =
100
7. The total number of ways of selecting two books from the Let B be the event that the number is divisible by 12
shelf = 10C2 = 45. 8
∴n(B) = 8 and P(B) = .
The favourable combinations of the positions of the books 100
selected are : (1, 2), (2, 3), (3, 4), (4, 5), (5, 6), (6, 7), (7, 8), A ∩ B is the event that the number is divisible by both 8
(8, 9) and (9, 10). and 12 i.e. 24
Hence, the required probability
4
9 1 ∴n(A ∩ B) = 4 and P(A ∩ B) =
= = Ans: (0.2) 100
45 5
∴P(A ∪ B) = P(A) + P (B) – P(A ∩ B)
8. The total number of outcomes when two dice are rolled 12 8 4 16 4
⇒= + – = = Ans: (0.16)
together is 6 × 6 = 36. 100 100 100 100 25
The favourable combinations for getting a sum 9 are :
(3, 6), (4, 5), (5, 4) and (6, 3). 14. When four coins are tossed, only if two coins show heads,
The favourable combinations for getting a sum 10 are : and two coins show tails, the person gets `50. ∴When four
(4, 6), (5, 5) and (6, 4) coins are tossed the probability of getting two heads is
The favourable combinations for getting a sum 11 are : 4 C2 6 3
(5, 6) and (6, 5) 4
= = Ans: (0.375)
2 16 8
The favourable combinations for getting a sum 12 is:
(6, 6) only
15. In a leap year there are 366 days = 52 weeks + 2days. The
Hence, P (getting a sum of at least 9 on the dice)
is two days may be any one of the following possibilities
4 + 3 + 2 +1 10 5 {(sun, mon), (mon, tue), (tue, wed), (wed, thu), (thu, fri),
= = = Choice (B)
6×6 36 18 (fri, sat) (sat, sun) i.e. = 7.If the two days are (mon, tue)
then the year had 53 Mondays and 53 Tuesdays.
9. The total number of ways of entering and leaving the park 1
∴ the required probability = Choice (B)
= 10 × 10 = 100. 7
The favourable number of ways in which a person enters
the park through a gate and leaves the park through 16. The number of five digits numbers formed using the digits
another gate = 10 × 9 1, 2, 3, 4, 7 is 5! =120.If unit place is even then the
10 × 9 9 numbers formed are even Consider five blanks ------ Unit
Hence, the required probability = = place can be filled in 2 ways (2 or 4). Remaining four
10 × 10 10 places can be filled with four digits in 4! ways. Total number
Ans: (0.9) of five digit even numbers are = 2 × 4! ∴Required
2 × 4!
10. The total number of outcomes when two dice are rolled probability = = 2/5 Ans: (0.4)
together = 6 × 6 = 36.
5!
The favourable combinations are: 17. One number can be selected from the given set in 10 ways
(1, 2), (1, 3), (1, 4), (1, 5), (1, 6)
The favourable outcomes are {5, 8, 9} i.e. = 3 ∴Required
(2, 3), (2, 4), (2, 5), (2, 6)
probability =3/10. Ans: (0.3)
(3, 4), (3, 5), (3, 6)
(4, 5), (4, 6)
18. Two balls can be drawn from 8 balls in 8C2= 28 ways. The
(5, 6)
two balls may be red or white. The number of ways of
15 drawing two red balls is 5C2 = 10. Number of two white balls
Hence, the required probability =
36 = 3C2 = 3 favoruable outcomes = 10 + 3 =13 Required
5 13
= Choice (C) probability = Choice (B)
12 28

19. (i) The probability of drawing black ball from 6 black balls
11. The total number of ways of drawing 4 cards from a pack
and seven green balls = 6/13.Since first ball is
= 52C4.
replaced, the number of balls in the urn is 13. The
The number of ways of selecting 2 diamonds and 2 spades
probability of drawing a green ball in the second draw
= 13C2 × 13C2
is 7/13.Hence the required probability is
13
C2 × 13
C2 6 7 42
Hence, the required probability =
52 × = Choice (A)
C4 13 13 169
Choice (C) (ii) If the first ball is not replaced then remaining balls in
the bag is (5 + 7) = 12 The probability of drawing
12. The total number of possible outcomes when three fair dice 6 7
are rolled together = 6 × 6 × 6 green ball is 7/12. ∴required probability = ×
13 12
The favourable combinations in which exactly two dice
show the number 6 are listed below: = 7/26 Choice (D)

Ι ΙΙ ΙΙΙ
2 5 3
20. P(A ∪ B) = P(A) + P(B) –P(A ∩ B) = + −
6 6 1, 2, 3, 4, 5 3 6 5
6 1, 2, 3, 4, 5 6 20 + 25 − 18 27 9
1, 2, 3, 4, 5 6 6
= = = Ans: (0.9)
30 30 10
Triumphant Institute of Management Education Pvt. Ltd. (T.I.M.E.) HO: 95B, 2nd Floor, Siddamsetty Complex, Secunderabad – 500 003.
Tel : 040–27898195 Fax : 040–27847334 email : info@time4education.com website : www.time4education.com SM1001920.Sol/61
21. The total number of balls in the urn = 5 + 3 + 4 = 12 27. To get a total sum of 3, the favourable combinations are:
So, two balls can be selected in 12C2 = 66 ways. (1, 2) and (2, 1)
The number of ways of selecting a blue ball and a red ball But the face ‘1’ never turns upon the die.
= 5C1 × 3C1 = 15 Hence, the required probability = 0. Ans: (0)
The number of ways of selecting a red ball and a green ball
= 3C1 × 4C1 = 12 28. Given, P(A winning the race) = 0.4 and P(B winning the
Number of ways of selecting a blue ball and a green ball race) = 0.55
= 5C1 × 4C1 = 20 But P(A) + P(B) + P(C) = 1
Hence, the required probability So, 0.4 + 0.55 +P(C) = 1
15 + 12 + 20 47 ⇒ P(C) = 1 – 0.95 = 0.05
= = Choice (C) Hence, the probability of C winning the race is 0.05
66 66 Ans: (0.05)

22. Given, the head appears twice as frequently as tail. 29. The total number of ways of selecting a natural number
So, P(H) = 2 × P (T) from 1 to 100 = 100.
Let P(T) be x, then P(H) = 2x L.C.M. (4, 6) = 12
But P(H) + P (T) = 1 ∴ the numbers which are multiples of both 4 and 6 are 12,
So, 2x + x = 1 24, 36, 48, 60, 72, 84 and 96, which are 8 in number.
1 8 2
⇒x= Hence, the required probability = = Choice (A)
3 100 25
2 1
∴ P(H) = and P(T) =
3 3 30. The total number of possible outcomes when 10 coins are
So, P(getting HH or TT) = P (HH) +P (TT) tossed together = 210.
The number of outcomes in which the head appears odd
2 2 1 1 5
= × + × = Choice (B) number of times
3 3 3 3 9
= 10 C1 + 10 C3 + 10 C5 + 10 C7 + 10 C9 = 210 – 1 = 29
23. The total number of possible outcomes when three dice are 29 1
rolled together = 6 × 6 × 6 Hence, the required probability = =
The favourable combinations of getting a total score of 4 210 2
are : (1, 1, 2), (1, 2, 1), (2, 1, 1) Ans: (0.5)
The favourable combinations of getting a total score of 17
are : (6, 6, 5), (6, 5, 6), (5, 6, 6) 31. The total number of possible outcomes = 6 × 6 × 6
Hence, P(Sum = 4 or 17) = P(Sum = 4) + P(Sum = 17) The favourable combinations of getting a triplet are:
3+ 3 1 (1, 1, 1), (2, 2, 2), (3, 3, 3), (4, 4, 4), (5, 5, 5) and (6, 6, 6).
= = Choice (A) 6 1
6 × 6 × 6 36 Hence, the required probability = =
6 × 6 × 6 36
24. Every questions can be attempted in 3 ways. Choice (B)
So, the number of ways of attempting the entire paper
= 34 = 81 32. A number which is a perfect square will have an odd
There is only one way in which the person can answer all number of factors. Hence, the required probability
the questions correctly. 10 1
= = Ans: (0.1)
1 100 10
Hence, the required probability = Choice (B)
81
33. P(at least one die shows the number 1) = 1 – P (None of
25. The total number of ways of drawing a card from a pack of the dice show the number 1)
cards = 52 5 5 5 125
There are 13 diamonds and 4 aces in a pack. =1– × × ⇒=1–
6 6 6 216
But one card belongs to both the diamonds and aces.
So, the favourable number of cards = 13 + 4 – 1 = 16 216 − 125 91
= = Choice (C)
16 4 216 216
Hence, the required probability = =
52 13
34. There are 25 primes and 74 composite numbers from 1 to
Choice (D)
100.
26. The total number of possible outcomes when two coins and ∴P(selecting a prime number and a composite number)
two dice are rolled together = 22 × 62 25 C1 × 74 C1 37
= = Choice (B)
= 4 × 36 100 C2 99
The two coins can show the same face in 2 ways (HH or
TT).
The two dice can show the same faces in 6 ways. (11, 22, 35. The total number of ways of drawing two cards in
33, 44, 55 or 66). succession without replacement = 52 × 51.
2 ×6 1 ∴P(selecting the diamond first and then secondly a spade)
Hence, the required probability = = 13 × 13 13
4 × 36 12 = = Choice (D)
52 × 51 204
Choice (C)

Triumphant Institute of Management Education Pvt. Ltd. (T.I.M.E.) HO: 95B, 2nd Floor, Siddamsetty Complex, Secunderabad – 500 003.
Tel : 040–27898195 Fax : 040–27847334 email : info@time4education.com website : www.time4education.com SM1001920.Sol/62

You might also like